ISC Business Studies Question Paper 2019 Solved for Class 12

ISC Business Studies Previous Year Question Paper 2019 Solved for Class 12

Maximum Marks: 80
Time allowed: Three hours

  • Candidates are allowed additional 15 minutes for only reading the paper. They must NOT start writing during this time.
  • Answer Question 1 (Compulsory) from Part 1 and five questions from Part II, choosing two questions from Section A, two questions from Section B and one question from either Section A or Section B.
  • The intended marks for questions or parts of questions are given in brackets [ ].

Question 1.
Answer briefly each of the questions (i) to (x): [10 x 2]
(i) Name the two sources of e-recruitment. Briefly explain any one of them.
(ii) State any two differences between training and development.
(iii) Briefly explain the two types of esteem needs, as per the human needs stated by Maslow.
(iv) Define Leadership.
(v) State any two limitations of promotion of employees in an organisation.
(vi) What is meant by gratuity ?
(vii) State any two benefits of e-business to the buyers.
(viii) What is meant by outsourcing ?
(ix) State any two functions of FSSAI.
(x) Expand the following :
(a) LPO
(b) KPO
Answer:
(i) Two sources of e-recruitment are :

  1. Resume Scanners
  2. Job portals

Job Portals: A job portal, also known as a career portal, is a modem name for an online job board that helps applicants find jobs and aids employers in their quest to locate ideal candidates. Some government agencies, non-profit organizations, universities and private businesses have their own job portals that applicants can access on the organization’s website.

(ii)

Training Development
(i) Training is a short term process. (i) Development is a long term process.
(ii) Focus on present or immediate needs. (ii) Focus on future role.
(iii)It is aimed at a specific task or job role (iii) It is aimed at developing relationships, often for the purpose of improving leadership skills.

(iii) Esteemed needs are : Status, power, confidence, self-esteem, achievement, respect, etc. Status : When a person has status, they are widely recognized and are effectively given social power through the influence that is gained. Status may be gained through basic admiration from others. It is also gained by position,

Esteem as a reward:
Esteem : Esteem is often used as a reward. An esteem reward may be a promotion or a ‘lifelong achievement’ award. Although money is useful, but beyond a reasonable salary, many people are not strongly influenced by cash incentives.

Esteem often acts as an intrinsic reward, and thus can be far more powerful than extrinsic rewards such as bonus payments or loyalty card points.

(iv) Leadership is the art of motivating a group of people to act towards achieving a common goal. In a business, this can mean directing workers and colleagues with a strategy to meet the company’s needs.Esteem : Esteem is often used as a reward. An esteem reward may be a promotion or a ‘lifelong achievement’ award. Although money is useful, but beyond a reasonable salary, many people are not strongly influenced by cash incentives. Esteem often acts as an intrinsic reward, and thus can be far more powerful than extrinsic rewards such as bonus payments or loyalty card points.Esteem as a reward:(iii) Esteemed needs are : Status, power, confidence, self-esteem, achievement, respect, etc.
Status : When a person has status, they are widely recognized and are effectively given social power through the influence that is gained. Status may be gained through basic admiration from others. It is also gained by position. such as when a person gets promoted within an organization.

(v) (a) Since the promotions are to be given to the existing employees, minimum standard of qualifications has to be ignored, if highly qualified employees are not available.
(b) Selection from the existing employees is a difficult process if the employees have limited knowledge and skills.

(vi) Gratuity is a monetary benefit given by the employer to his employee at the time of retirement. It is a defined benefit plan where no contributions are made by the employee.

(vii) (a) Products at competitive price are available on e-business platforms.
(b) A buyer can order for the product any time (24 × 7 basis).
(c) Products are delivered at doorsteps of the buyers.

(viii) (a) Outsourcing is cost effective to a firm.
(b) Outsourcing can promote efficiency. It can be a good option when the functions of an office are complicated.
(c) Outsourcing offers staffing flexibility.

(ix) Functions of Food Safety and Standards Authority of India are :

  • To promote general awareness about food safety and food standards.
  • To provide training to the persons who are involved or intend to get involved in food businesses.

(x) (a) LPO : Local Purchase Order
(b) KPO : Knowledge Process Outsourcing

Part – II (60 Marks)
Answer any five questions

Question 2.
(a) Briefly explain the significance of Human Resource Management to an enterprise. [4]
(b) Define staff selection. Explain the meaning and relevance of checking references and placement in a selection procedure. [8]
Answer:
(a) An organization cannot build a good team of working professionals without good Human Resources. The key functions of the Human Resources Management (HRM) team include recruiting people, training them, performance appraisals, motivating employees as well as workplace communication, workplace safety and much more.

HRM develops skills of the employees.

The HR department teaches the organization how to utilize this human capital as well as the non-human one and so, the organizations, who aim to efficiently utilize their resources and increase the profit invite the HR management people to formulate required objectives and policies.

HRM resolves the conflicts and helps in maintaining and restoring positive working relationships.

It has the responsibility of developing good public relations as well as maintaining the goodwill of the company among other public sectors.

(b) Staff selection is the process of interviewing and evaluating the candidates for a specific job and selecting an individual for employment based on certain criteria (qualifications, skills and experience).

Checking references: The person who gives the reference of a potential employee is also a very important source of information. The referee can provide info about the person’s capabilities, experience in the previous companies and leadership and managerial skills. The information provided by the reference person is meant to kept confidential with the HR department.

Relevance of checking references : Reference checking allows you to ensure that you are finding the most qualified person. By conducting reference checks, you can avoid costs associated with poor performance. These checks help in confirming information provided by the applicant in application form and resume. It can help in looking into insights of the candidate’s skills, knowledge and abilities from someone who has actually observed the candidate.

Placement : Placement is the process of assigning a specific job to each one of the selected candidates. Once an employee has been selected, he/she should be placed on a suitable job. Putting the right man at the right job is an important as hiring the right person. A misplaced employee remains dissatisfied and frustrated.

Relevance of placement:

  • reduces employee turnover
  • reduces absenteeism
  • improves morale

Question 3.
(a) What is campus recruitment ? How does campus recruitment benefit the students as well as business houses ? [4]
(b) Explain any four methods of raising staff morale. [8]
Answer:
(a) Campus Placements/Campus recruitment drives are conducted in various educational institutes for providing job opportunities to the students who are pursuing their particular academic courses. Campus placements offer a student a wonderful opportunity to get placed during the course of his academic pursuits, and provide him the comforts of a safe and secure future. Keeping in mind the importance of the campus placement programs, it is vital for a student to prepare adequately for these programs and make sine that they put their best foot forward.

Benefits for Business Houses: The business houses are benefited from getting wide choice of candidates to select for different job posts. They can select the right and talented candidate from a vast pool of young applicants within a limited time.

Campus recruitment helps in saving time and efforts of the business houses : An organization through effective campus recruitment finds an opportunity to establish a link with the next batch of students.

This in turn paves way to serve the future and long term recruitment needs of the business house. Students participating in internships and summer training programs may have direct recruitment to different job positions offered by the business house.

Advantages to the students : Students have the advantage of getting a good job according to their qualification level even before the completion of their academic course in college. Campus recruitment helps in increased selection ratio. More number of quality candidates can be selected through this recruitment process. The students have not to apply at various places for a job.

(b) Methods of Raising Morale :
Monetary Incentives : Monetary incentives like good salaries and wages help to encourage the employees for greater productivity. The management should evolve such a wage/salary structure which enables to workers to earn at a reasonable level and to live a decent life. Assurance of a sound wage structure builds up morale of workers.

Non-Monetary Incentives: Non-monetary incentives are also helpful in boosting the morale of employees. These incentives may be provided to them in the form of medical aid, recreational facilities, holiday homes, creche facility to the children of working couples etc.

Job Security : Security of job boosts the morale of the employees. The management should follow a good labour policy that ensures the security of the job to its employees. The workers should also be assured that in case their services are not required in their present jobs, they will be absorbed in alternative jobs sooner or later or immediately.

Sound Promotion Policy : A sound promotion policy is a must to boost the morale of the employees. Promotion policy should be so adopted by the management that it gives proper weight age to merit and seniority. If only one base either seniority or merit is adopted, it will not be good for the morale of the employees.

Healthy Working Conditions : The working conditions in the organisation play a . crucial role in boosting the morale of the employees. Working conditions should be healthy so that employees may work with best efforts and attain organisational objectives.

Question 4.
(a) What is meant by Induction Training ? State any two objectives of this training. [4]
(b) Briefly explain any four types of leave that may be availed by an employee. [8]
Answer:
(a) Induction is the process of receiving and welcoming an employee when he first joins a company and giving him basic information he needs to settle down quickly and happily and start work. The main purpose of induction training is to integrate new employees into the company and make them understand the systems and procedures followed by the organization. Induction training helps new employees settle down quickly in the new work environment, and gives them a sense of belonging.

  1. To reduce the initial anxiety all new entrants feel when they join a new job in a new organization.
  2. To familiarize the new employees with the job, people, work-place, work environment and the organization.

(b) Methods of Raising Morale :
Monetary Incentives : Monetary incentives like good salaries and wages help to encourage the employees for greater productivity. The management should evolve such a wage/salary structure which enables the workers to earn at a reasonable level and to live a decent life. Assurance of a sound wage structure builds up morale of workers.

Non-Monetary Incentives: Non-monetary incentives are also helpful in boosting the morale of employees. These incentives may be provided to them in the form of medical aid, recreational facilities, holiday homes, creche facility to the children of working couples, etc.

Job Security : Security of job boosts the morale of the employees. The management should follow a good labour policy that ensures the security of the job to its employees. The workers should also be assured that in case their services are not required in their present jobs, they will be absorbed in alternative jobs sooner or later or immediately.

Sound Promotion Policy: A sound promotion policy is a must to boost the morale of the employees. Promotion policy should be so adopted by the management that it gives proper weight age to merit and seniority. If only one base either seniority or merit is adopted, it will not be good for the morale of the employees.

Question 5.
(a) Write a short note on the Situational Leadership. [4] .
(b) Explain the components of 360° appraisal used to appraise staff in an organisation. [8]
Answer: .
(a) Situational leadership is an adaptive leadership style. This strategy encourages leaders to take stock of their team members, weigh the many variables in their workplace and choose the leadership style that best fits their goals and circumstances.

(b) 360° appraisal has four integral components :

  1. Self appraisal
  2. Superior’s appraisal
  3. Subordinate’s appraisal
  4. Peer appraisal

Self-appraisal gives a chance to the employee to look at his/her strengths and weaknesses, his achievements, and judge his own performance.

Superior’s appraisal forms the traditional part of the 360° performance appraisal where the employees’ responsibilities and actual performance is rated by the superior.

Subordinate’s appraisal gives a chance to judge the employee on the parameters like communication and motivating abilities, superior’s ability to delegate the work, leadership , qualities etc.

The correct feedback given by peers can help to find employees’ abilities to work in a team, co-operation and sensitivity towards others. Self-assessment is an indispensable part of 360° appraisals and therefore 360° Performance appraisals have high employee involvement and also have the strongest impact on employees.

Question 6.
(a) State any two advantages and any two disadvantages of merit based promotion. [4]
(b) Give any eight distinctions between oral communication and written communication. [8]
Answer:
(a) Advantages of merit based promotion :

  1. Efficiency and talents are rewarded in merit based promotion, therefore, it encourages the employees to increase their knowledge in maintaining a high level of productivity.
  2. It helps in putting the most productive and competent person for specialized jobs.

Disadvantages of merit based promotion :

  1. It may result in partiality, nepotism, favoritism, etc. by the management.
  2. Senior employees in the organization may become unsatisfied and frustrated and they may indulge in certain activities that may lead to industrial unrest, strikes, lockouts, etc.

(b)
ISC Business Studies Question Paper 2019 Solved for Class 12 1

Question 7.
(a) What is meant by retirement ? Explain the two types of retirement. [4]
(b) With reference to business finance, explain the following : [8]
(i) Preference shares
(ii) Debentures
(iii) Retained earnings
(iv) Public Deposits
Answer:
(a) Retirement means to withdraw from one’s position or occupation or from active working life. You can achieve retirement when you have sources of income that do not have to be earned by working. Two types of retirements are :

Superannuation : Superannuation refers to retirement you take after you have reached a predetermined age. In most countries, the government sets this age.

Voluntary retirement : Government servants may retire from service voluntarily with pensionary benefits before attaining the age of superannuation.

(b ) (i) The company law defines Preferences Shares as that part of share capital which enjoys the preferential right:

  • as to receive dividend at a fixed rate during the life of a company.
  • as to return of capital on winding up of the company. Preferential shareholders can enforce their right of getting dividend on priority over the equity shareholders only when the dividend is declared.

(ii) A debenture is an instrument of written acknowledgement of debt under the common seal of the company.

It consists of the contract for repayment of the money borrowed after a specific time and at a specified rate of interest. The person holding this written acknowledgement is called Debenture holder.

According to section 2 (12) of the Indian Companies Act, Debenture includes debenture stock, bonds and any other securities of a company whether constituting a charge on the assets of the company or not.

(iii) Retained earnings are the profits that a company has earned to date, less any dividends or other distributions paid to investors. This amount is adjusted whenever there is an entry to the accounting records that impacts a revenue or expense account. A large retained earnings balance implies a financially healthy organization. The formula for ending retained earnings is :
Beginning retained earnings + Profits/losses – Dividends = Ending retained earnings

(iv) A company can accept deposits from the public to finance its medium and short term requirements of funds. The companies always prefer to accept public deposits for meeting their financial needs, than approaching banks or other financial bodies. Companies generally receive public deposits for different periods ranging from 6 months to 5 years. The rate of interest on the deposits varies from 10 per cent per annum to 12 per cent per annum depending upon the period of deposits and the reputation and standing of the company.

Question 8.
(a) Explain B2C and C2C as online means of conducting business.
(b) Briefly explain any four types of reports.
Answer:
(a) Business to Consumer (B2C) : In B2C business model, a website sells its product directly to a customer. A customer can view products shown on the website of business organization. The customer can choose a product and order the same. Website will send a notification to the business organization via E-Mail and organization will dispatch the product/goods to the customer.

Consumer to Consumer (C2C): The C2C business model helps the consumer to sell their assets like residential property, cars, motorcycles etc. or rent a room by publishing their information on the website. Website may or may not charge the consutiler for its services. Another consumer may opt to buy the product of the first customer by viewing the post/advertisement on the website.

(b) Four types of reports are :
Formal Report: Formal report is prepared in a prescribed format and presented before the management in an established procedure. Reports submitted by officials of Companies are usually formal report.

Informal Report:
Informal report is prepared in a format of the convenience of the reporter and presented directly before the required person as and when demanded. An informal report is presented as in the form of letter or memorandum. Generally, it takes the form of a person to person communication.
The formal reports are classified into two types. They are statutory report and non- statutory report.

Special Report:
Special report is prepared and presented before the top management on specific requirement. It usually contains the opinions or recommendations of the reporter with the help of facts and arguments. Examples for special report are opening of branch, introducing a new product, improving the quality or changing the shape or size of the product etc.

Technical Report:
Technical reports describe the process, progress, or results of technical or scientific research, include in-depth experimental details, data, and results.

Question 9.
(a) Discuss any four ways in which globalization has transformed the manner in which business is conducted. [4]
(b) Draft a letter of appointment for a school teacher, offering him/her a job of a History teacher in a secondary school. [8]
Answer:
(a) Globalization means integrating the economy of a country with the economics of other countries. It is possible through free flow of trade, capital and movement of persons across borders. Globalization has transformed the manner in which business is conducted. Let us have a look on these ways:

  • Due to globalization, it has been possible to invest in newer technologies and production methods which have raised the production standards. Foreign trade has increased in the country.
  • Exchange of technology between countries has been possible.
  • Collaborations with foreign companies has been possible.
  • Due to improved products, it has become possible to compete and survive in international competition.
  • Due to globalization, many of the Indian companies have emerged as multinationals such as Tata Motors and Asian Paints.

(b)

ISC Business Studies Question Paper 2019 Solved for Class 12 2

ISC Class 12 Business Studies Previous Year Question Papers

ISC Political Science Question Paper 2010 Solved for Class 12

ISC Political Science Previous Year Question Paper 2010 Solved for Class 12

Maximum Marks: 80
Time allowed: Three hours

  • Candidates are allowed additional 15 minutes for only reading the paper. They must NOT start writing during this time.
  • Answer Question 1 (Compulsory) from Part I and five questions from Part II, choosing two questions from Section A, two questions from Section B and one question from either Section A or Section B.
  • The intended marks for questions or parts of questions are given in brackets [ ].

Part—I
(Compulsory)

Question 1.
Answer briefly each of the questions (i) to (xv): [15 x 2]
(i) How are states classified on the basis of territorial division of authority and on the basis of the nature of use of authority ?
(ii) What is a confederation ?
(iii) Differentiate between a totalitarian state and an authoritarian state.
(iv) What is the position of the cabinet in a Presidential system of government ?
(v) Explain the meaning of the term convention.
Give one example.
(vi) Classify the constitutions on the basis of their amending processes. Give examples.
(vii) Why did Montesquieu advocate the Theory of Separation of Powers ? What does the theory imply ‘
(viii) How does the President of the USA exercise checks upon the Judiciary ?
(ix) Explain Cumulative Vote System.
(x) What is functional representation ?
(xi) Explain what is meant by the term pocket veto of the U.S. President.
(xii) Define Rule of Law.
(xiii) Mention the special powers enjoyed exclusively by the Rajya Sabha.
(xiv) Name the apex judicial institutions in the U.K. and India.
(xv) What is the meaning of regional imbalance
Answer:
(ii) Confederation is an association of sovereign member states that by treaty have delegated certain of their competences to common institutions, in order to coordinate their policies in a number of areas, without constituting a new state on top of the member states. It is a permanent union of sovereign states for common action in relation to other states.

(iv) The Cabinet in the United States is in fact the President’s family : It is he who makes his Cabinet and he can also unmake it at his will. The Cabinet has a character and importance of its own. Membership in it continues to be an ambition of many politicians because the Cabinet examines and discusses broad policies of the Government.

While evaluating the role of the American cabinet it may be added that it is a body of advisers and not a council of his colleagues with whom he has to work and upon whose approval he depends.

(vi) A Flexible Constitution is one which can be easily amended. Many political scientists define a Flexible Constitution as one in which the Constitutional law can be amended in the same way as an ordinary law is made by the Legislature. Britain presents a classic example of a Flexible Constitution. The Rigid Constitution is one which cannot be easily amended. Its method of amendment is difficult. For amending it, the Legislature has to pass a proposal by a specific, usually big majority of 2/3rd or 3/4th or absolute majority, i.e., 51% of votes. The American Constitution is a classic example of Rigid Constitution.

(vii) The framers of the American Constitution wanted to prevent separate branches from encroaching upon one another, and to avert deadlock, the fathers of the Constitution provided an elaborate system of checks and balances. The judicial organ is checked by the fact that all the Judges are appointed ‘by the President with the approval of the Senate.’ Then, all judges can be impeached. The Congress can determine the size of the courts.

(xi) A Pocket Veto occurs when a Bill fails to become law because the President does not sign the Bill and cannot return the Billto Congress within a 10-days period because Congress is not in Session. Article 1, Section 7 of the U.S. Constitution States : ‘If any Bill shall not be returned by the President within ten days (excluding Sundays) after it shall have been presented to him, the same shall be a law, in like manner as if he had signed it, unless the Congress by their Adjournment prevent its return, in which case it shall not be a law.’

(xiii) Two powers enjoyed exclusively by the Rajya Sabha are:
The power to declare a subject of State List as a subject of national importance. Under Article 249, the Rajya Sabha can pass a resolution by 2/3rd majority of its members for declaring a subject of the State List as a subject of national importance.

Power in respect of creation or abolition of an All India Service. Art. 312 of the Constitution empowers the Rajya Sabha to create one or more new All India Services. It can do so by passing a resolution supported by 2/3rd majority on the plea of national interest.

Part—II
Section—A
Answer two questions

Question 2.
(a) What were the criteria used by C.F. Strong for ‘classification of States’ ? Give examples. [8]
(b) Critically evaluate Aristotle’s ‘classification of States’. [6]

Question 3.
(a) Distinguish between Unitary and Federal forms of Governments. [8]
(b) Discuss six conditions which are essential for the successful working of a federation. [6]
Answer 3.
(a) Unitary Government : Power is almost entirely centralized in a national Government. Power devolves to local Governments only for the sake of convenience (such as garbage collection times or issuing parking tickets). Any local Governments that exists hold power only with permission from the national Government, and they cannot ever conflict with national policy. Thus, in the UK, the counties into which England is divided have their own bureaucracies and regulations, but only in areas where the national Parliament has given them permission to set up those systems. Unlike a Federal system, there are no reserved powers for states or provinces.

Local power may be granted by the national legislature, but it can be modified or revoked. Unitary nation-states might set up regional Governments, but only to handle regulations that might be too burdensome to control entirely from a nationwide bureaucracy. Unitary Governments are relatively common.

Federal Government : The central (or federal) Government controls more trade policy, and makes decisions about policy areas that involve interactions between states (such as highway systems). It usually has the power to tax independently of the states and to control the money supply. A Federal Government also usually has its own mechanisms for enforcement. For example, in the USA, the FBI is the primary agency for investigating Federal crimes and crimes that occur between or among multiple states.

The Royal Canadian Mounted Police have a similar function in Canada. Federal systems (or federations) are more common than Confederal Governments today. The USA, Germany and Canada are Federal, and Russia is a rather centralized federation. Federal systems differ in how much power they give to the Federal, as opposed to the regional Governments, but they are all alike in that some powers are reserved at each level of Government in a balancing act.

Question 4.
(a) Define Constitution and discuss the various kinds of Constitutions. [8]
(b) Discuss the merits and demerits of the Presidential form of Government. [6]

Answer 4.
(a) In simple words, we can define a Constitution as the basic design of the organisation and power of the Government. A modern Constitution necessarily involves a charter of the rights and duties of its citizens. It is a body of the Constitutional Law of the state, the supreme and fundamental law of the state. Government is organised and it performs its functions according to the Constitution of the state. It lays down the nature of relations between the people with their Government. Woolsey defines a Constitution as “the collection of principles according to which the powers of the Government, the rights of the governed and the relations between the two are adjusted.”

Kinds of constitution are as follows :
Written Constitution : A written constitution means a Constitution written in the form of a book or a series of documents combined in the form of a book. It is a consciously planned and enacted Constitution which is formulated and adopted by a constituent assembly or a council or convention or a legislature. “A written Constitution”, observes Dr. Garner, “is a consciously planned Constitution, formulated and adopted by deliberate actions of a constituent assembly or a convention.” It provides for a definite design of Government institutions, their organisations, powers, functions and inter-relationships. It embodies the constitutional law of the state. It enjoys a distinct character, special sanctity and place of supremacy. The Government is bound by its provisions.

Unwritten Constitution : An unwritten Constitution is one which is neither drafted nor enacted by a Constituent Assembly and nor written in the form of a book. It is a product of slow and gradual evolution. The Government is organised and it functions in accordance with several well settled laws and several unwritten conventions. The people know their constitution and they accept and obey it, but they do not possess it in a written form.

Flexible Constitution : Constitutions can be classified on the basis of the nature of their amending processes into Flexible and Rigid constitutions. A Flexible Constitution is one which can be easily amended. Many political scientists define a flexible constitution as one in which the constitutional law can be amended in the same way as an ordinary law is made by the legislature. Britain presents a classic example of a flexible constitution.

Rigid Constitution : The Rigid Constitution is one which cannot be easily amended. Its method of amendment is difficult. For amending it, the legislature has to pass a proposal by a specific, usually big majority of 2/3rd or 3/4th or absolute majority, i.e., 51% of votes. For passing or amending an ordinary law, the legislature passes the measure by a simple majority of the members present. The constitutional law is considered the fundamental law of the land or the supreme law of the land.

Evolved Constitution : An Evolved Constitution is one which is not made at any time by any assembly or persons or an institution. It is the result of slow and gradual process of evolution. Its rules and principles draw binding force and strength from the fact of their being recognized as ancient, time tested and respected customs and conventions. Some of these conventions stand recognized by law and hence become enforceable while others are followed because of the sanctions of public opinion, practical utility and moral consciousness in their favor. The Constitution of Great Britain presents a classic example of an evolved constitution.

Enacted Constitution : Unlike an Evolved Constitution, an Enacted Constitution is a man-made constitution. It is made, enacted and adopted by an assembly or council called a Constituent Assembly or Council. It is made like a law after full discussions over its objectives, principles and provisions. The Constituent Assembly is constituted by the representatives of the people and it drafts and enacts the constitution.

The Presidential Government is a form of government in which the executive is constitutionally independent of the legislature and the legislative of the executive in respect to their policies and powers and duration of their tenure. The government of the U.S.A. is the best example of the presidential type. The President is the executive head and is indirectly elected by the people for a specified term fixed by the constitution.

(b) Merits:

  1. It is a stable government in which the tenures of the executive and legislature are fixed.
  2. It is a strong government because of the fact that the executive is independent of day to day interferences by the Legislature and because the President has the power to take all decisions.
  3. There is continuity in governmental policies because the tenure of the executive is definite and fixed.
  4. It makes possible a government involving professionals and experts because the cabinet is constituted by the President strictly on merit.
  5. It keeps contained the party politics to the Legislative sphere.
  6. Because of being a strong executive, it is more conducive to the preservation of unity and integrity of the nation.

Demerits:

  1. It depends upon the qualities of the single person who becomes the President.
  2. The President can behave arbitrarily because he wields a large amount of power both as the head of state as well as of the Government.
  3. Executive is not responsible before the Legislature.
  4. Due to separation of powers, there are frequent deadlocks between the legislature and executive.
  5. These are a source of inefficiency and mutual mud-slinging.
  6. Fixed tenure makes the executive rigid and irresponsible in approach.
  7. It is not free from evils of party system.

Section—B
Answer three questions

Question 5.
(a) Discuss the role of political parties in a democracy. [8]
(b) What is meant by universal adult franchise ? Explain its importance. [6]

(b) The principle of Universal Adult Suffrage has now received a universal acceptance and is implemented in all states which are democratic. This principle stands for the grant of right to vote/franchise or suffrage to all adult citizens, both men and women, without any discrimination. Only minimum age qualification is fixed. Usually all men women of 18 or 20 or 2 T years or above are given the right to vote. In India all the men as well as women who are adults i.e. who are 18 years or above of age, have right to vote. So is the case in the USA, UK, Canada, Australia, and in fact all democratic polities. In India, UK, USA, Russia the right to vote has been given to all citizens of 18 years or above of age.

Further, while no property, sex, and educational qualifications are now fixed for the right to vote, it is a universal practice to deny this right to minors, criminals, bankrupts, mentally unsound people and other persons who earn disqualification for one reason or the other. Minors are denied the right to vote till they attain adulthood i.e. when they cross the legally fixed voting age, as, for example, in India ,they attain the age of 18 years, they get the franchise.

The importance of Universal Adult Suffrage is that it is perfectly democratic, symbolizes the equality of all and assures faith in the ability of the people to elect their rulers i.e. representatives. It is a means for asserting the sovereignty of the people, which is the real basis of the authority of state and its govt. Right to vote acts as a source of strength and confidence for the people. It inculcates in them ‘ a sense of responsibility and participation in the political system.

Without suffrage i.e. without the people enjoying the freedom to vote for electing their representatives, there can be no democratic system. Without a system of elections there can be no democracy and the right to vote and without an electorate, there can be no elections. Right to suffrage constitutes the backbone of every modem democratic system. In a democracy, people are sovereign. They hold the supreme power.

Question 6.
(a) Explain the composition and powers of the House of Representatives. [8]
(b) Discuss the reasons for the decline in the position of the legislature in the last few decades. [6]
Answer:
(b) Regarding the position of Legislature, the situation remained really paradoxical in the 20th century. In almost all the states, the Legislatures remained working as the major instruments of law-making, custodians of the finances of the nation, as agencies of control over the executive, and as the representatives of the people. On the other hand, however, it was found that the Legislatures came to be dominated by the executives. Almost in all the states, the Legislatures suffered a loss of prestige and role due to the rise of powerful executives.

Reasons behind the Decline of Legislature. Several factors are responsible for the decline of Legislature:
Increased Burden of Work : The over-burdened Legislature of a contemporary welfare state is required to pass a very large number of laws most of which relate to highly technical spheres. The legislature finds itself living with paucity of time, resources and expertise, and hence has become dependent upon the executive. The latter has the potential to satisfy these needs of the legislature.

Rise of Delegated Legislation : The system of delegated legislation has considerably strengthened the hands of the executive. Every Legislature today, finds itself left with no other alternative except to delegate some of its law¬making powers to the executive. This has become inevitable because of the changes produced by the rise of industrial society and welfare state.

Role of Political Parties : Due to the emergence of political parties, the legislatures all over the globe have suffered a decline. First, because the parties have definitely limited the nature and scope of Legislative debates. The members of the legislature always speak as party-men and in accordance with the directives and interests of the parties to which they belong.

This has adversely affected the quality of the legislative debates. Secondly, the parties have made it possible for the leaders, the power-holders in the executive, to bank upon the committed support by the ‘majority’. The legislatures are under the control of the leaders, who in turn are arch-leaders. As a result of this, most of the Legislators are hardly in a position to assert their independence to any meaningful extent.

Rise of the Administrative State : The rise of the welfare state has been accompanied by a steep increase in the functions and responsibilities of the executive. The welfare state has given rise to the formulation of highly ambitious goals of socio-economic-cultural development. The people now accept and want the executive to run the administration as welfare administration and secure these goals. This has made it essential for the executive to take up new and newer responsibilities. Consequently, there has been a big increase in the quantity of work that the administration of the state has to handle. The modem state has in actual practice come to be an administrative state.

The System of Judicial Review : The system of Judicial Review has been a factor in the decline of legislature. The right of the courts to judge the constitutional validity of laws passed by the legislature and to reject those laws or such parts of the laws as are considered by them as unconstitutional, has definitely lowered the prestige of the . legislature.

System of Direct Legislation : It states where the system of direct legislation prevails, the people acting through such devices as referendum and initiative, have the right to pass the final verdict on the fate of the bills passed by the legislature. Under such a system, the prestige of the elected assemblies has suffered a decline.

Question 7.
(a) Explain the powers and functions of the Prime Minister of India. [8]
(b) Discuss the role of the Civil Services in the administration of States. [6]
Answer:
(b) The success or failure of the administration depends upon the qualities and dedication of the civil servants who run the administration. They perform several key functions which when performed efficiently lead to the success of the administration and when performed weakly and inefficiently lead to weakness or even failure of the administration. Ministers always depend upon civil servants for such advice and information which helps them to make decisions and policies. Civil Service is a source of stability and continuity in policies. The effective implementation of policies and laws is really the work of the civil servants who work in government departments.

A Civil Service performs the following main functions:
Implementation of Governmental Policies and Laws : It is the responsibility of the bureaucracy to carry out and implement the policies of the Government. To implement the laws in actual practice is its important function. Good policies and laws can really serve their objectives only when these are implemented by the civil servants in true spirit, and with vigour.

Role in Policy Formulation : The policy-making is the function of the political executive. However, the civil servants always play an active role in this exercise. They supply the data needed by the political executive for formulating the policies. In fact, they formulate several alternative policies in respect of a particular sphere of governmental activity and describe the merits and demerits, and pros and cons of each policy alternative. The ministers then select and adopt one such policy alternative as the government policy.

Conduct of Administration : To run the day to day administration in accordance with the policies, laws, rules, regulations and decisions of the Government with a view to realize the developmental goals set forth by the state, is also the key responsibility of the Civil Service. The political executive as the head of the administration simply exercises guiding, controlling and supervising functions.

Advisory Function : One of the important functions of the Civil Service is to advise the political executive. The.ministers receive all the information and advice regarding the functioning of the government departments from the civil servants. As amateurs, the ministers have little knowledge of the functions of their departments. They, therefore, depend upon the advice of professionally trained, qualified and expert bureaucracy.

Role in Legislative Work : The civil servants, particularly working within the parliamentary system, play an important and big but indirect role in law-making. They draft the bills which the ministers submit to the legislature for law making. The ministers provide all the information .asked for by the legislature and the legislative committees by taking the help of the civil servants. In this contemporary era when the system of delegated legislation has got involved the executive in the sphere of legislation, the role of bureaucracy has got a big boost. The delegated legislation is mostly done by the civil servants. The ministers simply perform a signing, supervisory and controlling role.

Semi-judicial Work : The emergence of the system of administrative justice, under which several types of the cases and disputes are decided by the executive, has further been a source of increased semi-judicial work of the bureaucracy. The disputes involving the grant of permits, licences, tax concessions, quotas etc. are now settled by the civil servants.

Question 8.
(a) Discuss the relationship between the :
(i) Judiciary and the Executive.
(ii) Judiciary and the Legislature. [8]
(b) Define Judicial Review. Explain how it works in India and the USA. [6]
Answer 8.
(a)Relation of the Judiciary with the Executive : Executive has always a role to play in the organization and working of the judiciary. Appointment by the executive is considered to be the best available method of appointing the Judges. Sometimes, the executive, in association with the legislature, is also assigned a role in the removal of the judges through impeachment. It is the responsibility of the executive to apprehend the culprit in case of every breach of law and bring him before the judiciary for his conviction and punishment. However, it is the supreme function of the judiciary to try and judge whether the person apprehended and produced before it by the executive, is really guilty of the charges leveled against him or not.

The judiciary, also, exercises some control over the executive. The executive is, in all the democratic countries, subject to the jurisdiction of the courts. The officials of the executive are to appear before the judiciary in all cases necessitating their evidence. In case any breach of law is committed by them, they too have to face the trial before the courts and undergo punishment if found guilty and convicted by the courts.

The judiciary has the power to nullify any and every act of the executive, which, in its opinion, involves a dishonest or excessive or arbitrary use of power by the executive. The judiciary can conduct judicial review over the rules made by the executive whenever any of these rules is challenged by the people. Further, the judiciary has the power to initiate contempt of court proceedings against any member of the executive in case he acts in a manner which tends to lower the dignity and honour of the courts.

Relation of Judiciary with The Legislature : Judiciary is also related to the legislature because the laws made by the legislature are in practice interpreted and applied by the courts. In the process, the courts really give meaning and effect to the laws. Furthermore, in almost all democratic countries, the judiciary has the power to conduct judicial review over the laws with a view to determine whether these are ultra vires or intra vires. Judiciary has the power to reject those laws or such parts of the laws which are considered and found to be ultra virus by it.

In the US Constitution, the judiciary has the power to reject as unconstitutional all laws which are considered to be violative of the high principle of “Due Process of Law.” The US Supreme Court can reject any law as unconstitutional and illegal if it is either found to be not passed in accordance with the procedure established by law or is held violative of the ends of justice and standards of goodness, or both. In India, the judiciary can reject a law as unconstitutional if it is found to be violative of the procedure established by law.

The judiciary, through the case laws or the judge-made laws, supplements the law making role of the legislature. While interpreting and applying laws, the judges also make laws. Judicial decisions, given by the Courts of Records, the higher level courts, are binding upon the lower courts as laws proper. All such judge-made laws supplement the legislation passed by the Legislature.

The legislature also plays a role in the organization and functioning of the courts. In some of the states, the appointment of the judges is made through election by the legislature, as for example is the case in Switzerland where the Federal Assembly elects the judges of the Federal Tribunal. In some other states, the appointment of a judge by the executive is final only when it gets the approval/ratification by the legislature.

(b) The Constitution is supreme in the U.S.A. and the Supreme Court has the power to interpret it and preserve its supremacy by preventing its violations by any institution or person. This provision has been the basis for a very important power of the Supreme Court the power of judicial review. It has come to be recognized as the most distinctive attribute and function of the Supreme Court. It is the power to judge the constitutional validity of the acts of other two organs of the government. In case the court finds that any act of these does not comply with the fundamental constitutional principles, it is declared null and void.

Features of Judicial Review in India :
The following are the salient features of the system of Judicial Review in India :
Both the Supreme Court and High Courts exercise the power of Judicial Review. But the final power to determine the constitutional validity of any law is in the hands of the Supreme Court of India.

Judicial Review can be conducted in respect of all Central and State laws, the orders and ordinances of the executive and constitutional amendments.

Judicial Review cannot be conducted in respect of the Acts incorporated in the 9th Schedule of the Constitution.

Judicial Review applies only to the questions of law. It cannot be exercised in respect of political issues.

Judicial Review is not automatic. The Supreme Court can conduct judicial review over a law only when it is challenged during the course of litigation. In other words, it becomes operative only when laws are specifically challenged before it or when during the course of litigation in a case, the question of constitutional validity of any law is raised before it.

The Supreme Court, after reviewing the challenged Act, can decide :

  • The law is constitutionally valid. In this case the law continues to operate as before, or
  • The law is constitutionally invalid. In this case the law ceases to operate with effect from the date of the judgement, or
  • Only some parts or a part of the law is invalid. In this case only invalid parts or part becomes non operative and other parts continue to remain operative. However, if the invalidated parts are so vital to the law that other parts cannot operate without these, then the whole of the law gets invalidated.

While declaring a law void, the Supreme Court has to cite the provisions/ articles of the constitution which the said law violates. It has to demonstrate the invalidity of the law struck down by it.

The work done on the basis of the law before it is declared unconstitutional and invalid by the court, continues to remain valid.

The Supreme Court can revise or annul its own earlier decisions.

Procedure established by Law vs Due Process of Law or Difference between Indian and the American Systems of Judicial Review.

U.S.A. : While discussing the working of judicial review we must clearly understand that it is neither automatic nor mechanical. The laws passed by the Congress and the State Legislatures become operative the moment these become laws. These do not go to the court for judicial review. It is only when any of the law is specifically challenged or when during the course of litigation in a case, the issue of constitutional validity of any law arises, that the court conducts judicial review. The court cannot of its own take up any law and conduct judicial review on it.

Decisions in Judicial Review : After conducting the judicial review, the Supreme Court can give 3 types of decisions:
It can declare the law unconstitutional or ultra virus. In this case the law stands struck down and it ceases to operate from the date on which the Supreme Court declares it invalid;

It can declare the law constitutional and fair. In this case the law continues to operate as before without any change.

It can declare any part or some parts of the law unconstitutional. In this case only the part or parts declared unconstitutional cease to operate and the rest of law continues to v operate.

Due Process of Law and Judicial Review : Since the passing of the fifth amendment of the Constitution, the scope of judicial review has become very vast. In one of its clauses it has been laid down that “the Government cannot deprive anyone of life, liberty or property without due process of law.” The term “Due Process of Law” means that the life, liberty or property of the people cannot be subjected to arbitrary and unfair limitations by the law or the executive and even by the judges in the process of awarding punishments.

Question 9.
(a) Discuss the different kinds of social inequalities that exist in India. What is the impact of these inequalities on the democratic system of India ? [8]
(b) Suggest remedial measures to remove regional imbalances in India. [6]

ISC Class 12 Political Science Previous Year Question Papers

ISC Hindi Question Paper 2013 Solved for Class 12

ISC Hindi Previous Year Question Paper 2013 Solved for Class 12

Section-A

Question 1.
Write a composition in Hindi in approximately 400 words on any ONE of the topics given below :- [20]

निम्नलिखित विषयों में से किसी एक विषय पर लगभग 400 शब्दों में हिन्दी में निबन्ध लिखिये :
(a) विज्ञान की चमत्कारिक देन ‘कम्प्यूटर’ आज के युग में अति आवश्यक है। इस विषय पर अपने विचार व्यक्त करें।
(b) समाज सेवा सच्ची मानव सेवा।
(c) शिक्षा का व्यवसायीकरण ही शिक्षा के स्तर में गिरावट का कारण है पक्ष या विपक्ष में अपने विचार लिखिए।
(d) किसी पर्वतीय स्थल की यात्रा का वर्णन कीजिए जो आपके जीवन की अविस्मरणीय यात्रा बन गई हो।
(e) “जिसने अनुशासन में रहना सीख लिया उसने जीवन का सबसे बड़ा खजाना पा लिया।” विवेचन कीजिए।
(f) निम्नलिखित विषयों में से किसी एक विषय पर मौलिक कहानी लिखिए.
(i) अस्पताल में बहुत भीड़ देखकर मन परेशान हो गया।
(ii) एक कहानी जिसका अन्तिम वाक्य होगा “इसलिए कहते हैं नैतिक पतन से देश का पतन होता है।”
Answer:
(a)

“विज्ञान की चमत्कारिक देन ‘कम्प्यूटर’
आज के युग की आवश्यकता”

आज के युग को विज्ञान के चमत्कारों का युग कहा जाता है। यह सच भी है कि सुबह से लेकर रात तक हम लोग जितनी भी वस्तुएँ प्रयोग में लाते हैं प्रायः वे सभी विज्ञान की ही देन हैं। मानव आदिकाल से ही अपनी कुशाग्र बुद्धि का परिचय हमें नित नवीन आविष्कारों को जन्म देकर देता रहा है। इस दिशा में विज्ञान व वैज्ञानिकों ने मानव जीवन में एक अद्भुत क्रान्ति ला दी है। विज्ञान की ही एक नई चमत्कारिक देन है कम्प्यूटर।।

कम्प्यूटर का प्रयोग गत दस-बारह वर्षों से अत्यधिक तीव्रता से बढ़ता जा रहा है। जनसामान्य के हितार्थ एवं तीव्र आर्थिक समृद्धि के लिए कम्प्यूटर का अधिकाधिक प्रयोग आवश्यक है। वास्तव में जिस कम्प्यूटर का प्रयोग शिक्षा के क्षेत्र में दस-बारह वर्ष पूर्व किया गया था वही आज कम्प्यूटर शिक्षा का एक आवश्यक अंग बन गया आज कम्प्यूटर के कारण उद्योग, शिक्षा, प्रशासन, कृषि विज्ञान एवं तकनीकी आदि सभी क्षेत्रों में व्यापक परिवर्तन हुए हैं। हमारे प्रधानमन्त्री मोदी जी का प्रत्येक कार्य कम्प्यूटर द्वारा कराने का सपना आज साकार दिखाई दे रहा है।

कम्प्यूटर का आविष्कार एवं भूमिका- कम्प्यूटर के जनक अंग्रेज गणितज्ञ चार्ल्स बेबेज को स्वीकारा जाता है। उन्होंने ही गणित व खगोल विज्ञान सम्बन्धी सूक्ष्म सारणी तैयार करने हेतु एक महान गणक यन्त्र की योजना बनायी, किन्तु इसके असफल हो जाने पर अमेरिकी वैज्ञानिक डॉ. बेन्नेवर बुश ने 1930 में यान्त्रिक कल-पुर्जी का एक यन्त्र बनाया, जिसने ही औद्योगिक कम्प्यूटर को जन्म दिया। वही आधुनिक इलैक्ट्रोनिक कम्प्यूटर के जनक कहे जाते हैं। दूसरे विश्वयुद्ध के समय पहली बार विद्युतचालित कम्प्यूटर का निर्माण गणना करने के लिए हुआ था। वर्तमान में छोटे-बड़े सभी उद्योगों, तकनीकी व्यवसायों, प्रशासनिक कार्यालयों, वैज्ञानिक संस्थाओं में कम्प्यूटर का प्रयोग बढ़ता जा रहा है। इसने अपनी उपयोगिता ही नहीं दर्शायी है बल्कि सूचना प्रौद्योगिकी के क्षेत्र में क्रान्ति उत्पन्न कर दी है।

कम्प्यूटर के क्षेत्र- आज कम्प्यूटर का क्षेत्र न केवल अत्यन्त व्यापक हो गया है बल्कि इसने अक्षर, शब्द, आकृति व तार्किक कथनों को भी ग्रहण कर विभिन्न व्यवसायों के लिए एक सरल व्यवहार प्रणाली की रचना की है।

कम्प्यूटर वर्तमान में सूचना प्रसारण एवं नियन्त्रण का शक्तिशाली साधन बन गया है। इससे उपग्रहों के अन्तरिक्ष में संचरण, उनका सम्पर्क व नियंत्रण, बड़े-बड़े उद्योगों में मशीनों के संचालन आदि के जटिल कार्यों को आसान बना दिया है।

कम्प्यूटर द्वारा बैंकों के कार्य-व्यवहार, हिसाब-किताब सरल बन सके हैं। यह न केवल गणना के लिए उपयोगी है अपितु भवनों, कारों, मोटरों, रेलों, वायुयानों तक के प्रारूप तैयार करने में सार्थक एवं समर्थ है। वास्तव में इसने हमारी हर मुश्किल आसान बना दी है। इससे विकास की गति कई गुना बढ़ गई है। यह मनुष्य के मस्तिष्क से भी द्रुत गति से कार्य करता है। आज हमें कूँची लेकर बैठने व कैनवास पर चित्र उकेरने की आवश्यकता नहीं क्योंकि कम्प्यूटर हमें एक नियोजित प्रोग्राम के अनुसार प्रिंट की कुंजी दबाते ही प्रिंटर द्वारा कागज पर चित्र छाप कर दे देता है।

अनुसंधान के क्षेत्र में इससे बड़ा क्रान्तिकारी परिवर्तन नहीं आया है। बच्चों के मनोरंजन और खेलों के कारण यह उनके आकर्षण का केन्द्र है। चिकित्सा के क्षेत्र में यह मानव हितार्थ कार्यों में संलग्न है। रेलवे स्टेशन, होटल, मॉल, बाजार सभी स्थानों पर कम्प्यूटर अपनी पहचान बना चुका है। सामान्य ज्ञान का यह भण्डार है। परीक्षा-परिणामों की गणना, योग, गुणा, भाग, समाचार पत्र, पाठ्य पुस्तकों का प्रकाशन आदि सभी कार्य आज कम्प्यूटर पर आश्रित हो गए हैं। इससे पल-भर में हमारे सभी कार्य पूर्ण हो जाते अतः समय की माँग व हमारी आवश्यकताओं ने हमें पूर्णत: कम्प्यूटर पर निर्भर बना दिया है। कम्प्यूटर का अधिक से अधिक प्रयोग करके हम अपने देश की प्रगति में वृद्धि कर सकते हैं। विज्ञान की चमत्कारिक देन ‘कम्प्यूटर’ आज के युग में अति आवश्यक है।

(b)

समाज सेवा–सच्ची मानव सेवा।

मनुष्य एक सामाजिक प्राणी है। समाज में वह एक-दूसरे के साथ हिल-मिल कर रहता है तथा एक-दूसरे के सुख-दु:ख में । सहयोगी बनता है। मानव-समाज के सुसंचालन के लिए मानव-हृदय में मानव के प्रति सेवा भावना का होना अति आवश्यक है।

समाज सेवा की भावना ही सच्ची मानव सेवा स्वीकारी गई है। वास्तव में मानवता को सबसे बड़ा धर्म माना गया है। कवि मैथिलीशरण गुप्त ने कहा है-‘वही मनुष्य है, जो मनुष्य के लिए मरे।’ यह प्रकृति का भी सहज स्वाभाविक नियम है। प्रकृति के सभी अंग किसी-न-किसी रूप में दूसरों की भलाई में तत्पर रहते हैं फिर मनुष्य क्यों नहीं? यदि मनुष्य अपने ही स्वार्थ की चिन्ता करे तो वह पशु की श्रेणी में आ जाता है, स्वार्थी कहलाता है।

सामाजिक प्राणी होने के नाते, दूसरों के सुख-दुःख की चिन्ता करना, उनका हित सोचना, सहयोग देना मानव का परम कर्तव्य एवं समाज सेवा होती है। मानव सेवा के पथ पर चलने हेतु हमें स्वीकारना होगा कि हमारा शरीर परोपकार के लिए है। परोपकार के भावों से ही सच्ची मानव सेवा एवं मानवता के आदर्श को पाया जा सकता है।

वास्तव में जब प्रकृति के जीव-जन्तु निःस्वार्थ भाव से दूसरों की भलाई में तत्पर रहते हैं तब विवेकशील प्राणी होते हुए भी मनुष्य यदि मानव जाति की सेवा न कर सका तो उसका जीवन कलंक स्वरूप ही है। मनुष्य होते हुए भी मनुष्य कहलाने का उसे कोई अधिकार नहीं। जिसमें समस्त मानव समुदाय के लिए सहानुभूति व प्रेम का भाव रहता है। वही सच्ची समाज सेवा, मानव सेवा के रूप में अपना सकता है।

यदि किसी व्यक्ति के मन में मनुष्य सेवा की भावना नहीं है, अपने पीड़ित भाई को देखकर जिसके हृदय में कसक नहीं उठती। उसकी सहायता के लिए वह तत्पर नहीं होता तो उसका मन्दिर में जाकर पूजा और अर्चना करना ढोंग और पाखण्ड है। प्रसिद्ध नीतिकार सादी ने कहा है-“अगर तू एक आदमी की तकलीफ को दूर करता है तो वह कहीं अधिक अच्छा काम है, बजाय तू हज को जाये और मार्ग की हर एक मन्जिल पर सौ बार नमाज पढ़ता जाए।”

सामाजिक प्राणी होने के नाते मनुष्य का यह कर्तव्य भी है कि वह दूसरों के सुख-दुःख की चिन्ता करे, क्योंकि उसका सुख-दुःख दूसरों के सुख-दुःख के साथ जुड़ा हुआ है। सच ही कहा गया है

“वह शरीर क्या जिससे जग का कोई भी उपकार न हो।
वृथा जन्म उस नर का जिसके मन में सेवा भाव न हो।”

यों जीने को तो सभी मनुष्य जीते हैं। केवल अपने लिए जीना न तो मनुष्यता का लक्षण है और न सच्चे अर्थों में जीवित रहने का लक्षण। महानता के आदर्श को लेकर जीने वाले बुद्ध, जैन तीर्थंकर, महात्मा गाँधी, मदर टेरेसा आदि लोग इसीलिए महान बने क्योंकि वह केवल अपने लिए नहीं बल्कि सम्पूर्ण मानवता के लिए जिये-मरे।

उनके शब्दों में ‘मानवता की सेवा ही ईश्वर की सच्ची प्रार्थना थी।’ समाज सेवा का बीड़ा उठाकर, समाज द्वारा परित्यक्त, रुग्ण व्यक्तियों के कल्याण एवं परित्राण में ही उन्हें जीवन का सन्तोष एवं सच्ची पूजा दिखाई दी।

उनके आदर्शों, विचारों एवं कर्म से निराश निर्धनों को न केवल आशामयी भविष्य की प्राप्ति हुई बल्कि जीवनेच्छा खो चुके कुष्ठ रोगियों को ममतामयी छाँव मिल सकी।

समाज सेवा के इसी भाव के परिणामतः सच्ची मानव सेवा का आदर्श रूप हमें दिखाई देता है। वास्तव में मदर टेरेसा ने मानवता की सेवा करके न केवल समाज के प्रति दया, प्रेम, सेवा भाव दिखाया बल्कि हमें मानव सेवा के महत्व से अवगत करा दिया। हमें स्वीकारना ही होगा कि सच्ची मानव सेवा ही समाज सेवा है व हमारा परम धर्म भी।

(c)

“शिक्षा का व्यवसायीकरण ही शिक्षा के

स्तर में गिरावट का कारण है।” शिक्षा का वास्तविक अर्थ कुछ पुस्तकें पढ़कर परीक्षाएँ पास कर लेना ही नहीं हुआ करता। शिक्षा का वास्तविक अर्थ है- अनेक विषयों में ज्ञान प्राप्ति अर्थात् शिक्षा का तात्पर्य ज्ञान से ही है।

शिक्षा व्यक्ति को विविध विषयों का ज्ञान करा कर उसके मनमस्तिष्क, इच्छा और कार्य-शक्तियों का विकास तो करती ही है, उसके छिपे व सोये गुणों को उजागर कर एक नया आत्मविश्वास भी प्रदान करती है जिससे सभी प्रकार की सफलताओं की सम्भावनाएँ लेकर व्यक्ति प्रगति-पथ पर निरन्तर आगे बढ़ सकता है।

इसे दुखद स्थिति ही कहा जा सकता है कि आज शिक्षा व्यवसाय का रूप लेती जा रही है। शिक्षा को एक प्रकार से रोजगार या व्यवसाय से सीधा जोड़ा जाने लगा है।

पक्ष में तर्क-आज शिक्षा का व्यवसायीकरण ही शिक्षा के स्तर में गिरावट का कारण है क्योंकि आज प्रत्येक व्यक्ति जिस किसी भी तरह से शिक्षा के नाम पर कुछ डिप्लोमा-डिग्रियाँ प्राप्त करके अपने आपको व्यवसाय या रोजगार का अधिकारी मानने लगता है। यही वह मानसिकता है जिसने आज न केवल शिक्षा के उद्देश्य को नष्ट कर दिया है, बल्कि उसे एक प्रकार का व्यवसाय ही बना डाला है।

शिक्षा के इसी व्यावसायिक रूप ने शिक्षा की महत्ता घटा दी है। आज की शिक्षा निश्चय ही अक्षर ज्ञान या अधिक से अधिक किताबी विषयों का ज्ञान कराने से अधिक कुछ नहीं सिखा पाती। वह अपनी सामयिक उपयोगिता खो चुकी है। वह अपने साथ-साथ राष्ट्र और मानवता का कल्याण करने में सहायक नहीं है। इसके साथ ही ‘गुरु’ का आदर्श अध्यापक ट्यूशन पढ़ने वाले छात्रों को प्रश्न-पत्र से अवगत करा देते हैं या वही प्रश्न पूछते हैं, जो उन्होंने पढ़ाये होते हैं। इससे ऐसे ट्यूशन वाले छात्रों का परीक्षा परिणाम तो अच्छा ही आता है पर वास्तविक शिक्षा या ज्ञान के नाम पर विद्यार्थी अपने को कोराका-कोरा ही पाता है। शिक्षा का यह व्यवसायीकरण ही आज सबसे बड़ी बिडम्बना है।

विपक्ष में तर्क-शिक्षा का व्यवसायीकरण शिक्षा के स्तर में गिरावट का कारण नहीं है। आज शिक्षा प्राप्त कर प्रत्येक व्यक्ति किसी-न-किसी रूप में व्यवसाय या रोजगार पाने की चाह रखता है। भारत जैसे अविकसित, निर्धन और बेकारों से सम्पन्न देश में शिक्षा के प्रति दष्टिकोण व्यवसायोन्मखी हो जाना बहत अधिक अस्वाभाविक या असंगत भी नहीं लगता। जिस देश में साधारण जनता रोटी, कपड़ा, मकान जैसी प्राथमिक और आवश्यक जरूरतें भी पूरी नहीं कर पाती। कदम-कदम पर अभावों में जीवन काटने को मजबूर होना पड़ता है। महँगाई की निरन्तर वृद्धि एवं उपभोक्ता वस्तुओं के दाम आकाश को छू रहे हों, वहाँ का खाता-पीता व्यक्ति भी केवल शिक्षा के लिए शिक्षा की बात नहीं सोच सकता। शिक्षा का व्यवसायीकरण होना स्वाभाविक ही लगता है। इसके साथ ही शिक्षा के व्यवसायीकरण हेतु अधिकांशतः हम अध्यापक को ही दोषी मान लेते हैं।

हम शायद ये भूल जाते हैं कि इन्जीनियर या डॉक्टर तैयार करने वाला अध्यापक वर्ग भी अपने बच्चों का भविष्य उज्ज्वल बनाना चाहता है। समय के साथ ताल से ताल मिलाकर चलने में वह सीमाओं का थोड़ा बहुत उल्लंघन कर बैठता था जो नहीं करना चाहिए।

वास्तव में कोई भी शिक्षा समय की माँग एवं आवश्यकता को ध्यान में रखते हुए ही सफल-सार्थक कही जा सकती है। आज समय की मांग भी यही है कि शिक्षा और व्यवसाय में परस्पर सीधा सम्बन्ध स्थापित किया जाए। तकनीकी, इंजीनियरी आदि शिक्षा देने के लिए सरकार ने कुछ अलग केन्द्र स्थापित किए हैं पर एक तो ऐसे केन्द्रों की संख्या बहुत कम है, दूसरे स्कूलों-कॉलेजों में दी जाने वाली शिक्षा के साथ इस प्रकार के शिक्षण-प्रशिक्षण का कोई तालमेल नहीं बैठता। अतः वर्तमान परिप्रेक्ष्य में हमें समूची शिक्षा-प्रणाली के ढाँचे में आमूल परिवर्तन लाकर प्रारम्भिक शिक्षा से ही व्यवसायोन्मुख प्रणाली लागू की जाए तभी आवश्यकता पूर्ति में शिक्षा का सार्थक सहयोग प्राप्त हो सकेगा।

(d)

“जीवन की अविस्मरणीय पर्वतीय यात्रा”

पहाड़ों की यात्रा का अपना-अलग ही महत्व एवं आनन्द हुआ करता है। साधारणतः लोग प्रकृति सौन्दर्य दर्शन के लिए पहाड़ों की यात्रा किया करते हैं, जबकि कुछ लोग मनोरंजन एवं आनन्दानुभूति के लिए पहाड़ों पर जाया करते हैं। इसके साथ ही धनाढ्य वर्ग गर्मी से राहत पाने के लिए दो-एक महीने ठण्डे स्थान पर, पहाड़ों पर बिताया करते हैं। कभी-कभी लोग बर्फ गिरने (Snowfall) के अवलोकन हेतु भी भयानक सर्दी में टिकट कटाकर पहाड़ों पर जाते हैं। अपनी-अपनी भावना के अनुसार मनुष्य आचरण करता है। यही मानव-स्वभाव की सत्यता है।

पिछले वर्ष मुझे भी पर्वतीय स्थल की यात्रा का अवसर प्राप्त हुआ। इस यात्रा पर जाने के लिए मेरे दो उद्देश्य थे। पहला तो मैं पहाड़ों के प्राकृतिक सौन्दर्य को निहारना चाहता था। दूसरा यह जानना चाहता था कि वहाँ के लोगों का जीवन कैसे व्यतीत होता होगा? इसलिए मैंने मसूरी की यात्रा की योजना बनाई। मेरे दो-तीन अन्य मित्र भी मेरे साथ चलने को तैयार हो गए।

कॉलेज की छुट्टियाँ होने के कारण घर से भी हमें अनुमति मिल गई। निश्चित तिथि पर हम बस द्वारा मसूरी की यात्रा के लिए चल पड़े। शहर की भीड़-भाड़ को धीरे-धीरे पार कर हम मसूरी की यात्रा पर थे। हमारी बस देहरादून की राह पर मसूरी जाने के लिए भाग रही थी। देहरादून की वादी में प्रवेश करने से पहले तक का रास्ता आम बस-मार्गों के समान ही रहा। कहीं सूखे मैदान, कहीं हरेभरे खेत, कहीं कुछ वृक्षों के साये और कहीं एकदम खाली सड़कें।

कुछ आगे जाने पर सड़क के आस-पास उगे वृक्षों ने अचानक घना होना शुरू कर दिया। वातावरण में उमस सी अनुभूति होने लगी। धीरे-धीरे घने वृक्षों के जंगल और भी घने होते गए। हमारी बस भी पहाड़ों पर चढ़ती हुई दिखाई दी। गोलाकार चढ़ाई, ठण्डी हवा के तीखे झोंके, जंगली फूलों की महक, दूर से दिखने वाली बर्फ जमी सफेद पहाड़ी चोटियाँ, लगता था हम प्रकृति के बड़े ही सुन्दर और रंगीन लोक में आ पधारे हैं।

जी चाहता था कि बस से उतरकर इन घाटियों में दूर तक चला जाऊँ। घूमते बादलों के टुकड़ों को पकड़ लूँ। इस प्रकार प्रकृति की आँख-मिचौली के दृश्य देखते हुए हम मसूरी पहुँचे।

बस के रुकने तक मेरा मन तरह-तरह की कल्पना की डोरियों में उलझा रहा, परन्तु मेरी भावना को उस समय गहरी ठेस लगी जब हम जैसा एक मनुष्य फटे-पुराने कपड़ों में ठिठुरते, पाँव में जूते के स्थान पर कुछ रस्सियों को लपेटे, पीठ पर रस्सी और एक चौखटासा उठाए आकर कहने लगा-“कुली साहब। आगे बढ़कर उसने हमारा सामान उठाना शुरू भी कर दिया।

पता नहीं किस भावना से जब मैंने उसे रोकना चाहा तो पास खड़े एक आदमी ने कहा- उठाने दीजिए साहब, आपके वश में बैग लेकर चढ़ पाना सम्भव नहीं। यही लोग सामान उठाकर व भागकर चढ़ाई चढ़ सकते हैं। बाद में मैंने अनुभव किया कि दयनीयता में जीवन गुजारते, पहाड़ों के ये पुत्र ही इतनी शक्ति रखते हैं।

उस दिन विश्राम करके सुबह ही हम केम्पटी फॉल, फोती घाट, नेहरू पार्क आदि दर्शनीय स्थल देखने में व्यस्त हो गए। प्रकृति इतनी विराट, सुन्दर एवं आकर्षक है इसका मुझे पहली बार अनुभव हुआ। साथ ही यह भी महसूस किया कि प्रकृति उदारता से भरी है। वह मुक्तभाव से हमें कितना कुछ प्रदान करती है।

अगले दो दिन मैंने वहाँ के साधारण लोगों का रहन-सहन, व उनकी स्थिति को देखने में लगाए। वहाँ के लोग जो आम दिखते हैं, उन्हें बड़ा ही कठिन जीवन व्यतीत करना पड़ता है।

इस पर्वतीय स्थल की मेरी यात्रा खट्टे-मीठे दोनों प्रकार के अनुभव दे गई। प्रकृति की वह अनोखी घटाएँ अविस्मरणीय हैं। मेरे __ मन मस्तिष्क से इसकी छाप कभी नहीं मिट सकती।

(e)

“जिसने अनुशासन में रहना सीख लिया उसने
जीवन का सबसे बड़ा खजाना पा लिया।”

‘अनुशासन’ शब्द ‘अनु’ और ‘शासन’ इन दो शब्दों के मेल से बना है। ‘अनु’ उपसर्ग है, जिसका अर्थ है- साथ, शासन अर्थात् नियम या विधान। अतः अनुशासन का तात्पर्य है कि हम जिस स्थान पर रहे हों, देशकाल के अनुसार वहाँ के जो नियम हैं, आचारव्यवहार हैं, उनका उचित ढंग से पालन करना।

अनुशासन में रहकर ही व्यक्ति सहज भाव से आगे बढ़ता हुआ अपने जीवन का लक्ष्य प्राप्त कर सकता है। मनुष्य को सामाजिक प्राणी इसी कारण ही कहा जाता है, क्योंकि वह प्रत्येक कार्य दूसरों का ध्यान रखकर अनुशासित ढंग से करता है। वास्तव में मनुष्यों का स्थान, समय और आवश्यकता को पहचान कर किया गया व्यवहार ही अनुशासन का रूप धारण कर लेता है।

शरीर को सुन्दर बनाने के लिए जिस प्रकार सन्तुलित भोजन आवश्यक है। उसी प्रकार जीवन और समाज को स्वस्थ, सुन्दर बनाने के लिए व्यक्ति का अनुशासित होना भी बहुत आवश्यक है; जैसेबासी, सड़ा-गला खाना, आवश्यकता या भूख से अधिक खाना अस्वस्थता एवं समस्याएँ प्रदान करता है, उसी प्रकार अनुशासनहीनता जीवन एवं समाज के स्वरूप को बिगाड़ देती है। हमें ऐसा कोई भी कार्य नहीं करना चाहिए जिसका समाज पर बुरा प्रभाव पड़े।

शिक्षा प्राप्त करने का अर्थ केवल किताबें पढ़कर परीक्षाएँ पास करना ही नहीं होता, बल्कि हर प्रकार से कुछ सीखना भी हुआ करता है। अपने गुरुजनों का आदर व उनके सद्भावों को अपनाना अपने सहपाठियों के साथ अपना व्यवहार ठीक रखना, सभी का सम्मान करना, जरूरत पर सबकी सहायता करना ही वास्तव में अनुशासन है।

अपने घर-परिवार के साथ रहते हुए, आस-पड़ोस के लोगों से आदर, सम्मान और नम्रता से पेश आना चाहिए। कोई भी ऐसा काम कभी भी नहीं करना चाहिए जिससे हमें या हमारे घर-परिवार को लज्जित होना पड़े। इस प्रकार स्पष्ट है कि जिसे अच्छा और उचित व्यवहार करना कहा जाता है, वास्तव में वही अनुशासन है।

हम देखते हैं कि सड़कों पर चलते हुए लोग दूसरों का ख्याल हैं। सड़कों, गलियों पर चलते हुए लोग जोर-जोर से चिल्लाते हुए या आपस में गाली-गलौच करते आते-जाते हैं। सार्वजनिक स्थानों या पार्कों में बैठकर जुआ खेलते हैं। इस प्रकार के सभी व्यवहार केवल अनुशासनहीनता ही नहीं, असभ्यता भी माने जाते हैं।

व्यक्ति अपने व्यवहार से ही पहचाना जाता है। जिस प्रकार अच्छे समाज और महान् राष्ट्रों की पहचान उसके अनुशासित नागरिकों से ही हुआ करती है। उसी प्रकार अनुशासन वह आईना है जिसमें सभ्य और सुसंस्कृत जातियों का प्रतिबिम्ब सरलता से देखा और परखा जा सकता है। यदि व्यक्ति मानवीय आदर्शों और अनुशासन का पालन करने वाला नहीं, तो उसकी सारी शिक्षा और वंश की उच्चता व्यर्थ होकर रह जाते हैं। अगर ऐसे लोगों की संख्या बढ़ने लगती है तो वह धर्म, जाति, देश और राष्ट्र सभी के नाम को डुबोने वाले साबित हुआ करते हैं। अनुशासन कठिन नियमों का पालन करना नहीं अपितु समय, स्थान एवं स्थिति के अनुसार उचित व्यवहार करना ही होता है। अतः अनुशासित देश और राष्ट्र ही सब प्रकार की प्रगति एवं विकास कर पाते हैं। “जिसने अनुशासन में रहना सीख लिया उसने जीवन का सबसे बड़ा खजाना पा लिया।”

(f) (i)

अस्पताल में बहुत भीड़ देखकर मन

परेशान हो गया। बात उन दिनों की है जब मेरे भाई-भाभी के यहाँ एक पुत्र का जन्म हुआ। मैं अपार खुशी से भर उठा कि हमारे घर में एक छोटा बच्चा आ गया है।

मैं दूसरे दिन उस बालक को देखने अपने पिताजी के साथ दिल्ली रवाना हो गया। आगरा से दिल्ली का सफर बड़ा सुहाना लग रहा था, क्योंकि मन की खुशी वातावरण को भी मोहक बना रही थी। मेरी भाभी अपने घर दिल्ली गई हुई थीं वहीं उन्हें पुत्ररत्न की प्राप्ति मुझे उन्हें देखने जब अस्पताल जाने का अवसर मिला तब तक मैं जीवन के सत्य से अनभिज्ञ-सा था। कितना दुख और दर्द छिपा है इंसान के जीवन में इसकी गहराई मुझे अस्पताल के अन्दर पहली बार महसूस हुई।

अस्पताल में आते-जाते लोग सभी दुख-दर्द से भरे थे। कोई . स्वयं परेशान था। किसी के साथ आए लोग परेशान थे। परेशानियाँ इतनी थीं कि खुशियों का अम्बार जो मेरे भीतर भरा था कहीं दब-सा गया था।

पिताजी ने मेरे साथ भाभी से मुलाकात की व मैंने छोटे से, कोमल बालक यानी अपने भतीजे से भी मुलाकात की। जो अपनी आँखें बन्द किए सो रहा था। पिताजी कुछ सामान लेने के लिए बाहर गए और मैं भी बाहर आकर खड़ा हो गया। खिड़की से नीचे देखने पर आते-जाते लोगों की भीड़ दिखाई दे रही थी। मन में एक अज्ञात प्रार्थना उमड़ी कि हे भगवान इन सभी की दुख-तकलीफों से इन्हें राहत प्रदान करना। मेरे अन्दर से शुभेच्छाएँ निकल रही थीं। सामान देकर पिताजी के आने पर मैं भाभी से विदा लेकर घर लौटने को चल पड़ा। सीढ़ियाँ उतरते समय लोगों की भीड़ का रेला इतनी जोर से आ जा रहा था कि समझ में नहीं आ रहा था कि क्या इतनी भीड़ कभी अस्पतालों में भी हो सकती है?

आज तक जनसंख्या वृद्धि का ये हाल मुझे रेलवे स्टेशन, बस स्टैण्ड पर ही देखने को मिला था। अस्पताल में इतनी भीड़ देखकर मन परेशान हो गया। लोगों के दु:ख-दर्द का आभास व कष्टों की अधिकता का यथार्थ अस्पताल में जाकर मुझे पहली बार महसूस हो सका।

अपनी खुशी का आभास था पर दूसरों के दुःखों से भी सहानुभूति का अहसास हो रहा था।

(ii)

“नैतिक पतन से देश का पतन होता है”

हमारे राजनेता हमारे समाज के अंग एवं हमारी राजनीतिक स्थिति के आधार माने गए हैं। वर्तमान परिप्रेक्ष्य में देखें तो राजनीति दलदल के समान मानी जाने लगी है जिसमें प्रत्येक नेता फँसता चला जा रहा है। ऐसा नहीं है कि ईमानदार व्यक्ति या ईमानदारी प्रायः समाप्त हो गयी है। गाँव देहातों में तथा शहरों में भी प्रायः दोनों ही रूप हमें देखने को मिल जाते हैं। बड़े खेद का विषय था कि रामपुर गाँव का प्रधान सहकारी स्कूल के मिड-डे-मील की व्यवस्थाओं में घपले के आरोप में शामिल पाया गया। दीनानाथ अध्यापक का परम शिष्य रहा सूरजसिंह अब गाँव का प्रधान था। गुरु का ज्ञान व ईमानदारी को कलेजे से लगाए जीवन भर जिन आदर्शों पर वह चला आज उसके इस व्यवहार ने उसके पालन-पोषण एवं शिक्षाओं को कलंकित कर दिया था।

भरे गाँव में जब अधिकारियों की कार्यवाही हुई तो सभी शिकायतें जो सरजसिंह के विषय में की गईं थीं. सही पाई गईं। उसे ‘काटो तो खून नहीं’ ऐसा महसूस हो रहा था क्योंकि उसका पक्ष लेने वाले मास्टर दीनानाथ को अपने शिष्य पर पूर्ण विश्वास व आस्था थी।

परन्तु जब अधिकारियों ने मास्टर साहब के सामने उसकी कलई खोल दी और सारे प्रमाण दिखा दिए जिससे साबित हो सकता था कि उसने व्यवस्थाओं में हेर-फेर करके पैसा कमाया व जो सुविधाएँ सरकार के द्वारा बच्चों के लाभार्थ दी गईं उन्हें बिना बच्चों को प्रदान किए स्वयं ही स्वीकृति दे दी। उनका यह व्यवहार न केवल इन्सानियत के विरुद्ध था बल्कि इसलिए भी अपमानजनक था कि जिस पद पर समाज के हितार्थ वह आसीन है उसके विपरीत आचरण उन्होंने किया। गाँव के विकास व कल्याण हेतु उन्हें मुखिया बनाया गया, उस पद को भी उन्होंने कलंकित किया।

सारा गाँव अवाक् खड़ा उनको पुलिस के साथ जाते देख रहा था। आज ऐसा व्यवहार समाज में बहुत बढ़ गया है। शीघ्रातिशीघ्र धन कमाने का प्रलोभन इन्सान को कर्तव्यनिष्ठा से विचलित कर रहा है। ऐसे ही हमारे देश में अधिकांश नेता, अधिकारी, कर्मचारी व आम आदमी करते आ रहे हैं। वास्तव में हर व्यक्ति अपने देश का एक भाग है उसके क्रियाकलापों का प्रभाव देश पर ही पड़ता है। आम आदमी देश की ही शृंखला है। इसलिए कहते हैं नैतिक पतन से देश का पतन होता है।

Question 2.
Read the following passage and briefly answer the questions that follow :-
निम्नलिखित अवतरण को पढ़कर, अन्त में दिये गये प्रश्नों के संक्षिप्त उत्तर लिखिए :

महाभारत का युद्ध जारी था। भीष्म और द्रोणाचार्य का वध हो चुका था और सेनाध्यक्ष पद की बागडोर दुर्योधन ने कर्ण को सौंपी थी। उसके रणकौशल के सामने पाण्डव-सेना के छक्के छूटने लगे। स्वयं अर्जुन भी हतोत्साहित हो गया था, किन्तु कर्ण का दुर्भाग्य कहिए या परशुराम का श्राप, कर्ण के रथ का पहिया कीचड़ में फंस गया। यह देख कर्ण तत्काल रथ से कूदा और उसे निकालने की कोशिश करने लगा।

श्रीकृष्ण ने कर्ण की यह स्थिति देख अर्जुन को उस पर बाणवर्षा जारी करने का इशारा किया। अर्जुन ने उनके निर्देशों का पालन किया, जिससे कर्ण बाणों के आघात को सहन न कर सका। वह अर्जुन से बोला, “महाधनुर्धर, थोड़ी देर रुक जाओ। क्या तुम्हें दिखाई नहीं देता कि मेरा ध्यान रथ का पहिया निकालने में जुटा हुआ है? क्या तुम नहीं जानते कि जो योद्धा रथविहीन हो, जिसके शस्त्र नष्ट हो गये हों, या जो निहत्था हो, युद्ध रोकने की प्रार्थना कर रहा हो, ऐसे योद्धा पर धर्मयुद्ध के ज्ञाता और शूरवीर शस्त्र प्रहार नहीं करते? इसलिये महाबाहो! जब तक मैं इस पहिये को न निकाल लूँ, मुझ पर प्रहार न करो, क्योंकि यह धर्म के अनुकूल नहीं होगा।”

कर्ण द्वारा अर्जुन को दिया गया यह उपदेश श्रीकृष्ण को शूल की तरह चुभा। वे कर्ण से बोले, “बड़े आश्चर्य की बात है कि आज धर्म याद आ रहा है। सच है कि जब नीच मनुष्य विपत्ति में पड़ता है, तो उसे अपने कुकर्मों की याद तो नहीं आती, मगर दूसरों को धर्मोपदेश देने का विचार अवश्य आता है। उचित होता, तुमने अपने धूर्त कर्मों और पापों का विचार किया होता। हे महावीर कर्ण! जब दुर्योधन के साथ मिलकर तुमने पाण्डवों के लिए लाक्षागृह बनवाया, भीम को खत्म करने के इरादे से विष दिलवाया, तेरह वर्ष की अवधि समाप्त होने के बाद भी पाण्डवों को राज्य नहीं दिया, द्रौपदी का चीरहरण करवाया, निहत्थे अभिमन्यु को तुम्हारे समेत सात महारथियों ने मारा, तब तुम्हारा धर्मज्ञान कहाँ गया था? क्या तुम्हें तब धर्मपालन की विस्मृति हो गयी थी।”

कर्ण को इसका उत्तर देते न बना। वह अर्जुन की बाणवर्षा के सामने टिक न सका और धराशायी हो गया।

Questions :
(a) भीष्म और द्रोणाचार्य की मृत्यु के बाद दुर्योधन ने सेनाध्यक्ष की बागडोर किसे सौंपी और क्यों?
(b) युद्ध के मैदान में कर्ण के साथ क्या घटना घटित हुई? [4]
(c) लगातार बाणवर्षा के आघात को सहन न कर सकने पर कर्ण ने अर्जुन को रोकते हुए क्या कहा? [4]
(d) कर्ण की उपदेश भरी बातों को सुनकर श्रीकृष्ण ने क्या जवाब दिया?
(e) प्रस्तुत गद्यांश को पढ़कर आपको क्या शिक्षा मिलती है? समझाकर लिखिए। [4]
Answer:
(a) भीष्म और द्रोणाचार्य की मृत्यु के पश्चात् दुर्योधन ने कर्ण को अपना सेनाध्यक्ष बनाया क्योंकि वह एक शूरवीर योद्धा था। उसके रणकौशल के सामने पाण्डव सेना का टिक पाना असम्भव था। यहाँ तक कि अर्जुन भी कर्ण के सामने घबरा जाता था। कर्ण की वीरता पर दुर्योधन को विश्वास था।
(b) युद्ध के मैदान में दुर्भाग्यवश कर्ण के रथ का पहिया कीचड़ में धंस गया। कर्ण ने तत्काल रथ से उतर कर उसे निकालने की कोशिश की, परन्तु वह उसे निकालने में असमर्थ रहा। उधर श्रीकृष्ण ने मौके का फायदा उठाकर अर्जुन से कर्ण पर लगातार बाणवर्षा करने को कहा। अर्जुन ने भी कृष्ण का कहना मानते हुए लगातार कर्ण पर बाणवर्षा जारी रखी।
(c) लगातार बाणवर्षा के आघात को सहन न कर सकने पर कर्ण ने अर्जुन को रोकते हुए कहा- महाधनुर्धर तुम कुछ समय के लिए वार करना बन्द कर दो क्योंकि तुम देख रहे हो कि मैं अपने रथ का पहिया निकालने में व्यस्त हूँ और तुम युद्ध के नियम जानते हो कि युद्ध में यदि योद्धा रथविहीन हो या किसी योद्धा के ‘शस्त्र’ नष्ट हो गए हों या वह निहत्था हो और युद्ध को रोकने की प्रार्थना कर रहा हो तो ऐसे योद्धा पर धर्म युद्ध के ज्ञाता और वीर पुरुष वार नहीं करते। इसलिए जब तक मैं रथ का पहिया बाहर न निकाल लूँ तब तक तुम मुझ पर तीर मत चलाओ क्योंकि धर्म के अनुसार यह उचित नहीं होगा।
(d) कर्ण की उपदेश भरी बातों को सुनकर श्रीकृष्ण ने जवाब देते हुए कहा कि अब तुम्हें धर्म की बातें याद आ रही हैं जब तुम विपत्ति में पड़े हो, तब तुम्हारा धर्म कहाँ था जब तुमने दुर्योधन के साथ मिलकर लाक्षागृह बनवाया था। पाण्डवों को जिन्दा जलाने के लिए तथा भीम को विष देकर मारने का प्रयास किया। तेरह वर्ष समाप्त होने पर भी पाण्डवों का राज्य वापस नहीं किया। द्रौपदी का चीरहरण करवाया और इतना ही नहीं तुम्हारे समेत सात महारथियों ने मिलकर अकेले अभिमन्यु को घेरकर मार डाला तब तुम्हारा धर्मज्ञान कहाँ गया था। उस समय तुम्हें धर्मपालन की याद नहीं आई।
(e) प्रस्तुत गद्यांश को पढ़कर हमें यह शिक्षा मिलती है कि हमें हमेशा धर्मज्ञान अर्थात् नियमों को याद रखना चाहिए। विपत्ति के समय, स्वार्थवश नियमों या धर्म को याद करना नीचता है। वह इंसान नीच ही होता है जो बारी आने पर धर्म की बात करता है। दूसरों के मौके पर धर्म को याद करता है। घमण्ड में चूर रहकर दूसरों को नुकसान पहुंचाता है जैसा कि कर्ण का व्यवहार था।

Question 3.
(a) Correct the following sentences :
निम्नलिखित वाक्यों को शुद्ध करके लिखिए:
(i) गर्म गाय का दूध स्वास्थ्यवर्धक होता है।
(ii) कृपया मेरे पत्र पर ध्यान देने की कृपा करें।
(iii) प्रत्येक व्यक्तियों का यह कर्तव्य है।
(iv) बच्चा दूध को रो रहा है।
(v) मैंने उसे हजार रुपया दिया।

(b) Use the following idioms in sentences of your own to illustrate their meaning : [5]
निम्नलिखित मुहावरों का अर्थ स्पष्ट करने के लिए वाक्यों में प्रयोग कीजिए:
(i) खाक छानना।
(ii) घोड़े बेच कर सोना।
(ii) नाक में दम करना।
(iv) दिन फिरना।
(v) मीन मेख निकालना।
Answer:
(a) (i) गाय का गर्म दूध स्वास्थ्यवर्धक होता है।
(ii) कृपया मेरे पत्र पर ध्यान दें।
(iii) प्रत्येक व्यक्ति का यह कर्तव्य है।
(iv) बच्चा दूध के लिए रो रहा है।
(v) मैंने उसे हजार रुपये दिए।

(b) (i) बेरोजगारी के कारण लाखों नौजवान सड़कों की खाक छानते फिरते हैं।
(ii) परीक्षाएँ समाप्त होने पर विद्यार्थी घोड़े बेच कर सोते
(iii) अवकाशकाल में बच्चे अपनी माँ की नाक में दम कर देते हैं।
(iv) बेटे की नौकरी लगते ही वर्मा जी के दिन फिर गए।
(v) कविता को मीन मेख निकालने की आदत है।

Section B is not given due to change in present Syllabus.

ISC Class 12 Hindi Previous Year Question Papers

ISC Political Science Question Paper 2011 Solved for Class 12

ISC Political Science Previous Year Question Paper 2011 Solved for Class 12

Maximum Marks: 80
Time allowed: Three hours

  • Candidates are allowed additional 15 minutes for only reading the paper. They must NOT start writing during this time.
  • Answer Question 1 (Compulsory) from Part I and five questions from Part II, choosing two questions from Section A, two questions from Section B and one question from either Section A or Section B.
  • The intended marks for questions or parts of questions are given in brackets [ ].

Part—I
(Compulsory)

Question 1.
(i) Name the six forms of Aristotle’s classification of states.
(ii) How is the role of the executive defined and limited in a Liberal Democracy?
(iii) Why is the existence of a Supreme Court a necessity in a federation ?
(iv) What is meant by Cabinet Dictatorship ? Which type of government would you associate it with ?
(v) Enumerate two important factors that have led to a centralizing tendency in the contemporary federations.
(vi) State two reasons why the British cons-titution has been classified as unwritten.What is Delegated Legislation ?
(vii) What is Delegated Legislation ?
(viii) What are the two methods of election ? Explain each briefly.
(ix) Mention the main bases for the formation of political parties.
(x) Why is the Bureaucracy viewed as neut¬ral ?
(xi) What is the difference between a real and a nominal executive ?
(xii) Does the Indian Constitution strictly follow the Theory of Separation of Powers ? Explain briefly.
(xiii) What is a Jury System ?
(xiv) What is Casteism ?
(xv) Suggest two measures for dealing with the problem of communalism in India.
Answer:
(iii) A federation necessarily requires a strong and impartial judiciary which is protecting the supremacy of the constitution. The method of ensuring this is judicial review as a power.

Working of a federation always involves the possibility of deputes of jurisdiction between center and state for the purpose of resolving this, a strong judiciary is needed.

(v) (a) The role of the central government as a coordinating agency among state governments has made it stronger.
(b) The central government has control over the vitally important areas like- defense, foreign affairs and international trade.

(viii) The two types of elections are Direct election and Indirect election. Direct election is a democratic system wherein the people elect their representatives through free and fair direct election and by casting their votes, for example, in the Indian system. Indirect election is the system in . which people elect their representatives indirectly through an electoral college. For example, the election of the American President.

(xi) The difference between real and nominal executive is evident in parliamentary system of government. Nominal executive only has power the name, decisions are taken in his name, but he was no say in the same, while rest executive holds all the power of taking decisions.

(xiv) Caste determines social and political life of India. Caste determines the nature, organi-zation and working of political groups, bureaucracies and all political structures and it is the biggest challenge to democracy.

(xv) Reforms in the education system and proper use of mass media to spread awareness are two ways to deal with the problem of communalism.

Section—A
Answer two questions

Question 2.
(a) Discuss JA.R. Marriot’s classification of the forms of government. [8]
(b) Distinguish between the features of the political systems of USA and UK, based on Leacock’s classificatory scheme. [6]

Question 3.
(a) Examine the essential features of a Unitary System of government. State four advantages and four disadvantages of such a system. [8]
(b) Distinguish between a Federation and a Confederation. [6]
Answer:
(a) Features of a Unitary Government are as follows:
A single central all powerful government : All the powers are vested with one single central government whose authority is supreme over all the parts and people of the state.

Local governments exist at the will of the central government: The local governments are created and vested with powers, by the central government. They work as administrative units of the central government and operate as the central government directs.

Constitution can be written or unwritten : Since there is no division of powers and all the powers belong to the central government, there is no special need for a written constitution. But it may be written or unwritten in accordance with the wishes of the people.

Flexibility of constitution and administration : The central government alone has the power to amend the constitution and in this sense the constitution is always flexible.

Single uniform administration : The existence of an all powerful central government exercising power over all the people and places leads to single stable administration for the whole state.

Advantages:

  1. It is simple, less expensive and is ideally suitable for small states 2. It is a source of strong, all powerful government.
  2. It ensures single uniform administration for the whole of the state.
  3. Suitable for meeting emergencies because being an all powerful government, the central government can take all necessary decisions quickly.

Disadvantages:

  1. Possibility of a central dictatorship and avoidance of local areas.
  2. Central government often behaves inefficiently because of being over-burdened with all the work.
  3. Being located at the center and faced with pressing national problems, it very often fails to satisfy local needs.
  4. Unitary government is suitable only for small and homogeneous states.

Question 4.
(a) Would it be correct to classify Constitutions into rigid and flexible type’s in absolute terms? Argue your case with relevant examples from UK, USA and India. [8]
(b) State six essential qualities of a good Constitution. [6]
Answer:
(a) Rigid Constitution refers to constitutions which can’t be amended or repealed like ordinary laws. In case it needs formal procedures for amendment it is formal constitution. It can’t be amended by ordinary legislature through ordinary legislative procedures. It also involves a strick distinction between ordinary laws and constitutional laws as constitutional laws are considered to be superior to ordinary laws.

US Constitution is considered to be a rigid constitution as it has undergone just 27 amendments during the past 220 years. It is possible to pass ordinary law in US by simple majority of the Congress. However, the amendment of constitutional law is possible through 2/3rd majority or Congress and 3/4th of the states.

Flexible constitution is the one which can by ordinary legislative through ordinary legislative procedure. It treats constitutional laws and ordinary laws as same. It is possible to make ‘ constitutional amendment through simple majority of the legislature.

A typical example of flexible constitution is that of UK. The British Parliament can pass, amend or repeal any constitutional law through legislative process as both constitutional laws and ordinary laws are considered as equal. Indian constitution maintains a balance bet¬ween flexibility and rigidity as it is neither too rigid nor too flexible. For amending the laws. the constitution is divided into following :

  1. Some provisions of the constitution can be amended by simple majority in Parliament, (flexible)
  2. Some provisions can be amended by 2/3rd majority in Parliament and approved by at least 50% states, (rigid)
  3. The remaining provisions can be amended by 2/3rd majority in Parliament, (rigid)

Question 5.
(a) Explain the system of Proportional Representation with reference to the two schemes Single Transferable Vote System and the List System. [8]
(b) What is bi-party system ? Examine the merits and demerits of this system. [6]
Answer:
(b) When a state has only two major political parties in the political system, it is said to have the bi-party or the dual party system. In bi¬party system the party which gets majority in the elections, forms the government and the other party assumes the role of opposition party. In bi-party system, there is a distinct polarization of opinion on different issues. Merits of Bi-party System :

Following are the merits of a bi-party system :

  1. Bi-party system is the essence of a successful parliamentary democracy. It ensures a strong and stable government.
  2. The opposition under the dual-party system is more responsible and dignified.
  3. The bi-party system offers a clear-cut alternative to the voters as they have to choose one out of two parties.
  4. Formation of governments is simple and easy under the dual-party system.
  5. It is easy to fix responsibility for failure of national policy in a bi-party system.

Demerits of the Bi-party System :

  1. The bi-party system splits the nation into two irreconcilable camps.
  2. Under the bi-party system the majority party in the legislature might have been elected with minority of votes and thus, the minority may rule in the name of majority.
  3. The bi-party system undermines the prestige of the legislature and-results to cabinet dictatorship.
  4. The bi-party system leads to despotism of the majority which rides rough-shod over the wishes of the minorities.

Question 6.
(a) Make a comparative study of the composition and powers of the US Senate and the Indian Rajya Sabha. [8]
(b) State six limitations imposed on the Sovereignty of the British Parliament. [6]
Answer:
(a) Composition : There are 100 members in the Senate, while there are 250 members in the Rajya Sabha. In the Senate, majority of its members (51) constitute the quorum for its meetings, in Rajya Sabha 1/10th of the members constitute the quorum. Tenure of member of both Senate and Rajya Sabha is six years.

Powers : Senate is the strongest upper house in the world and has power of making laws and passing of bills. It has the power to change and modify money bills. While Rajya Sabha has equal powers as the lower house but loses out in a conflict and it has the power to delay money bill passed by Lok Sabha, for a maximum period of 14 days. Senate has the power to ratify treaties and appointment made by the President. Senate plays an effective role in impeaching the President. Rajya Sabha plays an important role in changing the boundries of a state and along with the Lok Sabha, it can impeach the President on charges of violation of the constitution

Question 7.
(a) Discuss the powers of the Executive in modern democratic political systems. [8]
(b) Examine the powers and functions of the President of USA. [6]
Answer:
(a) The Executive enjoys vast powers in the modern systems :
Enforcement of. laws : Executive has the . power to enforce laws, as formulated by the legislature, and to maintain law and order in the state. Whenever a breach of law takes place, it is the responsibility of the executive to plug the breach and bring to book the offenders.

Appointment making powers : Executive has the power to appoint the members of the civil services. This is mostly done on the recommendation of a statutory services recruitment commission.

Treaty making powers : Executive has the power to decide as to which treaties are to be signed and with which other countries. The executive negotiates the treaties in accordance with the procedure defined by international law.

Defense powers : The chief executive of the state is also the supreme commander of the armed forces of the state. A minister always heads the defense department and executive has the right to declare war upon an enemy of the state.

Law making under the system of delegated legislation : Under this system, the legislature, because of its limitations, delegates a large amount of its law-making powers to the executive. Most of the bills passed by the legislature are passed in a skeleton form and the details are to be filled in by the executive through delegated legislation.

Power of appointing the judges : In almost all democratic systems, the chief executive has the power to appoint judges. Under the system of administrative adjudication, the executive agencies have the power to hear and decide cases involving administrative activity.

(b) Powers of the President of USA :

  1. Foremost power of the President is to enforce the federal laws and maintaining order throughout the USA.
  2. The President can nominate and with the consent of the Senate shall appoint ambassadors, other public ministers and judges of the Supreme Court.
  3. President has almost unhindered authority to remove persons, he disapproves of as executive members.
  4. The President has the power to make treaties with other states but all such treaties have to be got approved from the Senate with a 2/3rd majority of the members present and voting in the Senate.
  5. President is the commander in chief of the armed forces, so he is authorized to take all necessary measures for the defense of the country.

Functions of the President of USA :

  1. He functions as the head of state and performs all such ceremonial functions which are performed by heads of states of all other countries.
  2. All executive functions are taken by him or under his authority.
  3. He formulates the policies of the administration and secures their execution by his secretaries.
  4. He accredits American ambassadors to other countries and accepts the credentials of the ambassadors of other countries to the USA.
  5. President not only formulates foreign policy but also conducts foreign relations of the USA.

Question 8.
(a) Discuss the functions of the Judiciary in the modern political systems. [8]
(b) Compare the Judicial Systems in India, USA and UK. [6]
Answer:
(a) The judiciary interprets the constitution and safeguards the rights of the people.

The primary function of the judiciary is to hear and decide disputes. In accordance with the recognized procedure, the courts determine the facts of a case.

The judiciary interprets laws and applies them to specific cases that come before it.

The judiciary acts as the guardian of the constitution in a federal government. Consti-tution in a federation is the supreme law of the land.

It is the guardian of civil liberties of the people. It protects individual liberty and also protects people against an arbitrary action of the government.

By interpretation and expansion of existing laws and by deciding cases on the basis of equity for future judicial guidance and reference, the judges create new laws.

In some countries, the courts give advisory opinions when requested to do so, by the executive.

The Supreme Court and High Courts are also empowered to appoint their local officials and subordinate staff. This is an administrative function of the judiciary.

Question 9.
(a) What is Communalism ? State any six causes of communalism in India. [8]
(b) Define Separatism. Suggest four possible ways to combat separatism. [6]
Answer:
(a) There are various causes for the rise of communalism in India:

A Legacy of Past: Communal politics had played its nasty game during the immediate past of independent India. The partition of India was the ultimate outcome of the politics of the British government. Despite the emergence of Pakistan, a large number of Muslims are staying in India. India has adopted the principles of secularism and equality of the people. But communalism as a legacy of past, is continuing and expreshing itself in various form.

Presence of Communal Parties : Religion in India has become an important agency of political socialisation and it is also reflected in the ideology of a number of politial parties. A number of communal and sectarian political parties and organisations are present in India. Same political leaders preach communalism to attain their political goal.

Poverty : Mass poverty and unemploy-ment create a sense of frustration among the people. It generates backwardness, illetracy, ignorance, etc. The unemployed people are easily trapped by religious fundamentalists and fanatics. Such people are used to cause communal riots.

Communalism of Politics : Electoral politics in India has become competitive. Different political parties are not hesitating to use any means for electoral victory. They even create communal tensions and try to take politcal advantage out of it. Concessions are granted to various minority groups for appeasing them.

Cross-Border Factors : Communal tensions in India are sometimes highly intensified due to the rule of two neighbouring theocratic countries. These countries try to create communal problems in the border states. For example; the communal problems of Punjab and Nammu & Kashmir are caused due to
provocation of Pakistan.

The Social Causes : The two major communities, the Hindus and Muslims in India continue tobe suspicious of each other. The Muslims complain of the threat of Hindu cultural invasion and have become more assertive of their rights. Another cause is conversion of religion which creates negative reactions.

ISC Class 12 Political Science Previous Year Question Papers

ISC Physics Question Paper 2013 Solved for Class 12

ISC Physics Previous Year Question Paper 2013 Solved for Class 12

Maximum Marks: 70
Time allowed: 3 hours

  • Candidates are allowed additional 15 minutes for only reading the paper. They must NOT start writing during this time.
  • Answer all questions in Part I and six questions from Part II, choosing two questions from each of the Sections A, B and C.
  • All working, including rough work, should be done on the same sheet as, and adjacent to, the rest of the answer.
  • The intended marks for questions or parts of questions are given in brackets [ ].
  • Material to be supplied: Log tables including Trigonometric functions
  • A list of useful physical constants is given at the end of this paper.

Part-I
(Answer all questions)

Question 1.
(A) Choose the correct alternative (a), (b), (c) or (d) for each of the questions given below : [5]
(i) Relative permittivity of water is 81. If ∈w and ∈0 are permitivities of water and vacuum respectively, then.
(a) \(\epsilon_{0}=9 \epsilon_{w}\)
(b) \(\epsilon_{0}=81 \epsilon_{w}\)
(c) \(\epsilon_{w}=9 \epsilon_{0}\)
(d) \(\epsilon_{w}=81 \epsilon_{0}\)

ii) Five resistors are connected as shown in figure. The effective resistance i.e., equivalent resistance between the points A and B is :
(a) 4 Ω
(b) 5 Ω
(c) 15 Ω
(d) 20 Ω
ISC Physics Question Paper 2013 Solved for Class 12 1
(iii) The Biot Savart’s Law in vector form is:
(a) \(\overline{\delta \mathrm{B}}=\frac{\mu_{0}}{4 \pi} \frac{d l(\overrightarrow{\mathrm{I}} \times \vec{r})}{r^{3}}\)
(b) \(\overline{\delta \mathrm{B}}=\frac{\mu_{0}}{4 \pi} \frac{\mathrm{I}(\overrightarrow{d l} \times \vec{r})}{r^{3}}\)
(c) \(\overline{\delta \mathrm{B}}=\frac{\mu_{0}}{4 \pi} \frac{\mathrm{I}(\vec{r} \times \overrightarrow{d l})}{r^{3}}\)
(d) \(\overline{\delta \mathrm{B}}=\frac{\mu_{0}}{4 \pi} \frac{\mathrm{I}(d \vec{l} \times \vec{r})}{r^{2}}\)

(iv) In an astronomical telescope of refracting type :
(a) Eyepiece has greater focal length
(b) Objective has greater focal length
(c) Objective and eyepiece have equal focal length
(d) Eyepiece has greater aperture than the objective.

(v) The particles which cannot be accelerated by a cyclotron or a Van de Graff generator are :
(a) Alpha particles
(b) Beta particles
(c) Neutrons
(d) Protons [15]

(B) Answer all questions given below briefly and to the point:
(i) A large hollow metallic sphere has a positive charge of 35.4 μC at its center. Find how much electric flux emanates from the sphere.

(ii) A current ‘I’ flows through a metallic wire of radius ‘r’ and the free electrons in it drift with a velocity vd. Calculate the drift velocity of the free electrons through the wire of the same material, having double the radius, when same current flows through it.

(iii) Name any one instrument which works on the principle of tangent law in magnetism.

(iv) State the SI unit of magnetic dipole moment.

(v) Alternating current flowing through a certain electrical device leads over the potential difference across it by 90°. State whether this device is a resistor, capacitor or an inductor.

(vi) What is the shape of the wave front diverging from a point source of light ?

(vii) The critical angle for a given transparent medium and air is ic. A ray of light travelling in air is incident on this transparent medium at an angle of incidence equal to the polarising angle ip. What is the relation between the two angles ic and ip ?

(viii) Find the focal length and nature of a lens whose optical power is – 5D.

(ix) What is Modulation ? Explain in brief.

(x) What are the dark lines seen in the solar spectrum called ?

(xi) What is the relation between wavelength and momentum of moving particles ?

(xii) Name the series of lines in the hydrogen spectrum which lies in the ultra-violet region.

(xiii) Fill in the blank in the given nuclear reaction:
\(\begin{array}{r}{+\frac{27}{13} \mathrm{Al} \longrightarrow_{12}^{25} \mathrm{Mg}+\frac{4}{2} \mathrm{He}} \\ {13}\end{array}\)

(xiv) Give an example where energy is converted into matter.

(xv) To convert a pure semiconductor into n-type semiconductor, what type of impurity is added to it ?
Answer:
Answer.
(A) (i) (d)
(ii) (a)
(iii) (b)
(iv) (b)
(v) (c)

(B) (i) Here, q = 35.4 x 10-6 C
By Gauss Law,
\(\phi=\frac{q}{\epsilon_{0}}=\frac{35 \cdot 4 \times 10^{-6}}{8 \cdot 85 \times 10^{-12}}=4 \times 10^{6} \mathrm{Nm}^{2} \mathrm{C}^{-1}\)

(ii) We know,
ISC Physics Question Paper 2013 Solved for Class 12 2
(iv) The SI unit of magnetic dipole moment is Coulomb-meter (Cm).
(v) The device is a capacitor.
(vi) Spherical.
(vii) If ip is the polarising angle and n the refractive index of the medium, then n = tan ip If ic is the critical angle,then
ISC Physics Question Paper 2013 Solved for Class 12 3
(viii) We know that,
\(f_{m}=\frac{1}{\mathrm{D}}=-\frac{1}{5}=-0 \cdot 2 \mathrm{m}=-20 \mathrm{cm}\)
∴ – ve sign shows that the lens is concave.

(ix) The audio signal cannot be transmitted efficiently as such. Modulation is the process of modifying some characteristic of a high frequency carrier wave (amplitude, frequency or phase) in accordance with the instantaneous value of the amplitude of modulating signal. The modulating signal is then transmitted across space.

(x) Dark lines has also called Fraunhoffer’s lines.

(xi) The required relation is λ = h/p where λ = wavelength and p the momentum of the particle and h = Planck’s constant.

(xii) The series emitted in the hydrogen spectrums in ultra-violet region is called the “Lyman Series.”

(xiii) The missing particle is \(_{1}^{2} \mathrm{H}\)

(xiv) In pair production. It is materialization of energy.

(xv) To get n-type semiconductor, the semiconductor is doped with an impurity like antimony.

Part- II
(Answer any six questions in this part, choosing two questions from each of the Sections A, B and C)

Section-A
(Answer any six questions)

Question 2.
(a) (i) Write an expression (derivation not required) for intensity’ of electric field in :
(1) Axial position.
(2) Broad side position of an electric dipole, in terms of its length (2a) dipole moment (p) and distance (r).
(ii) What is the ratio of these two intensities i.e., E1 : E2, for a short electric dipole ? [3]

(b) Three capacitors C1 = 6 μF, C2 = 12 μF and C3 = 20 μF are connected to a 100 V battery, as shown in figure alongside:
ISC Physics Question Paper 2013 Solved for Class 12 4
Calculate:
(i) Charge on each plate of capacitor C1.
(ii) Electrostatic potential energy stored in capacitor C3. [3]

(c) ‘n’ cells, each of emf’e and internal resistance ‘r’ are joined in series to form a row. ‘m’ such rows are connected in parallel to form a battery of N = mn cells. This battery is connected to an external resistance ‘R’. [3]
(i) What is the emf of this battery and how much is its internal resistance ?
(if) Show that current ‘I’ flowing through the external resistance R’ is given by :
\(\mathrm{I}=\frac{\mathrm{N} e}{m \mathrm{R}+n r}\)
Answer:
ISC Physics Question Paper 2013 Solved for Class 12 5
ISC Physics Question Paper 2013 Solved for Class 12 6
ISC Physics Question Paper 2013 Solved for Class 12 7
ISC Physics Question Paper 2013 Solved for Class 12 8
Question 3.
(a) In the circuit shown in figure, E1 = 17 V, E2 = 21V, R1 = 2 Ω, R2 = 3 Ω and R3 = 5 Ω. Using Kirchhoff’s laws,find the currents flowing through the resistors R1, R2 and R3. (Internal resistance of each of the batteries is negligible). [4]
ISC Physics Question Paper 2013 Solved for Class 12 9
(b) You are provided with one low resistance RL and one high resistance RH and two galvanometers. One galvanometer is to be converted to an ammeter and the other to a voltmeter. Show how you will do this with the help of simple, labelled diagrams. [2]

(c) (i) Plot a labelled graph to show variation of thermo-emf ‘e’ versus temperature difference ‘θ’ between the two junctions of a thermo couple. Mark ‘N’ as neutral temperature and ‘I’ as temperature of inversion.
(ii) What is Peltier effect ? [3]
Answer:
(a) The distribution of current in the given circuit is as given below:
Apply first law at the point E
ISC Physics Question Paper 2013 Solved for Class 12 10
ISC Physics Question Paper 2013 Solved for Class 12 11
ISC Physics Question Paper 2013 Solved for Class 12 12
(b) A galvanometer is connected into an ammeter by connecting a low resistance of a suitable value in parallel with the galvanometer and into a voltmeter by connecting a high resistance in series with galvanometer as shown in fig. (a) and (b) respectively.
ISC Physics Question Paper 2013 Solved for Class 12 13
Question 4.
(a) Figure below shows two infinitely long and thin current carrying conductors X and Y kept in vacuum, parallel to each other, at a distance ‘a’. [2]
ISC Physics Question Paper 2013 Solved for Class 12 14
(i) How much force per unit length acts on the conductor Y due to the current flowing through X ? Write your answer in terms of (μ0/4π), I1, I2 and a.
(Derivation of formula is not required.)
(ii) Define ampere, in terms of force between two current earning conductors.

(b) A metallic rod CD rests on a thick metallic wire PQRS with arms PQ and RS parallel to each other, at a distance l = 40 cm, as shown in figure. A uniform magnetic field B = 0.1 T acts perpendicular to the plane of this paper, pointing inwards (i.e,.. away from the reader). The rod is now made to slide towards right, with a constant velocity of v = 5 0 ms-1. [3]
ISC Physics Question Paper 2013 Solved for Class 12 15
(i) How much emf is induced between the two ends of the rod CD ?
(ii) What is the direction in which the induced current flows.

(c) (i) Figur below shows a series RCL circuit connected to an ac source which generates an alternating emf of frequency 50 Hz. The readings of the voltmeters V1 and V2 are 80 V and 60 respectively.
ISC Physics Question Paper 2013 Solved for Class 12 16
Find :
(1) the current in the circuit.
(2) the capacitance C of the capacitor.
(ii) At resonance, what is the relation between impedance of a series LCR circuit and its resistance R ?
Answer:
(a) (i) Two conductors X and Y of infinite length and separted by a distance ‘a’ carry currents I1 and I2 respectively. The force of attraction per unit length is given by
\(\frac{\mathrm{F}}{l}=\frac{\mu_{0}}{4 \pi} \frac{2 \mathrm{I}_{1} \mathrm{I}_{2}}{a}\)
ISC Physics Question Paper 2013 Solved for Class 12 17
(ii) Definition of Ampere : 1 ampere is that much current which when flowing through each of the two infinitely long conductors separated by 1 m in vacuum results in a force 2 x 10-7 newton per meter length on each of them.
ISC Physics Question Paper 2013 Solved for Class 12 18
ISC Physics Question Paper 2013 Solved for Class 12 19
ISC Physics Question Paper 2013 Solved for Class 12 20

Section-B
(Answer any two questions)

Question 5.
(a) (i) In an electromagnetic wave, how are electric vector \((\overrightarrow{\mathrm{E}})\), magnetic vector \((\overrightarrow{\mathrm{B}})\) and velocity of propagation of the wave \((\overrightarrow{\mathrm{C}})\) oriented ? [2]
(ii) How long wold gamma radiation take to travel from sun to earth, a distance of 1.5 x 1011 m?

(b) With the help of a labelled diagram, show that fringe separation β in Young’s double slit experiment is given by : [4]
\(\beta=\frac{\lambda \mathrm{D}}{d}\)
where the terms have their usual meaning.

(c) (i) What is the difference between polarised light and unpolarised light based on the direction of electric vector \((\overrightarrow{\mathrm{E}})\) ?
(ii) What will be the effect on the width of the central bright fringe in the diffraction pattern of a single slift if:
(1) Monochromatic light of smaller wavelength is used.
(2) Slit is made narrower. [2]
Answer:
ISC Physics Question Paper 2013 Solved for Class 12 21
ISC Physics Question Paper 2013 Solved for Class 12 22

(b) In the fig. are shown two coherent sources S1 and S2 illuminated by the same source S.d is the distance between the two sources. D is the disance between the source and the screen. P is a point on the screen having an angle height θ. O is a point on the screen situated symmetrically w.r.t. two sources. Let OP = Y. Path difference between the disturbances starting from S2 and Si reaching P. is
\(p=\mathrm{S}_{2} \mathrm{P}-\mathrm{S}_{1} \mathrm{P}\)
ISC Physics Question Paper 2013 Solved for Class 12 23
ISC Physics Question Paper 2013 Solved for Class 12 24
ISC Physics Question Paper 2013 Solved for Class 12 25
(c) (i) (1) In unpolarised light, the direction of the electric field vector is in any direction in a plane perpendicular to the direction of propagation.
(2) In polarised light, the direction of the electic vector is one particular direction perpendicular to the direction of propagation.
(ii) Width of the central maxima \(\left(2 \frac{\lambda}{a} \mathrm{D}\right)\) clearly :

  • decreases when the wavelength is decreased.
  • increases when the slit width V decreases.

Question 6.
(a) At what angle, a ray of light should be incident on the first face AB of a regular glass prism ABC so that the emergent ray grazes the adjacent face AC ? [3]
ISC Physics Question Paper 2013 Solved for Class 12 26
See figure alongside. (Refractive Index of glass = 1.6)

(b) A convex lens ‘L’ and a plane mirror ‘M’ are arranged as shown in figure below. Position of object pin ‘O’ is adjusted in such a way that the inverted image ‘I’ formed by the lens mirror combination, coincides with the object pin ‘O’. Explain how and when this happens. [2]
ISC Physics Question Paper 2013 Solved for Class 12 27
(c) Starting with an expression for refraction at a single spherical surface, obtain an expression for lens maker’s formula. [3]
Answer:
ISC Physics Question Paper 2013 Solved for Class 12 28
ISC Physics Question Paper 2013 Solved for Class 12 29
(b) When the object O is at the focus of the lens, the rays after refraction through the lens will be rendered parallel to the principal axis. These rays will strike the mirror normally and will retrace their path giving rise to an image, which will coincide with O as shown.
ISC Physics Question Paper 2013 Solved for Class 12 30
(c)
ISC Physics Question Paper 2013 Solved for Class 12 31
Consider a thin convex lens XY having R1 and R2 as the radii of curvatures of the two surfaces XP1Y and XP2Y respectively. Let n1 be the refractive index of the medium in which the object O is situated and n2 the refractive index of the material of the lens. The first surface of radius of curvature R1 forms the image of the object O at a distance v’ from the surface XP1Y. This image then serves as a virtual object for the second surface giving rise to a final image at I. For refraction at the first surface, we can write
ISC Physics Question Paper 2013 Solved for Class 12 32
ISC Physics Question Paper 2013 Solved for Class 12 33

Question 7.
(a) Show that the axial chromatic aberration (fr – fv) for a convex lens is equal to the product of its mean focal length (f) and dispersive power (ω) of its material i.e., Prove : [3]
fr – fv = ωf

(b) Draw a labelled diagram of an image formed by a compound microscope, with the image at least distance of distinct vision. Write any one expression for its magnifying power. [3]

(c) What is meant by long-sightedness ? How can this defect be corrected ? [2]
Answer:
ISC Physics Question Paper 2013 Solved for Class 12 34
ISC Physics Question Paper 2013 Solved for Class 12 35
ISC Physics Question Paper 2013 Solved for Class 12 36
ISC Physics Question Paper 2013 Solved for Class 12 37
Here, μ0 and v0 are the objective distance and v0 the image distance from the objective lens and D, the distance of distinct vision.

(c) A person suffering from long-sightedness can see clearly far objects but the close objects appear to be blurred. This defect can be corrected by using a convex lens of a proper focal length.

Section-C
(Answer any two questions)

Question 8.
(a) (i) What is meant by ‘Quantization of charge’ ?
(ii) In Thomson’s experiment, prove that the ratio of chage to the mass (e/m) of an electron is given
\(\frac{e}{m}=\frac{1}{2 \mathrm{V}} \cdot \frac{\mathrm{E}^{2}}{\mathrm{B}^{2}}\)
where the terms have their usual meaning. [3]

(b) In a photoelectric cell, a retarding potential of 0.5 V is required to block the movement of electrons from the cathode when monochromatic light of wavelength 400 nm is incident on its surface. Find the work function of the material of the cathode. [3]

(c) Name a phenomenon or an experiment which proves : [2]
(i) Particle nature of electromagnetic radiations. [2]
(ii) Wave nature of particles.
(Description of the phenomenon/experiment is not required).
Answer:
ISC Physics Question Paper 2013 Solved for Class 12 38
(c) (i) The phenomenon of photoelectric effect proves the particle nature of electromagnetic radiations.
(ii) Davisson-Germer experiment proves the wave nature of particles.

Question 9.
(a) (i) State the postulate of Bohr’s theory regarding : [3]

  • Angular momentum of an electron.
  • Emission of a photon.

(ii) Total energy of an electron orbiting around the nucleus of an atom is always negative. What is the significance of this ?

(b) (i) Draw a labelled graph showing variation of relative intensity of X-rays versus their wavelength λ. Mark λmin on the graph.
(ii) State how the value of λmin can be varied. [3]

(c) Half life of a certain radioactive substance is 6 hours. If you had 3.2 kg of this substance in the beginning, how much of it will disintegrate in one day ? [2]
Answer.
(a) (i) Postulates of Bohr’s theory:
(1) According to angular momentum postulate, electron can revolve only in those orbits around the nucleus in which its angular momentum is an integral multiple of h/2π
i. e., \(m v r=n \frac{h}{2 \pi}\)

(2) The emission or absorption of energy takes place when an electron jumps from one excited state to the other. If the energy state E2 to the energy state E1, the frequency of the emitted radiation is given by
E2-E1 =hv
(ii) If the total energy of an electron orbiting around the nucleus is – ve, it implies that the electron is not free but is bound to the nucleus. To make the electron, energy must be supplied from an external source.

(b) (i) The relation between the intensity of X-rays emitted and the corresponding wavelength is as shown below
ISC Physics Question Paper 2013 Solved for Class 12 39
ISC Physics Question Paper 2013 Solved for Class 12 40
Question 10.
(a) (i) What is the significance of binding energy per nucleon of a nucleus ?
(ii) In a certain star, three alpha particles undergo fusion in a single reaction to form \(_{6}^{12} \mathrm{C}\) nucleus. Calculate the energy released in this reaction in MeV
\(\text { Given } : m\left(\begin{array}{l}{4} \\ {2}\end{array} \mathrm{He}\right)=4 \cdot 002604 \mathrm{u} \text { and } m(\stackrel{12}{6} \mathrm{C})=12 \cdot 000000 \mathrm{u} \text { . }\)  [3]

(b) Show by drawing labelled diagrams, the nature of output voltages in case of: [3]
(i) A half wave rectifier.
(ii) A frill wave rectifier. .
(iii) An Amplifier,
(In each case, input is an ac voltage)
Circuit diagrams of these devices are not required.

(c) Identify the logic gate whose truth table is given below and draw its symbol: [2]
ISC Physics Question Paper 2013 Solved for Class 12 41
ISC Physics Question Paper 2013 Solved for Class 12 42
Answer:
(i) Binding energy per nucleon of a nucleus is defined as the average energy required to remove a nucleon from the nucleus. More the binding energy per nucleon more stable is the nucleus.
ISC Physics Question Paper 2013 Solved for Class 12 43

ISC Class 12 Physics Previous Year Question Papers

ISC Political Science Question Paper 2012 Solved for Class 12

ISC Political Science Previous Year Question Paper 2012 Solved for Class 12

Maximum Marks: 80
Time allowed: Three hours

  • Candidates are allowed additional 15 minutes for only reading the paper. They must NOT start writing during this time.
  • Answer Question 1 (Compulsory) from Part I and five questions from Part II, choosing two questions from Section A, two questions from Section B and one question from either Section A or Section B.
  • The intended marks for questions or parts of questions are given in brackets [ ].

Part-I
(Compulsory)

Question 1.
Answer briefly each of the questions (i) to (xv): [15 x 2]
(i) What was the basis of Plato’s classification of States ?
(ii) What is the meaning of social and economic checks and balances in a liberal democracy?
(iii) Give one example each of a unitary state which is democratic, and one which is a totalitarian communist state.
(iv) What is Unitary Federalism ?
(v) Mention two demerits of a written constitution.
(vi) What is the role of civil servants in policy formation ?
(vii) What is meant by single member constituency ?
(viii) Explain the second ballot system.
(ix) What is a reserved constituency in the Indian electoral system ?
(x) With whom are the residuary powers vested in :
(a) U.S.A.
(b) India?
(xi) List two ways in which the Lok Sabha is more powerful than the Rajya Sabha.
(xii) Define senatorial courtesy.
(xiii) Mention the basic difference between the position of the British Speaker and his American counterpart.
(xiv) What is the mode of appointment of judges in the apex courts of:
(a) India (b) U.K.?
(xv) Suggest any two measures to eradicate economic inequality.
Answer:
(ii) In a Liberal Democracy, there are social and economic checks and balances on the activities of government. Characterized by the active existence and activities of a large number of social and economic associations/ interest groups of the people which enjoy a large amount of autonomy, therse groups play a vital role in the political process. The bulk of economic decisions in liberal democracy is not taken by the government alone. The interest groups play a vital role in it. The duty of the government is to harmonies and coordinate social and economic relations and to impose and coerce only in the last resort.

(iii) The example of a unitary and democratic state is United Kingdom. The example of a totalitarian communist state is China.

(iv) Unitary federalism means a federation with increasing powers of union government vis – a – vis states.

(v) Two demerits of written constitution :
(a) It does not keep pace with changing conditions.
(b) It is not suitable to face emergencies.

(vi) The civil servants provide necessary information and advice to political executive in policy formulation.

(x) In USA, residuary powers are vested in the states. In India, residuary powers are vested, in the union government.

(xi) Lok Sabha is more powerful than Rajya Sabha in following two areas :

  • No-confidence motion against council of ministers is passed only in Lok Sabha.
  • A Money Bill can be introduced in Lok Sabha only.

Part-II
Section—A
Answer two questions

Question 2.
(a) Explain Aristotle’s classification of States and critically evaluate it. [8]
(b) No rule is better than self-rule. In the light of the statement, discuss any six merits of Liberal Democracy. [6]
Answer:
(b) Self rule means rule by the persons who are selected on the consent of ruled. No rule is better than self rule because it provides best opportunities for the development of persona¬lities of people, effective participation of people in the government process and more effective channels of political education. Self rule is the chief characteristics of liberal democracy. Merits of Liberal Democracy : Liberal Democracy is characterized by representative government, provision for civil liberties and limited scope of the government.
Six main merits of liberal democracy are given below:

  1. It provides for the rule of people and a representative and responsible government.
  2. It promotes the interest of common man. It provides various social, economic and political benefits to all citizens.
  3. It provides for a secure and stable government with no chance for revolutions and violent means.
  4. It treats all people as equal partners and participants in the government. Equality is the cornerstone of liberal democracy.
  5. It provides for effective channels for popular participation and political education to people.
  6. It prevents the arbitrary and despotico rule as rulers are changed after regular interval through periodic election.

Question 3.
(a) Define Federation and explain its four merits and four demerits. [8]
(b) Give any six salient features of a parliamentary system of government. [6]
Answer:
(a) Federation is a form of government where ruling powers are divided between the union or central government and the state or provincial governments. The division of powers is effected through the constitution. According to Garner, federal government can be defined as a system of central and local (provincial) governments, combined under common sovereignty, both the central and local governments being supreme within definite spheres, marked out for them by the general constitution or by the act of parliament which creates the system.
Merits of Federation : A federation is opposite to unitary government. It has the following merits :

Suitable for big states : Big states cannot be ruled from a single center. Hence federal governments are best suited for big states to ensure effective rule.

Division of work : A federation works on the basis of division of work and responsibilities between the union and provincial governments. Hence both governments are able to concentrate on the designated functions.

Truly Democratic : A federal state is truly democratic as it decentralizes political authority, checks arbitrariness of central government and provides more opportunity for political participation and political education.

Society is federal : According to Laski, since society is federal in nature, the federation is the natural form of government.

Demerits of federation : A federal state displays the following demerits :

Division of power is a source of weakness : A federation is based on the principle of division of powers between the central and state governments. This leads to the weakness of both the governments as none of them is in the control of all powers.

Division of responsibility : A federal environment also divides responsibility between the central and state governments. The division of responsibility undermines effective administration and creates problem of coordination.

Conflict between center and states : In federal state the division of power leads to strain between central and state governments. This may result in serious conflicts between the two.

Not suitable for emergencies : A federal system is also not suitable to face emergencies as due to lack of coordination, the decision making is delayed.

(b) Six salient features of parliamentary form of government : Governments are classified as parliamentary and presidential on the basis of relationship between executive and legislature. If executive is responsible to the popular house of legislature, it is known as parliamentary form of government. If executive is not responsible to the legislature, it is known as presidential form of government.
Following are the salient features of parliamentary form of government:

Responsibility of executive to legislature : In parliamentary system, executive (council of ministers) is responsible to the power house of legislature. That means the legislature may pass a no-confidence motion any time to remove the executive.

Close relation between executive and legislature : In parliamentary system, there is close relation between the executive (council of ministers) and legislature (parliament) as the members of council of ministers are required to be the members of legislature and they participate in the meetings of legislature.

Indeterminate term of executive : In parliamentary system the term of office of executive is not fixed as it can be removed by legislature any time before its normal term by passing a no-confidence motion.

Collective responsibility : In parliamentary system, the members of executive are collectively responsible to legislature. It means that if a no-confidence motion is passed against a single member of council of ministers, the entire executive has to go.

Difference between real and nominal executive : In parliamentary system, there are two types of executive e.g^one nominal and another real. In India, the president is nominal executive and the council of ministers is the real executive, which exercises all powers in practical.

Strong position of Prime Minister : In parliamentary system, the executive or council of ministers is headed by the Prime Minister, who presides over the meetings of council of ministers and plays key role in the appointment and removal of members of council of ministers.

Question 4.
(a) What is a Constitution? Explain the role of conventions with two examples each from the written Constitutions of U.S. and India and two examples from the unwritten Constitution of U.K. [8]
(b) What are rigid and flexible Constitutions ? Discuss any two merits and any two demerits of a flexible Constitution. [6]
Answer:
(b) The distinction between the rigid and flexible constitutions is based on the distinction between the law-making procedure and constitution amendment procedure. If constitution amendment procedure is distinct and difficult in comparison to law-making procedure, the constitution is said to be rigid, as in U.S.A. On the other hand, if a country follows the same procedure for law making and constitution amendment, it is said to be a flexible constitution, as is the case in U.K.

Two Merits of Flexible Constitution : The two merits of flexible constitution are :

  1. The flexible constitution is suitable to emergency conditions as it can be easily changed.
  2. It keeps pace with the changing conditions and adapts to new situations.

Two Demerits of Flexible Constitution :
A flexible constitution is a source of inconsistency in governance, as it enables the government in power to give it desired content and directions.

Flexible constitution is not suitable in a federal polity because in a federation, there is a division of power between the union and states. A rigid constitution is required to maintain this division of power.

Section—B
Answer three questions

Question 5.
(a) Explain the limitations of the theory of Separation of Powers. [8]
(b) Discuss three advantages and three dis-advantages of functional representation. [6]

Question 6.
(a) Discuss the powers of the British House of Commons. [8]
(b) Give six factors responsible for the legislatures becoming mere talking shops in the developing countries. [6]

Question 7.
(a) Compare the powers and functions of the President of U.SA. and the Prime Minister of India. [8]
(b) Distinguish between the permanent and the political executive. [6]
Answer:
(a) Head of the State : The US President is both the Head of the State and Head of the Government, whereas the Indian President is only the head of the State. President of the US is the real executive. Indian President like the British monarch is only a titular head.

Term of office : American President holds the office for four years and he can seek re-election only once, while the Indian President holds the office for five years and is eligible for re-election any number of times. Responsibility to legislature : The US President is not part of any legislature and is not responsible to the legislatures. In India, the Parliament includes the President and the two Houses-Lok Sabha and Rajya Sabha.

Removal process : Both the American and Indian Presidents can only be removed from the office through impeachment. In India either House can initiate impeachment proceedings against the President and with the concurrence of the other House can impeach the President. In US the power to impeach solely lies with the Senate (Upper House).

Emergency powers : Indian President wields huge powers during emergency situations. He can proclaim both external and internal emergencies. Indian President can declare emergency in a state, dissolve state legislature and dismiss the state council of ministers. During financial emergency, he has the power to reduce the salaries of all the government officials including that of the Supreme Court and High Court judges. He has a formidable list of executive, financial and legislative powers which he can exercise on the advice of the cabinet during emergency.

Independence of office : The Indian President needs to act as per the advice of the Council of Ministers (CoM) while discharging his duties. The CoM takes all the decisions pertaining to the affairs of the country and executes them under the President’s name.

Int he US, the members of the cabinet are not members of the Congress and are not responsible to it. The cabinet members are appointed by the President and they are liable to be dismissed by the President. The American President has the power to deter-mine the policy of his government independently without external interference.

Veto power : Indian President has comparatively lesser veto power than the President of US. The President of US can veto a bill passed by the Congress. He needs to sign the bill if it is once again passed by two-thirds majority of both the Houses.

The Indian President on the other hand, can send the bill for reconsideration only once. If the bill is passed again even by a simple majority in the Parliament he is obliged to sign the bill.

Pocket veto : US President can exercise his pocket veto power by not signing the bill for 10 days if he knows the session of the Congress will end within 10 days. In such cases the bill dies.

Indian President can keep the bill for indefinite period as there is no constitutionally prescribed time limit to give his assent. The President can use his pocket veto if the fall of the government appears imminent. However, he has to act as per the advice of the new government and cannot take his own decision if the current government falls. Pocket veto was first exercised by the President Giani Zail Singh with respect to the Indian Postal Bill which was passed by the Rajiv Gandhi government. Subsequently, the new government withdrew the bill in 1980.

State bills : Certain state bills need the previous consent of the President and he possess absolute veto power with respect to some types of state bills. The US President does not have such powers.

Dissolving the legislature : The Indian President can dissolve the Parliament while the US President does not have such powers.

(b) Distinction between the permanent and political executive : The executive branch in a modem state has two branches. the political executive and the permanent executive. The political executive consists of ministers and permanent executive consists of civil servants. Both together carry out the functions of executive branch of government. Following are the main distinctions between the political executive and permanent executive.

Political and non-political character : The political executive consists of ministers, which belong to a political party and are committed to the policies and ideology of their political party. On the other hand, the members of permanent executive the civil servants are politically neutral. Political neutrality is the basic tenet of civil servants.

Policy formulation is done by political executive : The final authority to formulate policies lies with the political executive. The civil servants provide information as well as advice and assist political executive in policy formulation.

Political executive is responsible to people : Political executive is responsible to people through their representative bodies. On the other hand civil servants are not directly responsible to people. In fact, they are responsible to political executive.

Political executive consists of Amateurs, Civil servants are experts : The members of political executive are not trained people. They are members of political parties. On the other hand, civil servants are specially trained people to perform their administrative duties. Short tenure vs. long and fixed tenure : The members of political executive have a short tenure of four to five years. On the other hand, members of permanent executive have long tenure. They pursue their profession on permanent basis.

Question 8.
(a) Discuss the conditions essential to ensure the independence of judiciary. [8]
(b) Explain the original, appellate and advisory functions of the Supreme Court of India. [6]
Answer:
(b) Supreme Court of India : India has adopted integrated structure of judiciary in which Supreme Court is placed at the apex, High Courts at the middle and district courts at the bottom. Indian Supreme Court has following types of jurisdiction :

Original jurisdiction : The original jurisdiction of the court is mainly concerned with its federal functions. Under original jurisdiction the disputes between the center and states or between the states are first brought before the court, which has the final authority to pass judgement on them. Also, the cases involving the violation of fundamental rights are also placed under the original jurisdiction of Supreme Court.

Appellate jurisdiction : Under appellate jurisdiction the Supreme Court entertains three kinds of appeals against the decisions of the High Courts.
First – constitutional appeals, which involve the substantial question of law which requires the interpretation of constitution.

Second – civil appeals, which are related to cases involving substantial question of law of public importance.

Third – is the criminal appeals which are brought before the Supreme Court under the following circumstances,

  • If the District Court has acquitted an accused and High Court has reversed the order and has awarded death sentence,
  • If the High Court has remanded a case from the lower court and has awarded death sentence to the accused.

Advisory jurisdiction : The President of India may ask the Supreme Court to tender advice on any legal question of public importance. The court may or may not tender such advice. On the other hand, if such advice is tendered the President is not bound to follow such advice.

Question 9.
(a) Discuss measures that should be taken to meet the challenge of political violence in India. [8]
(b) Give any six factors responsible for regional imbalances in India. [6]
Answer:
(a) The measures to meet the challenges of political violence in India : Political violence as an organized activity takes different forms in India, like caste violence, mass movement, election violence, terrorism, separatist violence or communal violence. The following measures are suggested to check the political violence in India:
1. Developing a psychology of peace and non-violence through the formal and non- formal means of political socialization.
2. Poverty, non-employment, communalism and ignorance should be eliminated from the society as these evils are sources of political violence in India.
3. There should be a code of conduct for political parties to follow peaceful and constitutional methods.
4. People should be mobilized by civil society organizations against all forms of political violence.
5. There should be effective check on political corruption as it is the root cause of political violence.
6. Peaceful means of conflict resolution must be made comprehensive, popular and efficient.
7. The law and order machinery particularly the police administration must be made transparent, efficient, responsible and accountable.
8. Vocationalisation of education, and right to work should be implemented as it would lead to gainful employment of youth.

ISC Class 12 Political Science Previous Year Question Papers

ISC Political Science Question Paper 2013 Solved for Class 12

ISC Political Science Previous Year Question Paper 2013 Solved for Class 12

Maximum Marks: 80
Time allowed: Three hours

  • Candidates are allowed additional 15 minutes for only reading the paper. They must NOT start writing during this time.
  • Answer Question 1 (Compulsory) from Part I and five questions from Part II, choosing two questions from Section A, two questions from Section B and one question from either Section A or Section B.
  • The intended marks for questions or parts of questions are given in brackets [ ].

Part—I
(Compulsory)

Question 1.
Answer briefly each of the questions (i) to (xv): [15 x 2]
(i) Explain in brief the classification of states as suggested by Pindar and Thucydides.
(ii) Mention the difference between a totalitarian state and an authoritarian state.
(iii) How is liberal democracy a representative government ?
(iv) Mention four essential features of a federation.
(v) Why is it necessary to have a written constitution in modern times ?
(vi) Define the theory of Separation of Powers.
(vii) Why is bi-party system considered best for parliamentary form of government ?
(viii) State two duties of an elected representative.
(ix) Explain the meaning of Positive Dimension of Sovereignty of British Parliament.
(x) Mention any two privileges of the members of House of Lords.
(xi) What is the role of civil servants in law-making
(xii) State the judicial powers of the President of India.
(xiii) What is Equity Legislation ?
(xix) Explain the meaning of Rule of Law.
(xv) Mention the different steps taken by the constitution of India to combat untouchability.
Answer:
(ii) Totalitarianism is the type of government or state that work to hold all power and authority to control each and every aspect of life, e.g., economial, political, social, etc. Its purpose is to grant all power and authority to a single person, the dictator. So that no other individual can question his authority. For the totalitarian state, the one in power has a Christine the people. Examples of individuals who have ruled using totalitarianism are Benito Mussolini in Italy, Adolf Hitler in Germany.

Authoritarian State, on the other hand, have social and economic institutions that are not under the government’s control. Authoritarians are more focused on the status quo and are driven by control. They impose their rule through fear. Examples of famous authoritarians are Saddam Hussein of Iraq, Ferdinand Marcos of Philippines.

(iii) In a liberal democracy, government is the representative of the people. It is elected by the people without any discrimination, except a minimum age qualification criterion. People can change the government through elections. Government represents the people and has to continuously prove its representativeness. So, in a liberal democracy, government is called a representative government.

(iv) (a) There are two or more levels of govemment.
(b) Although the different tiers of government govern the same citizens, but each tier has its own areas of jurisdiction in relation to matters of legislation, taxation and administration.
(c) The area of jurisdiction of the each tires of government is clearly specified in the constitution.
(d) The fundamental provisions of the constitution cannot be unilaterally changed by one tier of government. Such changes require the consent tires of government.

(v) Written constitution means a constitution written in the form of a book or a series of documents combined in the form of a book. A written constitution is an enacted constitution. It provides a definite design to government institutions, their organizations.

powers, functions and inter-relationships. It embodies the constitutional law of the state. The government is bound by its provisions and has to work strictly with accuracy towards its provisions.

(vii) Bi-party system is considered best for parliamentary form of government because :

  • Bi-party system is the essence of a successful parliamentary democracy. It ensures a strong and stable government.
  • Bi-party system offers a clear programme of the two parties before the voters who know certainly for which policies they are voting.

(viii) Two duties of an elected representative are as follows:

  • They act as a communication link between the people of his constituency and the government.
  • They work for ensuring the fulfillment of promises made by their party in the Election Manifesto during elections.

(xi) Civil servants play an important but indirect role in law making. They draft the bills which the ministers submit to the legislature for law making. The ministers provide all the information asked for by the legislature and the legislative committees by taking the help of the civil servants.

(xii) Judicial Powers of the President of India are:

  • The President has the powers to grant pardons, reprieves, respites or remissions of punishment or to suspend, remit or commute the sentence of any person convicted for an offence. This is particularly in all cases involving punishment with death [Article 72(1)].
  • He appoints Judges of the High Courts and the Supreme Court and other Administrative Tribunals.

(xv) Untouchability is constitutionally prohibited and now this evil practice is crime punishable by law. The government of India has recognized and protected them as ‘Scheduled Castes’. And a system of reservation of seats in the legislatures, admissions in educational institutions, government and private sector for people belonging to scheduled caste has been made for their upliftment.

Part-II
Section—A
Answer two questions

Question 2.
(a) Explain the classification of states and governments as given by Stephen Leacock. [8]
(b) Discuss any six essential features of a Presidential form of government. [6]
Answer:
(b) The six essential features of a presidential form of government are as given below :
Head of State Real Executive : In the presidential system there is only one chief of the executive, and that is real. The chief of state is not merely the titular executive but he is the real executive and actually exercises the powers which the constitution and law confer on him.

Real Executive Elected by People : The executive is not hereditary or nominated. He is elected by the people. The President of the United States is elected indirectly by the people through an electoral college.

Fixed Tenure : The executive has a fixed tenure. For example, the President of the USA is elected for a term of four years. He cannot be removed from office before the expiration of his term.

Executive Not Member of Legislature : The executive in the presidential system is not a member of the legislature. The President of the USA is neither a member of any of the chambers of congress, nor is he present in the legislature to introduce and pilot legislative measures. Here the principle of separation of powers is followed.

Executive Not Responsible to Legislature : In the presidential system, the executive is not responsible to the legislature. His tenure does not depend on the wishes of the legislature. His term of office, is fixed and he cannot be removed before its expiration. The executive is independent of the legislature. The legislature is not empowered to control and direct the executive.

Legislature cannot be Dissolved before Expiry of its Term : Dissolution of legislature is not possible in presidential system. For example, in USA the President is not empowered to dissolve any of the chambers of congress before the expiration of its term.

Question 3.
(a) Discuss the various conditions which are essential for the success of a federation. [8]
(b) Give any six problems faced by federal governments. [6]
Answer:
(a) Various conditions are essential for the success of a federation. Some important conditions are discussed below:

Desire for union : The first essential condition for the formation and successful working of a federation is strong desire for unity among the people of the federating units.

Preservation of individuality : The federating units while desiring for union should also be conscious of their individualities and be . committed to preserve these even after becoming part of the federation.

Political maturity and education : Both play an important role behind a successful federation. The system of federation demands an understanding on the part of the people regarding the respective roles of the Central and State governments in the operation of the federal system.

Geographical Contiguity : Geographic contiguity greatly helps the people of a federation to work as a united nation. Geographical distance definitely played a big role in the breaking up of the Pakistani federal system into two sovereign states Pakistan and Bangladesh.

Community of Race, Language, Culture and Religion : A federation characterized by community of race, language, culture and religion alone can work in a strong and healthy manner. These elements can create a sentiment of unity, hold together the people and bind them into one nation.

Approximate Equality of units : The working of a federation also depends upon the size and resources of the federating units. The ideal condition is that as far as possible, there should be approximate equality among them. Wide differences in size, population and resources can prevent the growth of the feeling of equal participation as equal partners of the federation.

Similarity of Social Institution : The federation must be characterized by similar social institutions, only then it can work in harmony. Dissimilar social institutions can hinder the desire for union. These can create great difficulties in the way of working of a federal system.

Economic Development : The factor that influences the federating units to form a federal union is the expectation that the union will be a source of strength and economic development for them. The pooling and coordinating of their resources will result into big economic prosperity and development and is essential for the successful working of a federation.

(b) On the basis of division of powers between the central government and regional or state governments, the governments are classified as unitary and federal. A federal government is one where the powers are divided between the union government and states by the constitution itself. According to Garner, “Federal government can be defined as a system of central and local governments combined under a common sovereignty, both the central and local governments being supreme within definite spheres, marked out for them by the general constitution or by the act of parliament which creates them.”
The federal form of government has been adopted in most of the big countries like the USA, Australia, Canada, India etc. The problems faced by federal governments today are examined under the following points.

A federation is a complex system of government which involves a detailed division of powers and dual administration and justice. All federations face a major problem to ensure a clear cut and settled division of power between the union and the federating units.

Due to division of powers and dual administration, the disputes between the centre and states and between the states are common. There is always a problem of satisfactory settlement of these disputes.

The federation involves dual system of administration and justice for a single political system. This creates problem of co-ordination among states as well as central and state governments. India always faces a problem of co-ordination between the central agencies and state machinery in the field of law and order.

A federal government faces the problem of correcting regional imbalances.

In federal government, the problems of national unity and integrity, delay in decision making and weak central government are common due to the very nature of federal government.

The problem arising from the need to make quick decisions and to secure a full mobilization of resources particularly during times of an emergency.

Section—B
Answer three questions

Question 4.
(a) What is the theory of checks and balances ? Explain how it has been implemented in the US Constitution. [8]
(b) Explain any six merits of a multi-party system. [6]
Answer:
(b) The six merits of a multi-party system are as follows :

  1. The multi-party system ensures the representation of all shades of public opinion.
  2. Under the multi-party system, the authoritarian tendencies in political parties are checked as there is greater competition among them for popular votes.
  3. Under multi-party system, the legislature enjoys authority and thus cabinet dictatorship is not possible.
  4. The multi-party system does not lead to despotism as the coalition government is afraid of legislature.
  5. In the multiple party system the voter has a wider choice to caste his vote.
  6. In a multi-party system there is greater individual freedom and all shades of opinion can express themselves through various groups and parties.

Question 5.
(a) What are the different functions performed by the Executive in modern times? [8]
(b) Explain the three important rights exclusively exercised by the British Monarch. [6]
Answer:
(a) In modern times, Executives perform many different types of functions. The following are the major functions of the executive :
Enforcement of Laws : The primary function of executive is to enforce laws as formulated by the legislature and to maintain law and order in the state. .

Appointment-making Functions : All major appointments are made by the chief executive. For example, the President of India appoints the Chief Justice and other Judges of the Supreme Court and High Courts, Ambassadors, Advocate General of India, Chief of the Armed Forces, Governors of the States etc.

Treaty-making Functions : It is the responsibility of the executive to decide as to which treaties are to be signed and with which other countries. It is again the responsibility of the executive to secure legislative ratification of the treaties negotiated and signed by it.

Defense, War and Peace Functions : The Chief executive of the state is also the Supreme Commander of the armed forces of the State. A minister always heads the defense department. In this way, the executive symbolizes the supremacy of the civil over the military. In most of the states, the executive has the right to declare war upon any enemy of the state.

Foreign Policy-making and the Conduct of Foreign Relations : The executive first formulates the foreign policy of the nation and then implements it for securing the desired goals of national interest. The executive also appoints the ambassadors of the state to other states and conducts foreign relations. The department of the foreign affairs is headed by the foreign minister and he is responsible for the working of his department.

Policy-making : Modern welfare state has to formulate policies, prepare short-term and long-term plans and implement these policies and plans. These two functions can be legitimately described as the most important functions of the executive, through which the state carries out its objective of promoting the welfare all around and development of its people.

Function relating to Law-making : In a Parliamentary system, the executive plays a leading role in law-making by legislature. Financial Functions : The executive has the responsibility to prepare the budget. It proposes to levy new taxes or changes in tax structure and administration. It collects and spends the money as sanctioned by the legislature.

Besides the above eight functions, there are several other functions like developmental functions, planning, securing socio-economic and cultural development of the people of the state which are also performed by a modern executive.

Question 8.
(a) Discuss the powers of the Supreme Court in the USA. [8]
(b) What are the features of Judicial Review in India ? [6]
Answer:
(a) In the U.S. governmental system, the Supreme Court potentially exercises the highest power.
Some important powers of U.S. Supreme Court are given below :

As the Interpreter of the Constitution : The Supreme Court acts as the final interpreter of the constitution. Its interpretations of the constitutional provisions are “considered inherently superior and final”.
The Supreme Court determines as to whether a law is in conformity with the constitution or not. As the final interpreter of the constitution, the Supreme Court possesses the power to strike down any portion of law from the statute book if it is to be found in conflict with the constitution. It is thus the power of the Supreme Court to judge the constitutionality of any law or executive action. The Supreme Court has built up the doctrine of ‘Judicial Supremacy’.

As Guardian of the Constitution : The Supreme Court is the protector and guardian of the constitution of the United States and also the protector and guarantor of the rights of the American citizens. The Supreme Court has the power to declare any law Ultra vires if, in its opinion, the law is not in conformity with the constitution. As a guardian of the constitution, it keeps the powers of various organs and functionaries of the government within their respective spheres of activities.

As an agency for the Development of the Constitution : The Supreme Court has an important role towards the development of the constitution. By the power of Judicial Review, The Supreme Court always interprets the constitution to meet the needs of the time. This power also enables the Supreme Court to ensure that the acts of legislature and executive orders do not violate the provisions of the constitution. The transformation of USA as the top industrialized and technologically advanced country in the comity of modern states has been possible due to the interpretation and development of the constitution as a harbinger of progress and the credit for it goes to none other than the Supreme Court. Its interpretation has helped the federal government to become more powerful and play a leadership role in the developmental process.

As the Guardian of Fundamental Rights : The Supreme Court has always acted as the custodian and protector of Fundamental Rights and Freedoms of the American people. The Supreme Court has always maintained the doctrine of ‘clear and present’ danger. It means, ‘any law restricting the right of freedom of speech, press, religion and assembly would be declared invalid unless it is shown that limitation thereon is justified because of ‘clear and present danger’ to the public security. The concept of inequality and segregation in the field of education was declared illegal by the Supreme Court in 1954. The Supreme Court has to its credit many decisions which uphold the concept of liberty and rights.

Power of Supreme Court in Law-making : The Supreme Court has the power to make a new law or make any changes over any existing law. This power has made the Supreme Court ‘not merely a court of law but also a law-making body’ indeed a third house of legislature. It has also the power to reject any law. “The Supreme Court has no legislative initiative but acting negatively it sets the framework within which the legislature must work”. This has made the Supreme Court a super legislature.

(b) The following are the salient features of the system of Judicial Review in India :
1. Both the Supreme Court and High Courts exercise the power of Judicial Review. But the final power to determine the constitutional validity of any law is in the hands of the Supreme Court of India.
2. Judicial Review can be conducted in respect of all Central and State laws, the orders and ordinances of the executive and constitutional amendments.
3. Judicial Review cannot be conducted in respect of the Acts incorporated in the 9th Schedule of the Constitution.
4. Judicial Review applies only to the questions of law. It cannot be exercised in respect of political issues.
5. Judicial Review is not automatic. The Supreme Court can conduct judicial review over a law only when it is challenged during the course of litigation. In other words, it becomes operative only when laws are specifically challenged before it or when
during the course of litigation in a case, the question of constitutional validity of any law is raised before it.
6. While declaring a law to be void, the Supreme Court has to cite the provisions/ articles of the constitution which violate the law. It has to demonstrate the invalidity of the law struck down by it.

Question 9.
(a) Give any eight remedial steps for meeting the evils of communalism. [8]
(b) Suggest any six measures to eradicate economic inequality. [6]
Answer:
(a) Communalism is a major social and political problem of India which threatens the fabric of Indian democracy.
The eight remedial steps for meeting the evils of communalism are as follows :

Communalism breeds in the minds of people. Hence, the values and orientations of people have to be changed through well designed education system. People have to inculcate the values of brotherhood and social harmony.

Media and other forms of modern communication should be used to generate awareness among people about the dangers of communalism and importance of communal harmony and social equality of people of all religions.

Poverty and economic backwardness are the breeding grounds for communalism. Hence, the government should undertake vigorous social and economic welfare programmes to ensure social and economic progress of all.

Legal provisions against communal practices should be strengthened. The communal organisations and associations of all forms should be banned. The political leaders mobilizing support on religious lines should be punished.

Special anti-riots police force having people from all communities should be organised on national basis. Such forces can be more effective in the situation of communal riots.

The Muslims in India lack a viable middle class status due to lack of education and economic development. By education and economic progress, an enlightened middle class should be strengthened among minorities.

Concrete steps should be taken to prevent the use of religious places for preaching communalism.

Foreign intervention should be carefully watched and presented from encouraging communalism in India.

ISC Class 12 Political Science Previous Year Question Papers

ISC English Language Question Paper 2013 Solved for Class 12

ISC English Language Previous Year Question Paper 2013 Solved for Class 12

Question 1.
Write a composition (in approximately 450-500 words) on any one of the following subjects: [30]
(You are reminded that you will be rewarded for orderly and coherent presentation of material, use of appropriate style and general accuracy of spelling, punctuation and grammar.)
(a) ‘One should judge a man not by what he has, but by what he is. ’ Express your views on this statement.
(b) Write a description of the most interesting person you have ever met.
(c) It is right for a person to interfere in the affairs of another. ’ Write for or against this statement.
(d) Memories.
(e) Relate how travel has been of educational value to you.
(f) Write an original story with the following ending :
“…………………… It was exactly as he had said it would be.”
Answers:
(a) Make use of the following hints and develop them into a complete essay.

  • man may be rich
  • but dishonest and greedy
  • a man may be poor and humble
  • very helpful and useful
  • a great use to society
  • enjoys material comforts
  • of no use to society
  • attaches importance to values in life
  • a selfless man

(b) Make use of the following hints and develop them into a complete essay.

  • Dr. A.P.J Abdul Kalam – a Missile Man
  • faced problems and hardships of life
  • rose to the .highest position of the President of India
  • his achievements as a scientist
  • honoured with Padma Bhushan in 1981
  • successful testing of Trishul (1985), Prithvi (1988) and Agni (1989)
  • his role in the Pokhran atomic bomb blast (1998)
  • popularly known as the Missile Man of India
  • author of a number of books
  • friendly with the children

(c) Make use of the following hints and develop them into an argument against the statement:
It is right for a person to interfere in the affairs of another.

  • no good to interfere in the affairs of another
  • bad manners
  • causes disturbance
  • zoss of concentration
  • output poor
  • liberty and freedom in doing something is undermined
  • hampers prosperity
  • conclusion

(d) Make use of the following hints and develop them into a complete essay.

  • good memories
  • bad memories
  • memories of childhood
  • imaginative process
  • mind becomes active
  • a good pastime
  • sharing memories with others

(e) Make use of the following hints and develop them into a complete essay.

  • classroom teaching and educational value of travelling
  • theoretical knowlege and practical knowledge
  • the importance of educational tours
  • a close study of natural surrounding and things like meadows, mountains, rivers, gardens and valleys.
  • architectural beauty of historical monuments
  • a close study of history
  • travelling – a medium of education
  • addition to knowledge

(f) Make use of the following hints and develop them into a complete story.

  • a man hired a room in a street in the city
  • patches of discolouration on the walls; one of these looked exactly like a human face
  • the man came to think of it as real
  • the patch resembling the face of a person took hold of the man
  • it grew more and more real and remarkable
  • the man decided to find this man in the city street and public places
  • the search became a mania with him
  • finally he could locate the man in a busy place
  • a surprising discovery
  • “it was exactly as he had said it would be.”

Question 2.
Write a review of a book you have recently read (in approximately 300 words) for your school magazine. Make use of the given points. [20]
Name of the book — author — publisher — readership — cover and illustrations—plot — characters — insight into some interesting incidents — message if any — appeal the book holds for you.
Answer:
Name of the book: The Canterville Ghost
Author: Oscar Wilde
Publisher: Maple Press Classics
Readership: a great number of readers
Cover and illustrations: Suggestive and meaningful
Plot: The ghost in the story is real. Weary of his long, unhappy life, the ghost seeks salvation through peaceful death which is denied to him because of his diabolic act of murdering his wife. Virginia, the American girl, prays for him and weeps for him—as desired by the ghost—and Death shows mercy on him at last. The ‘golden girl’ toms out to be a saviour for him. Through her caring and concern, she proves that love is stronger than life and even death.

Characters:
1. The Ghost: The ghost of Sir Simon de Canterville is the central figure in the novel. Unlike conventional ghosts, he is a moody, weary, helpless ghost. Though he has done enough mischief, he fails to do any harm to the stubbornly materialistic American family. He becomes a pathetic figure with repeated failures to frighten the new inhabitants. In spite of his supernatural powers, he is unhappy and yearns for the release of his soul. When Virginia helps him get peaceful death, he gives her precious jewels out of gratitude.

2. Mr Otis : Mr Otis, the American minister, is totally materialistic and ruthless in his attitude. He believes that money can buy anything, even a ghost. He does not have faith in the world of ghosts. So he confronts the ghost boldly and defeats him in his bad intentions. He is a fond father. When Virginia disappears, he does his best to find her out. He does not like British conventions and traditions. He is a thorough Republican.

3. Virginia : Virginia is a young daughter of the American minister. She is an expert rider. She loves the young Duke of Cheshire whom she marries later. She is unlike other members of her family. She remains aloof from her family’s anti- ghost campaign and does not scoff at the ghost. The ghost, too, is soft towards her. She is caring and affectionate. She takes pity on the ghost in his extreme depression. She takes a calculated risk in helping the ghost get peaceful death. By her attitude she proves that love is really stronger than life and even death.

Insight into some interesting incidents:
The ghost of Sir Simon appears before Mr Otis who is awakened by a curious noise at 1 o’clock in the night. The ghost, and old man of terrifying appearance, fails to frighten Mr Otis.

When (he ghost again tries to terrify the family, hc meets the same fate as before. The twins shoot two pellets on him which their peashooters. and Mr Otis covers him with his revolver and asks him to hold up his hands. The ghost shrieks and gives his most horrible laugh, but to no avail.

The ghost gives up the hope of ever frightening the American family. Virginia happens to see the ghost in extreme depression.

The disappearance of Virginia causes a flutter in the house. It is thought that she has been abducted. When she appears finally, she tells her flimily that the ghost is finally dead. The ghost is ceremonially buried.

Message : Oscar Wild conveys the message that love is a potent force. It can achieve anything by overcoming all obstacles. It can conquer even life and death.

Appeal the book holds : The Canterville Ghost is a ghost story in an uncoventional for you way. The ghost has some supernatural powers but he is a weary ghost. He is unable to inflict any real harm. Throughout the novel there is nothing violent and ghoulish. The novel has a popular appeal.

Question 3.
Answer sections (a), (b) and (c).
(a) In each of the following items, sentence A is complete, while sentence B is not Complete sentence B, making it as similar as possible to sentence A. Write sentence B in each case. [10]
Example : (0)
(A) : Though it was a cloudy day, the temperature was 45°C.
(B) : Despite ………………………………….
Answer :
(0) Despite it be ng a cloudy day. the temperature ivas 45°C.

(1)
(A) : They are demolishing the entire block of flats.
(B) : The entire ………………………………….

(2)
(A) ; Anil thought the examination would be held later.
(B) ; Anyway ………………………………….

(3)
(A) : I do not like this food as much as you do.
(B) : You ………………………………….

(4)
(A) : Raju failed in the examination, yet he was not upset.
(B) : In spite of ………………………………….

(5)
(A) : “Don’t lend Ramesh any money;” I said to Rita.
(B) : I advised ………………………………….

(6)
(A) : As soon as the arrangements were made to release extra water from the reservoirs, the rain came in a downpour.
(B) : No sooner ………………………………….

(7)
(A) : He had to sign or be arrested.
(B) : libe ………………………………….

(8)
(A) : Meena complained of not being able to see the performance from the back row.
(B) : Meena complained that ………………………………….

(9)
(A) : Many ambitious people see politics as a stepping stone to personal power.
(B) : Politics ………………………………….

(10)
(A) : Rahul has wasted bis walth and ruined his health by his drinking habit.
(B) : Rahul has not only ………………………………….

(b) Fill in each blank with a suitable word. (Do not write the sentence.) II
(1) Let us talk …………………………………. then tomorrow about their homework.
(2) The Principal gave us a tallc …………………………………. bullying.
(3) Manoj was not at home when his grandfather passed ………………………………….
(4) When we visited Daihousie, we passed …………………………………. the house in which we had lived twenty years ago.
(5) This novel has been written …………………………………. Jeffrey Archer.
(6) In the Middle Ages, books were written …………………………………. a variety of subjects.
(7) He had promised her that he would come to the party but he didn’t turn ………………………………….
(8) I was forced to turn …………………………………. Maya’s invitation as I was going to be out of town the following week.
(9) R.ashi has applied …………………………………. a scholarship in an American university.
(10) Arjun applied …………………………………. the university for higher studies but was denied admission.

(c) Fill in the banks in the passage given below with the appropriate form of the verb given in brackets. Do not write the passage, but write the verbs in the correct order. [5]

Breathing is good, on this we …………………….. (1) (can) agree. That is why sleep apnoea. a condition in which sufferers repeatedly …………………….. (2) (stop) breathing and …………………….. (3) (awake) during the night …………………….. (4) (be) so distressing and dangerous. Doctors …………………….. (5) (Xhave) long known that the majority of people with sleep apnoea also …………………….. (6) (havc) gas and reflux, so they oflen …………………….. (7) (treat) the reflux with heartburn medica tion. To prove the connection between reflux and sleep apnoea, scientists at an army hospital …………………….. (8) (observe) apnoea patients as they …………………….. (9) (sleep). They (10) (find) that reflux could, in fact, be the cause of many night-time awakenings.
Answers:
(a)
(1) The entire block of flats is being demolished.
(2) Anil was of the opinion that the examination would be held later.
(3) Youlikctliisfoodasldo.
(4) In spite of his failure in this examination Raju was not upset.
(5) I advised Rita not to lend Ramesh any money.
(6) No sooner were the arrangements made to release extra water from the reservoirs, the rain came in a downpour.
(7) If he did not sign, he would be arrested.
(8) Meena complained that she was unable to see the performance from the back row.
(9) Politics is a stepping stone to personal power for many people to realise their ambitions.
(10) Rahul has not only wasted his wealth but also ruined his health by his drinking habit.

(b) (1) to (2) about (3) away (4) by (5) by (6) on (7) up (8) down (9) for (10) to

(c) (1) can (2) stop (3) awake (4) is (5) have (6) have (7) treat (8) observed (9) slept (10) found

Question 4.
Read the passage given below and answer the questions (a), (b) and (c) that follow :

Boyhood And Balance

(1) My eldest boy has just turned nine. One day not long after, he was on his way to the pool on his bike when he slammed on his brakes in front of me and did a really neat skid. He looked back and gave me his cheesy smile of satisfaction.

(2) Normally that little skid would have provoked a stem warning not to “wear out your tyres.” I was always taught to take care of my things and keep them in good shape. I’ve come to learn, though, that there is a difference between the natural course of w earing things out and trying to make things last at any price. The difference became acute to me one bright sunny day four years ago.

(3) My then five-year-old boy had spent a normal summer afternoon playing in the park and swimming at the neighborhood pool. At his bath time that night, I noticed he had small red spots all over his body. Some kind of measles, I thought, and called my wife to take a look. Bu pure chance, the next day he had an appointment with his pediatrician.

(4) Several tests (and long hours) later, the doctors figured my son had an illness called ITP (idiopathic thrombocytopenic purpura), which causes your spleen to kill of your body’s own blood-clotting platelets. If it got worse, he could bleed to death internally. Only time would tell whether it would get better on its own. You can imagine the fear that crept into our hearts.

(5) Three days into his stay at the hospital, I went to pick out a present for him. I reached for a yellow toy convertible (he loves cars). My hand hesitated. I knew that it wouldn’t last long with a five-year-old boy. Then I thought to myself: So what if the doors break off and the wheels fall off. If those things happen, it will mean he is alive and well. I bought it in the hope that he would be well enough to play with it. He was thrilled to receive it, and it helped him pass the week he spent at the hospital.

(6) I saw the car the other day, sitting on a shelf in his room. The wheels have fallen off, the doors are broken and all of the chrome has worn off. I see it and I smile to myself. My boy has been perfectly well these past four years, and is full of vitality. His mysterious illness came and went.

(7) And I learned that thi’h^k’ (ire things and can be replaced if necessary. If any one of my three boys happens to break something or wear it out playing with it, instead of chastising him for carelessness, I now prefer to celebrate his boyhood. The empty shell of what once was a nice car, the balding tyre of his new bike, the lost pieces of his Monopoly game, all bear witness to the fact that there lives a healthy, happy boy.

(8) Prudence and preservation have their place, as do experimentation and curiosity. In the end, it is my relationships with my sons and my wife that have real lasting power. I choose to celebrate our lives with the losses.

(9) Besides, I used to skid my tyres the same way – just not in front of my dad.

Elliot Van Egmond – The Globe and Mail (.Adapted)

(a) (i) Given below are four words and phrases. Find the words which have a similar meaning in the passage : [4]
(1) strict
(2) energy and vigour
(3) scolding
(4) strong desire to know

(ii) For each of the words given below, write a sentence of at least ten words using the same word unchanged in form, but with a different meaning from that which it carries in the passage : [4]
(1) shape – (Line 5)
(2) park – (Line 9)
(3) present – (Line 17)
(4) well – (Line 20)

(b) Answer the following questions in your own words as briefly as possible :
(i) Why is the disease ITP so serious?
(ii) What does the yellow car look like at present?
(iii) What did the writer learn from his experience?
(iv) What is most important to the writer now?

(c) Summarize thc given passage in not more than 100 words. Failure to keep within the word limit will he pcnalised. You will he required to:
(i) List your ideas clearly in point form. [6]
(ii) In about 100 words, write your points in the form of a connected passage. [6]
Answer:
(a) (i)
(1) strict : stem
(2) energy and vigour : vitality
(3) scolding : chastising
(4) strong desire to know : curiosity

(ii) (1) shape : The painter has given a beautiful shape to his model.
(2) park : Do not park your vehicle here.
(3) present : She was present in the class.
(4) well : The child has fallen int the well.

(iii) (1) shape The painter has given a beautiful shape to his model.
(2) park Do not park your vehicle here.
(3) present She was present in die class.
(4) well The child has fallen into the well.

(b) (i) The disease ITP is serious because it causes patient’s spleen to kill off his body’s own blood-clotting platelets. If it becomes worse, the patient could bleed to death internally.
(ii) The yellow car looks like a present. It is convertible and it would be well enough for the boy to play within it. Moreover it does not matter if its doors break off and the wheels fall off.
(iii) The writer learnt a lot from his experience. He noticed the car on a shelf in his room. It has no wheels and doors. The writer smiled and concluded that boy’s mysterious illness came and went. The boy was perfectly well.
(iv) The writer concludes that prudence and preservation have their place as do experimentation and curiosity. Relationship within the family is the most important thing. Therefore, the writer chooses to celebrate their lives and live with the losses.

(c) (i)
(a) Writer’s way of thinking and handling things in his boyhood.
(b) his five-year-old boy had developed red spots all over his body; a serious kind of disease called ITP.
(c) The boy admitted in the hospital, spent three days there.
(d) the writer presented a yellow toy car to his son; the boy played with the toy, broke it and became well; his illness had gone.
(e) the writer learnt from his experience. He concluded that prudence and preservation have their place in family relationship.

(ii) The writer compares his childhood with that of his children. He finds that there is a difference between the natural course of wearing things out and trying to make things last at any price. The contrast became clear to him when his five-year-old boy was admitted in the hospital for treatment of ITP (idiopathic thrombocytopenic purpura). The disease was serious but it disappeared mysteriously. The writer experimented with a toy car and it worked very well. The boy became perfectly well.

ISC Class 12 English Language Previous Year Question Papers

ISC Home Science Question Paper 2019 Solved for Class 12

ISC Home Science Previous Year Question Paper 2019 Solved for Class 12

Maximum Marks: 70
Time allowed: Three hours

  • Candidates are allowed additional 15 minutes for only reading the paper. They must NOT start writing during this time.
  • Answer Question 1 from Part I and five questions from Part II,
  • The intended marks for questions or parts of questions are given in brackets [ ].

Part-I (20 Marks)
Answer all questions

Question 1. [10 x 2]
(i) Why has solar cooking become popular as a modern method of cooking ?
(ii) What is meant by pounding ? Give one example of its usage.
(iii) State two changes that occur in the nutritive value of food during germination.
(iv) List any two effects that advertisements have on consumers.
(v) Mention two ways of increasing life expectancy of the elderly.
(vi) State one difference between bactericidal and bacteriostatic methods of food preservation.
(vii) Define binge eating disorder.
(viii) Name any two adulterants commonly found in spices.
(ix) State the basic difference between the wet and dry methods of cleaning clothes.
(x) State any two common safety measures adopted in the home that help to minimise the risk of electrocution.
Answer:
(i) Solar cooking has become popular as a modern method of cooking due to rising crisis of non-renewable sources of fuel and energy. It is cost effective, natural, easily available, no maintenance and eco friendly method of cooking. It has also gained popularity because of growing awareness of environmental issues among the younger generations.

(ii) Pounding is a technique of particle size reduction of foods usually done by forceful blowing in mortar and pestle. This helps releasing of juices and flavours from the food. It may also help in processing steps like dehulling.
Example: Husk removal of seeds and grains like rice; pounding of meat to release juices and flavors.

(iii) The two changes that take place in nutritive value of food during germination are:

  • Germination increases the content of Ascorbic acid (Vitamin C) by 4 to 20 times and that of Riboflavin (Vitamin B2) by 2.5 to 4.5 times.
  • There is also a significant reduction in anti-nutritional factors like phytates and trypsin inhibitors that hinder the digestion of proteins, making proteins easily digestible and available to the body.

(iv) Two effects of advertisements on consumers are:

  • Advertisements make a long lasting impact on consumers’ minds by connecting emotionally and making a memory of the advertisement.
  • It develops a confidence among the consumers for the product, giving it recognition and acceptance in the market.

(v) Two ways to increase life expectancy among elderlies are:

  • Healthcare by regular exercise, monitoring and controlling the prevalent conditions, taking multivitamin and antioxidant supplements and regular health check-ups can be helpful.
  • Building a positive and happy environment around the elderly, increasing social interaction and- acceptance can help protect their social, mental and emotional well-being, leading to longer lifespan.

(vi) One difference between bactericidal and bacteriostatic methods of food preservation is Bactericidal method kills the bacterial cells and prevents any further growth whereas bacteriostatic method prevents the growth (multiplication) of existing bacterial cells.

(vii) “Binge eating disorder” is a serious eating disorder in which a person (usually overweight or obese) frequently craves and consume large amounts of food and lacks resistance and control over eating.

(viii) Two adulterants commonly found in spices are:

  • Cassia in cinnamon.
  • Dry tendrils of maize cob in saffron.
  • Brick powder in chili powder.
  • Argemon seeds in mustard seeds.
  • Lead chromate in turmeric powder.

(ix) Basic difference between wet and dry methods of cleaning clothes are wet cleaning involves use of water and detergents or bleaches whereas dry cleaning involves non- aqueous chemical solvents.

(x) Safety measures that can be taken to make home safe from electrocution are :

  • Keep all circuits grounded.
  • Keep all electric appliances and panels away from wet areas and avoid using them with wet hands.

Part-II (50 Marks)
Answer any five questions.

Question 2.
Cooking is a means of processing food to make it fit for human consumption. In this context, discuss :
(a) The advantages of using pressure cooker to cook food. [5]
(b) Merits and demerits of cooking food. [5]
Answer:
(a) Advantages of cooking food in pressure cooker are:

  1. Most nutrients are retained in pressure . cooking.
  2. Foods get optimally cooked. Example: Bengal gram requires pressure cooking to get completely cooked.
  3. Taste and flavour are retained in pressure cooking.
  4. It saves time and fuel making cooking process easy and quick.
  5. It uses less water than conventional boiling method by using steam under pressure.
  6. Multiple foods can be cooked at a time.
  7. It makes the food sterile and safe by using steam under high pressure.

(b) Merits of cooking food are:

  1. Cooking makes the food easily digestible by converting long chain molecules to short chain molecules.
  2. It destroys the anti-nutritional factors that inhibit availability of nutrients.
  3. It helps to release many nutrients from complexes, making them easily available upon digestion.

Demerits of cooking food are:

  1. Cooking destroys heat sensitive nutrients. Example : Vitamin C.
  2. Over cooking of food may lead to production of carbon products that are harmful to health.
  3. If the water used for cooking is discarded, the water soluble nutrients and flavours are lost.

Question 3.
(a) Briefly explain the importance of fats, when used during cooking of food. [5]
(b) State five precautions to be taken to minimize loss of nutrients during cooking. [5]
Answer:
(a) Importance of fats when used for cooking food are:

  1. Fats are excellent medium of heat transfer.
  2. It ensures that the food is evenly cooked on all sides, inside and out as it penetrates the food particles and coats it well all around.
  3. Fats make vitamin A, D, E and K available to the body as they are fat soluble.
  4. Fats also make non-nutrients like carotenoids and lycopene available to body that have health values.
  5. Fats add to calorific value of the food, making the food more nourishing to the body.

(b) To minimize the loss of nutrients during cooking:

  1. Do not discard the water used for boiling foods like vegetables, chicken, legumes and rice.
  2. Reuse the water used for boiling foods to knead dough or in soups and curries.
  3. Pressure cooking and steaming food retain more nutrients than conventional boiling method.
  4. Microwave cooking also retains most of the nutrients than other methods of cooking.
  5. Avoid overcooking of foods like toasting bread, chapatis for long time to retain the nutrients.
  6. Cut the vegetables and fruits only at the time of cooking or consumption.
  7. Prior cutting/chopping makes the water soluble nutrients evaporate with the moisture of foods, especially vitamin C.
  8. Do not store cooked foods for long periods of time. If required, store in refrigerator in airtight containers.

Question 4.
(a) Discuss any five steps to ensure safety in the kitchen. [5]
(b) Define the term, quality. Give any four reasons for considering food to be of poor quality. [5]
Answer:
(a) Five steps to ensure kitchen safety are :

  1. Keep LPG cylinders away from electric panels and make sure they do not leak.
  2. Keep electric appliances away from wet areas like washing areas.
  3. Keep all circuits and heavy appliances grounded.
  4. Do not touch or use appliances with naked wire. Regular maintenance and immediate repair of gas stove and other electric appliances.
  5. Keep fire extinguishers easily reachable and educate the workers/ family members for their use and other precautions in case of emergency.

(b) ‘Quality’ is optimum standard of food as compared to the legal standards as well as consumer acceptance.
Four reasons to consider a food of low quality are :

  1. Infestation of foods like grains are considered substandard.
  2. Off flavour or odour due to spoilage of food by microorganisms is considered as low quality food.
  3. Uneven size and shape of foods like vegetables can be considered as low quality foods.
  4. Tampered packaging, gnawed and broken foods may also be considered as low quality.
  5. Presence of extraneous materials like hair/pins/stones/thread or others is considered as low food quality.
  6. Adulterated foods are considered low quality foods.

Question 5.
(a) List five advantages of food preservation with suitable examples. [5]
(b) Discuss any five points to be kept in mind in order to retain the nutritive value of green vegetables. [5]
Answer:
(a) Five advantages of food preservation are :

  1. Non-seasonal foods can also be available due to preservation of foods (Ex. pineapple candies).
  2. Preservation of foods like fruits (Ex. mango .pineapple, amla) can add to nutritional value of foods.
  3. It prevents loss of food materials when available in bulk (Ex. banana chips) and makes food available in drought or other reasons of unavailability (Ex. biscuits).
  4. It makes cooking easier, faster and more convenient (Ex. Frozen peas/com)
  5. It adds variety to the dishes. Ex. sauces, pickles, jams.

(b) Five ways to retain the nutritive value of green leafy vegetables are :

  1. Do not chop the leaves before cooking time as they have lot of water soluble nutrients that are lost during storage.
  2. Blanch the leaves and cooking them just to the optimum point prevents losses of heat labile nutrients.
  3. Use the leaves when fresh as they tend to lose nutritional benefits along with storage.
  4. Use variety of leaves to get optimum nutrition.
  5. Cover the cooking pan with a lid to prevent losses with steam.
  6. Use only the required amount of water for cooking.
  7. Reuse the water to make soups or curries as the water contains lot of vitamins and minerals.
  8. Do not serve leafy vegetables with milk and milk products and phytates in milk bind iron

Question 6.
(a) Explain any five non-nutritignal factors to be kept in mind while planning a meal for the family. [5]
(b) Discuss any five factors to be observed while preparing a special diet for a diabetic patient.
Answer:
(a) Five non-nutritional factors to be kept in mind before planning a meal for family are:

Glucosinates present in broccoli,cabbage, radishes and cauliflower prevent the uptake of iodine, causing health complications related to thyroid functions.

Protease inhibitors (Trypsin inhibitors) in soybeans inhibit the actions of trypsin, pepsin and other proteases, hindering complete digestion and absorption of proteins.

Lipase inhibitors found in some algae, soybean and wheat interfere with enzymes, such as human pancreatic lipase, that catalyze the hydrolysis of some lipids.

Phytic acid present in nuts and seeds has a binds to minerals like calcium and iron making them unavailable upon digestion.

Amylase inhibitors found in various beans prevent the action of enzymes that break the glycosidic bonds of starches and other carbohydrates, preventing the release of simple sugars and absorption by the body.

The foods containing inhibitors and their respective nutrients should not be consumed together in a meal.

(b) Five factors to remember while preparing a special diet for a diabetic patient are :

  1. Avoid added sugars and sweet fruits like mango, banana and sapota.
  2. Avoid foods containing trans fats and saturated fats. Include foods varieties rich in polyunsaturated fatty acids.
  3. Replace processed, deep fried foods and bakery products in snacks with fresh cut salads, fruits (selected), nuts and puffed cereals.
  4. Replace refined and polished cereals products with whole cereals.
  5. Include variety of protein rich foods like paneer, eggs and boiled legumes.

Question 7.
(a) Explain any five causes of delinquency among adolescents. [5]
(b) Describe the role of parents in guiding adolescents. [5]
Answer:
(a) Five causes of delinquency among adolescents (juvenile crime) are :

  1. Use of substances such as drugs and alcohol.
  2. Wrong ways of parenting like harsh and abusive ways of punishment.
  3. Incorrect selection of friends and peers, resulting in unwanted, irresistible reasons to commit crime.
  4. Social labelling at young age can lead to personality changes and biases.
  5. Economic and social complications leading to forced crime commitment.

(b) Role of parents in guiding adolescents are:

  1. Parents should spare sufficient time to listen to and try to understand the adolescent’s fears and concerns.
  2. They should be positive in their approach and communication by giving friendly advice.
  3. They should accept and respect the adolescent.
  4. They need to be sensitive and responsive to siblings of different ages and genders, including adolescents.
  5. They should give directions or suggestions in a positive way, so that the adolescents do not need to decipher information from wrong sources.

Question 8.
(a) Explain the steps involved in the laundering of woollen garments. [5]
(b) Discuss any five points to be kept in mind while storing expensive and delicate clothes. [5]
Answer:
(a) Steps involved in laundering woollen garments are:

  1. Mix the required amounts of wool washing liquid in a bucket of cold water. Soak the woollen garment in this for 15 to 20 minutes.
  2. Gently agitate and softly rub the garment with your hands to remove dirt, if any.
  3. Rinse under cold water to remove soap.
  4. Gently press the water out of the woollen, do not squeeze, wring or drip dry.
  5. Do dry, lay them flat in natural shape on a towel spread on drying rack.

(b) Five points to be kept in mind while storing expensive and delicate garments are :

Make sure there is ample ventilation and sunlight in the room and the wardrobe where expensive and delicate garments are stored. Do not store them in a plastic bag, but use muslin or tissue bags to provide enough circulation and prevent odors and fungal growth.

Knitted, beaded and lacy garments require tissue paper between folds to prevent tangles with fabrics.

Plastic or wooden hangers retain the shape whereas wire hangers de-shapes the garment and it may stain the garment with rust in long run and moist air.

Do not store worn and clean clothes in a single shelf or drawer as it may attract pests.

Cedar oil or lavender oil sachets can be stored in the wardrobe as alternative to mothballs.

Question 9.
Write short notes on any two of the following :
(a) Role of Agmark as a standardised mark. [5]
(b) Stages of cognitive development during adolescence. [5]
(c) Disposal of household wastes to maintain sanitation. [5]
Answer:
(a) Role of Agmark as a standardized mark :
Agmark was termed by “Ag” for agriculture and “mark” for certification. It gives certification to quality agriculture products through the Government of India by its agency Directorate of Marketing and Inspection (DMI).

Though it is voluntary certification, it is designed to benefit the local farmers throughout India to protect them from exploitation and raising their standards with certification.

Currently AGMARK covers guidelines for 222 different commodities spanning a variety of pulses, cereals, essential oils, vegetable oils, fruits and vegetables, as well as semi- processed foods.

It has testing laboratories across India where analysis of food samples are done for chemical and microbiological parameters, pesticide residues, adulterants.

It evaluates various food product ranges like spices, fats, oils, cereals and their products, legumes and their products, fruits, vegetables, honey, and animal source food products like meat.

(b) There are three stages of cognitive development during adolescence:
First stage is the Early adolescence. In this stage, thinking becomes more complex with reasoning and is focused on personal decision making in all aspects, like home, play and school.

Second stage is Middle adolescence in which complex thinking process as well as experience makes the adolescent more concerned about abstract issues like the philosophical and futuristic ones.

The third stage is the Late adolescence where complex thinking processes are focused on self centered concepts and decision making again, but with increased thoughts about global concepts like justice, history, politics, and patriotism.

(c) Disposal of House-waste to maintain sanitation are:
Reduce, Reuse and Recycle is the key to all kinds of waste disposal and is easiest to follow in households.

  1. Reduce the use of plastic and paper products. Use cloth bags rather.
  2. Reduce buying toys and materials that are made up of plastics or are packaged in plastic wrappers or containers.
  3. Reuse those products that have to be discarded but can be reused in some way.
  4. Reusing clothes can be done in many ways like making an art piece or reusing as a duster.
  5. Recycling of waste materials can be done at home in many different ways or it can be given away to scrap collectors.
  6. Unwanted items can be donated to the needy.
  7. Vermi-composting can be done by using the wet waste produced at home instead of discarding it.

ISC Class 12 Home Science Previous Year Question Papers

ISC Sociology Question Paper 2014 Solved for Class 12

ISC Sociology Previous Year Question Paper 2014 Solved for Class 12

Maximum Marks: 70
Time allowed: Three hours

  • Candidates are allowed additional 15 minutes for only reading the paper. They must NOT start writing during this time.
  • Answer Question 1 from Part I and five questions from Part II,
  • The intended marks for questions or parts of questions are given in brackets [ ].

Part—I (20 Marks)
Answer all questions.

Question 1.
Answer briefly each of the questions : [10×2]
(i) Mention any two features of Tribal Law.
(ii) What is meant by phratry?
(iii) Define probationary marriage.
(iv) Mention any two features of nuclear family.
(v) Define status f
(vi) Explain Kulinism.
(vii) Mention two reasons why law is obeyed in primitive societies.
(viii) Mention any two determinants of status according to Malinowski.
(ix) What is Fetishism?
(x) Explain the term ‘modernization’.
Answer:
(iv) Nuclear family is a group that comprises a man, his wife and their children, both biological or adopted. It is basically an independent family. It is generally said that the members of a nuclear family always live together in the same household. The nuclear family is divided into-a) complete nuclear family and incomplete nuclear family. Two important features of a nuclear family are :
Nuclear family is smaller in size : It is comprised of immediate family members such as husband and wife, and their child/children. Sometimes the family may have a single parent and his/her child/ children.

Nuclear family is grounded on a very strong bond of emotion and sentiment : It is built upon sentiments of love, devotion, affection, sympathy, cooperation and friendship owing to the fact that there are just a few immediate members in the family,

(viii) Malinowski and Lowie have pointed out four different things as determinants for the status of women in tribal societies :

Actual treatment : What What sort of treatment is given to women is a key factor in determining their status. Are they sufficiently cared for ? Are they given the opportunity to realize their potentials ? Do they share feelings and likings with the male counterparts ? These are some of the questions that need to be considered in determining their actual status.

Legal status : Legal status refers to acceptable roles’ that are considered fit for women. For example, the Toda women are not allowed to take part in dairy activities though dairy is the principal source of economy for them. At the same time they are pampered and cared for by the male members. Hence, though their legal status is low, their social status can hardly be considered to be low.

Opportunity for social participation : It refers to the scope of opportunity that women are given to participate in household activities and religious rituals.

Character and extent of work : It refers to the types of work women are subjected to in the family and society, and to what extent such works are valued.
They felt that these are to be considered while examining the status of the women in tribal society.

(x) Modernization is a process whereby people cast aside the traditional outlook on reality and accept changes characterized by freedom of choice, self-affirmation, sense of self-confidence, high degree of social mobility, realization of innate potential to think and act independently, etc..,
Modernization implies freedom of choice and expression, free access to new ‘experiences, high aspirations, self-assertion and confidence. Man does not depend on destiny and ascribed status. There is a high degree of social mobility in modernized society. People want to achieve something with their ability to work hard, using their innate intellectual and other capabilities. Industrial centers provide such free choices to all.

Part—II (50 Marks)
Answer any five questions.

Question 2.
(a) What are kinship terms? Discuss the types of kinship terms. [5]
(b) Discuss the kinship usage of avoidance. [5]
Answer:
(a) In every society people make use of special terms/names to differentiate between the different kins. E.g. terms like father, mother, uncle, aunt, brother, sister, etc. These terms in social anthropology are termed as kinship terminology. In other words, these kinship terminologies classify our relatives and their relationships. There are two types of kinship terminology systems that are used widely:

  • Descriptive
  • Classificatory.

Descriptive terminology : It is used to specify or describe the person’s direct blood relationship with the person being addressed e.g. when I say my father, my mother, then I am talking about my biological parents and no one. else. Therefore, the two terms father and mother are descriptive terminologies. Some of its characteristic features are :
1. These terms refer to blood relationship with the particular person.
2. These terms keep the collateral lines clear and distinct from each other.

Classificatory terminology : These are terms which are used to relate to more than one relations in a kinship group. Terms like aunt, uncle or brother, sister, etc. refer to persons who stand in different relationships e.g. when I say ‘uncle’ it may mean my father’s brother, mother’s brother or my father’s friend, or even my aunt’s husband. The term ‘uncle’ is, therefore, classificatory term. Some of the characteristic features are :

  1. Same term is used to designate more than one person of the kin relations.
  2. It is a method of dividing the kins according to the social relations and not always blood relations.

(b) Avoidance is designed to avoid intimacy and closeness among certain members in a family or kinship groups. Certain relatives need to maintain distance and avoid each other so that they do not enter into incestuous relationship (taboo) or conflicts with one another. These avoidance rules are generally practiced between female-in-laws and male in-laws or son-in-laws and daughter-in-laws.

Reasons for the practice of avoidance behavior:
There are various reasons cited by the social anthropologists for the practice of avoidance behavior. According to Tylor, in the matriarchal families of earlier times, son-inlaw used to stay in his wife’s family and as a stranger to that house, he avoided his mother- in-law. Frazer opined that among some primitive tribes, avoidance was common among brothers and sisters in order to prevent any sexual intimacy between them. Low felt that avoidance rules were attached to the daughter-in-law who came from another background and had to adjust to new social, cultural and moral attitudes.

Consequently, she remained alien to new family. So, to avoid any probable sexual harassment from the male members of the new family, she generally avoided any direct intimacy. Radcliffe Brown has provided the most contemporary explanation, when he said that people coming from another family may always face difficulty in coping with the new family and the chances of conflict are high, so to avoid such possibility, society has offered the rule of avoidance. Also to help the daughter-in-law avoid facing role conflict between her role of a daughter in law and that of a wife, it was advised that she maintained avoidance and distance from her parents-in-law.

Thus, it seems that initially it was introduced so that sexual relations could be restricted to formal relationships. At the same time, the intention to prevent any possibility of hostile relationship between two kin group members was also one of the reasons for the introduction of avoidance behavior.

Question 3.
(a) Examine the role of family in human life. [5]
(b) Discuss the causes for the breaking of Joint families in India. [5]
Answer:
(a) Family is the most primary group in the society. It is the simplest and the most elementary form of the society. It is the first and the most immediate social environment to which a child is exposed. It is in the family that the child develops physically, psycho¬logically and socially.

The important role of family in human life must be examined from the point of view of the varied functions that are performed by family in human life and society. Maclver classifies the functions of family into two categories :

  1. Essential/Primary and
  2. Secondary

(i) Essential/Primary functions:

Stable satisfaction of sex needs : Sex desire is powerful in human beings. Family regulates the sexual behaviour of a man and a woman through marriage. Thus, it provides for the satisfaction of the sex need.

Reproduction and procreation : The process of reproduction is institutionalised in the family. Hence, it assumes regularity and a stability that all societies recognize as desirable. Family makes the act of reproduction as a legitimate act.

Upbringing of children : Family gives the individual his life and a chance to survive. All individuals owe their life to the family. No other institution can, as effectively bring up children, as family does.

Emotional function : Family, as an institution, provides for the mental or emotional satisfaction and security of its individual members. It is the family which provides the most intimate and dearest relationship to all its members.

(ii) Secondary functions:
Economic function : Family fulfills the economic needs of its members. This has been the traditional function of a family. The income generated by the members is used for the development and progress of the family.

Social-cultural function : Family guarantees not only biological continuity of the human race but also the cultural continuity of the society of which it is a part. It transmits ideas and ideologies, folkways, customs and traditions, beliefs and values from one generation to the next. It prepares the children for participation in larger world and acquaints them with a larger culture.

Educational function : Another important role of the family is to provide education and enable its members to socialize. Family provides the basis for a child’s formative learning. The culture and traditions of society, the dos and don’ts of society and its values are taught by parents at home. Family is also responsible for the shaping up of the personality of the individual members.

(b) India, as a country has been witnessing a steady decline in the joint family structure, not just in the cities, but also in rural areas. The shift has occurred due to the following reasons:
Industrialization and Urbanization : The impact of industrialization and urbanization is felt everywhere. The movement of the people towards urban areas and industrial zones for better education and job opportunities has resulted in the disintegration of family from joint to nuclear. It has led to the preference for families that are smaller in size.

Westernisation : Migration of the young adults to urban centers and their adaptation of modern western – views in family life and structures have also caused a disintegration of the traditional joint family structures. The spirit of independence asserted by these young adults in terms of choice of career and spouse from other communities has added to the reasons for family disintegration.

Level of education : There is a positive correlation between the level of education and the incidence of people’s mobility to urban centers and consequently a shift from joint family to nuclear family. Thus, education has greatly affected joint family ties.

Spirit of individualism : It is one of the most important causes which has destroyed the joint family life. The individuals think of themselves rather than for the welfare of the members of whole family. It has also affected the authority structure of the family.

Economic independence of women : Women today are becoming more independent financially .The education and employment opportunities are available for women as well as men. The economic independence has increased their status, but has also affected their attitude towards authority, structure and consequently their family life.

Question 4.
(a) Analyze Tylor’s theory of Animism. [5]
(b) Discuss the relationship between religion and morality. [5]
Answer:
(a) Animism refers to belief in soul or ancestral spirits. It is a theory of religion propagated by E. B. Tylor. He believed that religion originated and was maintained on the beliefs of soul, ghosts, ancestral spirits and other things which were imagined and accepted without much real rationality in them. He named this as animism.

Analysis of Tylor’s theory of Animism: According to Tylor, belief in spirit beings (or soul) was the minimum definition of religion.
Animism has two abiding principles:

  • There is life after death;
  • There are greater and lesser spirits. It has two dimensions: life after death, and hierarchies.

Tylor’s theory has two aspects:
The argument that soul explains dreams and other psychic states: This has been criticized on the basis that dreams could not provide the emotional state needed for religious experience.

The argument that from the idea of soul comes animism, polytheism, and monotheism: This has been criticized because it must assume that primitive man remained unchanged [in his animist belief] for hundreds and thousands of years.

(b) Religion is generally understood as a belief in supernatural things and beings. It is a belief system that explains supernatural spirit which is over and above everything else.

Morality is often equated with moral code and is sometimes taken for those rules of behaviour which are admitted at large in society. In this sense, morality is equivalent to mores. Moral principles, ideas and notions are expressed in the form of set of values called morality. Morality has been a vital factor in all societies of the world, including the uncivilized, in controlling the social behaviour of man. Moral rules which prescribe the wrong or right are the very basis of our social life. Moral principles get the sanction of the society and strongly support the general system of values. Since moral ideas are inculcated into the minds of the children from the very beginning, they become habitual to obey them.

Some important relationships between religion and morality can be listed as follows:
Both religion and morality prescribe and control human behaviour. The prime objective of both religion and morality is to provide sound principles of action that will guide peoples’ way of thinking and behaving.

Each religion has a code of conduct of its own which is very often based on moral values. In other words, religion is not devoid of human values. Human values, on the other hand, are intrinsically linked to religious precepts. So, the religious precepts and ethical standards go hand-in-hand.

Moral standards draw their support from religion itself. Much of the moral practices of society are justified on the grounds of religious belief systems. Religion gives sufficient justification to existing social and moral practices.

If the moral standards are addressed to man directly, the religious standards are addressed to him indirectly. So, the violation of moral codes have direct consequences whereas, the disobedience of religious codes have indirect consequences.

Question 5.
(a) Explain the important factors responsible for changes in tribal economy. [5]
(b) Examine the basic principles of Christianity . [5]
Answer 5.
(a) The tribals are mostly engaged in various types of economies such as, food gathering economy (collecting economy), agriculture, shifting-axe cultivation, handicrafts, and pastoralism. However, a last few decades have seen rapid changes in tribal economy. Important factors responsible for changes in tribal economy :

Industrialization : The impact of industrialization is felt everywhere. The movement of the people towards industrial zones for jobs and other opportunities has great impact on tribal economy. The young tribals are no longer keen to play traditional roles in their respective places. There is a mass migration of people from tribal belts to industrial belts.

Urbanization and Modernization : The movement of the people towards urban areas for better lifestyle is another reason for the change in tribal economy. They are attracted to modem ways of living in urban centers. As a result, tribal economic structure has been disintegrating at a rapid speed.

Need for Education : There is a positive association between the level of education and the incidence of people’s mobility to urban centers. The educational opportunity has opened up the doors of possibility for almost every segment of population. Higher the level of education, greater is the mobility of the people. Many educated-tribals are working in the industries and organizations as professionals, executive officers and other skilled laborers. They have given up their traditional occupations.

Land alienation : Many tribals who lived in secluded areas, such as forests, for centuries together, have come in headlong conflict with the government as well as private organiza¬tions because of their undue encroachment. Displacement of the tribals for development works such as constructing roads, dams, establishment of towns and industries has greatly affected tribal life and economy.

Government restrictions : Tribal economy is largely dependent on shifting cultivation and use of forest resources. The restriction on the part of the modem govern-ment concerning the same, citing the reason of environmental imbalance, has curbed the progress of their traditional economy. Conse quently, many tribals are forced to move away and take up industrial labour as their occupation.

Question 6.
(a) Examine the status of women in patrilineal societies. [5]
(b) Explain the role of social movements in society. [5]
Answer:
(a) The status of women in patrilineal societies is attributed to two main reasons – protective arid preventive. For example, a Toda woman of South India, immediately after the childbirth, is not permitted to go to the fields or touch the buffaloes or to take part in religious ceremonies. As she is still assumed to be unclean, the harvest may be badly affected. However, it does not mean that the Toda women are regarded inferior or ill treated. They do enjoy certain status in the society. Though the Gond women also enjoy certain status and freedom, they are a depressed lot when it comes to choosing their husbands or in the cases involving divorce.

In the patriarchal ‘Tharu’ society, the wives are dominant and are supposed to be experts in sorcery and witchcraft. The Tham women are notorious and have plenty of influence over many things. They are usually of a loose character, but are exceptionally good looking. Among the ‘Khasa’ tribe, the women are under the constant domination of their husbands-physically, emotionally and socially. It is said that the Khasa women lead a dual life – when they are with their husbands they are completely dominated with no rights and freedom of any kind. But, whenever they go to their parents home, they are totally free from all obligations and restrictions and may even enter into physical relationships with other men. Among the Khasa tribe, the woman is not allowed to enter her husband’s house and village directly when she returns from her parents’ house. She is allowed to enter the house/village only after the purification ceremony.

Among the Nagas of Nagaland, the status of women varies from tribe to tribe. The Sema Nagas are better placed against the Ao and Angami Naga women. However, the Angami and Ao women are well placed than the Sema women in terms of properties. Sema women have no voice as far as choosing husbands is concerned, though their consent is required before marriage. The Naga women are given importance according to their ability to work and not their physical appearance.

Tibetans and Bhutias treat their women with great respect and they share equal rights with men. However, despite all these many of them are still subject to exploitation and subjugation.

Among the other Indian tribes as well, women work along with their husbands and more or less enjoy the same rights and freedom as their husbands. However, in spite of all these, they still continue to suffer from exploitation, discrimination and all forms of dehumanizing treatment.

(b) Social movements are intrinsically related to social change. Social Movement reflects, the faith that people can collectively bring about or prevent social change, if they will dedicate themselves to the pursuit of a goal. The role of social movements in society must be viewed from the perspective of the functions they perform in the society:

Social movements create social aware-ness : The social movements make people aware of the issues that affect their lives. Awareness is a key for the development of collective consciousness. It helps people to develop certain perspective towards the issues .v

Social movements facilitate collective action : Social movements are rooted in collective action. People who join the movement feel the sense of collective responsibility to seek necessary changes for the good of the society. Collective action helps to develop a sense of unity and solidarity leading to progress and development.

Social movements seek to transform unjust social structures and practices :
Social movements seek to transform the existing social structures for the better. They fight against the structures of injustice, inequality, discrimination, etc. and thus, establish a new social order which is more conducive for human life and development.

Question 7.
(a) Write a brief note on the educative functions of dormitories in tribal India. [5]
(b) Discuss the role of education in creating social change ? [5]
Answer:
(a) The functions of the tribal dormitories are mostly educative in nature. The young tribals in these dormitories enjoy their collective life with dance and music, telling stories to each other and sharing their life experiences. Dormitories are the places which provide opportunities for the young tribals to learn about their tribal customs and traditions, culture, language, religion, beliefs and ritual practices. The tribals are also given sex education so that they are able to lead a happy married life in future.

The young tribals in these dormitories are divided as seniors and juniors, and the senior members take care of the juniors and teach them about tribal way of life and tribal etiquettes.

According to the famous Indian Anthropologist, Surat Chandra Roy, ‘the dormitories help the young tribals in the gathering of edible things from the forest making them economically organized’. The dormitories also provide the young people with training to serve their community by involving them in agricultural operations, food gathering, hunting, assisting the community during marriage or during the harvest Self-discipline, social justice, respecting law and order, reward and punishment of the society are also taught in these dormitories. Since dormitories work under strict codes of discipline, the young tribals learn to live a life of discipline and self-control. Thus, tribal dormitories play a very important role in tribal society.

(b) Education is the most important factor for social change and development, because it is through education that people develop ideas, attitudes, behaviour patterns, skills and technological know-how, leading to social change and transformation.
The role of education for social change can be analyzed as follows:

Education helps to develop independent critical thinking and judgment: It is through education that the young minds develop the ability to think critically. They are able to look into various aspects of society more deeply and critically evaluate and make value judgement based on their own observations. Thus, it develops the ability to distinguish between right and wrong.

Education reforms attitudes : Education , is a means through which people reform their attitudes concerning various social beliefs and practices. As a result, the old and the obsolete ideas are replaced by newer attitudes and beliefs. Thus, education leads to abolishment of social evils and practices leading to progress of society.

Education generates ideas : It is through education that the new ideas are generated in the minds of people which lead to inventions, innovations, reformations, reorientation etc. Education results in improved vision, that govern the society and its well being, leading to development of new and improved tools for living life with ease.

Education provides skills : Skills that are developed by people in every sphere of life owe their origin to education. Education prepares a person to adapt to new skills that will be essential in today’s life. The old skills are handed, down from generation to generation through the rigorous process of education.

Education provides technological tools for development The invention of new technological tools in practically every aspect of life is engineered by education. The newer and more efficient technological tools are ever in want for the development and progress of any given society. Evolution of new and improved technologies has led to improved lifestyles.

Education imparts values: The values of life based on equality and justice are constantly reinforced in society through the process of education. It is through education that people have become more aware of their rights and duties. As a result, the society has witnessed, the emergence of new social order, where individual rights have been safeguarded.

Question 8.
(a) Discuss the various problems faced by tribals in India. [5]
(b) What are the steps taken by the Government for the upliftment of the Indian Tribes ? [5]
Answer:
(a) The numerous problems of the Indian tribals are as follows:

Problem of poverty and indebtedness : The majority of tribals are poor and even lack the basic necessities like food, clothing and shelter. Among them, the rate of indebtedness is also very high. To fulfil their basic needs, they have to take loan at a very high rate of interest. Tribals do not prefer to take loans from the banks because of tedious procedure. Also, banks can hardly provide loans for non productive purposes such as marriage, death, festivals, ceremonies, rituals, etc.

Problem of housing : The tribes are basically hunter-gatherers or engaged in shifting cultivation. So, there is a tendency among them of moving from one place to another. Usually their huts are made up of grasses, bushes, leaves, bamboo, woods etc. which are prone to fire or any sort of natural calamity. They are now suffering from the problems of non-availability of adequate forest products required for their huts. Along with this, due to increase in population, the division of homestead land is taking place leading towards the lack of housing facilities.

Problem of education : Educationally also, the tribals are backward as there are not enough educational institutions for them. Due to less educational opportunities, many of them are not able to’ find proper jobs. Educational problem in tribal area is also related to the wandering economy as practiced by them and their culture of relating education with everyday living. Due to lack of education the tribals are quite superstitious and believe in all forms of magic and totems and have remained backward for centuries together.

Problem of unemployment : The lands on which the tribals abide have now been brought under the government administra-tion. The new forest policies have snatched away the traditional rights of the tribal people over the forest. The division of land in each generation and the pressure of population on the land have reduced the employment opportunities among the tribals. In mining industries too they are facing the problem of unemployment because skilled labourers are employed and often non-tribal labour can be brought due to easy transportation.

Problem of health and sanitation : Due to isolation and lack of communication, the tribal people are not able to attain the benefits of the programmes related to the general health, reproductive health, child develop¬ment, family welfare, communicable diseases, sex-linked diseases, etc. They are also deprived of medical facilities, resulting in unsolved health issues.

(b) The steps taken by the Government for the upliftment of the Indian tribes can be listed as follows :
Training and coaching centers : To help the tribals secure good employment in future, training and coaching centers have been established by the government all over India. The young tribals are especially trained in these centers to prepare them for the examinations conducted by the Union Public Service Commission (UPSC) and State Service Commission (SSC).

Post metric scholarship : All eligible students belonging to ST/SC community, after passing their matriculation or class 10, are awarded financial assistance for post metric scholarships to pursue future education. These scholarships are given to ST/SC students who come from very low economic background. Many tribal youth have benefitted from this scheme of the government.

Hostels for ST/SC girls : The Government of India is giving financial assistance to all the state governments for the construction of hostels for ST/SC girls, especially those coming from the villages to study or work in towns. The state governments have benefited many of these
hostels and the tribal girls or women have benefited a lot.

Ministry of tribal welfare : To impart financial assistance for the tribal groups and their development, the National Democratic Alliance Government under the former Prime Minister Sri. Atal Bihari Vajpayee sanctioned 500 crores to establish the National Schedule Tribe Finance and Development Corporation. A loan up to ? 50,000 at the rate of 4% is given to the Tribals (women) by the NSTFDC. This loan, at the rate of 4% has made Tribal women financially sound.

TV programmes like’ ‘Jan Jatiya Darpan’ are telecasted on the channel Doordarshan and plenty of information regarding the tribals is made available from the websites of the Ministry of Tribal Welfare. About ? 10 crores financial assistance is provided for the preservation and maintenance of the various Tribal Sites and Historical Places. The Vajpayee Government also allotted ? 649.75 crores which was increased to ? 1090 crores for the various development programmes for the Scheduled Tribes and Castes.

Constitutional measures : The Indian Constitution provides certain safeguards for STs mainly for their educational and economic interests and also for the prevention of social distances with other communities. Some of the measures could be listed as follows:

  1. Article 16-There is equality of opportunities in matters of employment under the government.
  2. Article 29-Protection of cultural and educational rights of the community.
  3. Article 46-The state will provide special care, education and economic assistance to the weaker sections of the people provided they fall into the category of ST/SC. They will also be protected from all kinds of injustices.
  4. Article 164-Provides a Ministry of Tribal Welfare in the states of Bihar, M.P. and Orissa.
  5. Article 275-Makes provision for the granting of special funds by the Union Government to the State Government, for promoting welfare of the STs including better administration for them.
  6. Article 330/332-Reservation of seats for ST’s/SC’s in the public sector and state elections.
  7. Article 335-Assures the ST’s/SC’s that they will be given special attention while filling up vacancies for jobs.
  8. Article 339-The President of India can ask for reports regarding the administration of ST’s/SC’s areas and welfare of the ST’s/SC’s.

ISC Class 12 Sociology Previous Year Question Papers

ISC Chemistry Question Paper 2013 Solved for Class 12

ISC Chemistry Previous Year Question Paper 2013 Solved for Class 12

Maximum Marks: 70
Time allowed: 3 hours

  • Answer all questions in Part I and six questions from Part II, choosing two questions from Section A, two from Section B and two from Section C.
  • All working, including rough work, should be done on the same sheet as, and adjacent to, the rest of the answer.
  • The intended marks for questions or parts of questions are given in brackets [ ].
  • Balanced equations must be given wherever possible and diagrams where they are helpful.
  • When solving numerical problems, all essential working must be shown.
  • In working out problems use the following data:
    Gas constant R = 1.987 cal deg-1 mol-1 = 8.314 JK-1 mol-1 = 0.0821 dm3 atm K-1 mol-1. 1L atm = 1 dm3 atm = 101.3 J.
    1 Faraday = 96500 Coulombs, Avogadro’s number = 6.023 × 1023

Part – I
(Answer all questions)

Question 1.
(a) Fill in the blanks by choosing the appropriate word/words from those given in the brackets: [5]
(zero, first, second, increased, decreased, anode, cathode, active, inactive, potassium cyanide, internal, external, dependent, independent, red, benzoic acid, benzoin, common ion effect, salt hydrolysis, alkali, potassium hydroxide.)
(i) In a galvanic cell, electrons flow from ………. to ……….. through the connecting wires.
(ii) Racemic mixtures are optically …………. because of …………… compensation.
(iii) The half-life period of a …………. order reaction is …………. of the concentration of the reactant.
(iv) Benzaldehyde when treated with an alcoholic solution of ……… forms ………..
(v) The solubility of calcium oxalate is …………. in the presence of ammonium oxalate because of ……….

(b) Complete the following statements by selecting the correct alternative from the choices given: [5]
(i) The compound which is optically active is:
(1) 1-butanol
(2) 2-butanol
(3) 1-propanol
(4) 2-methyl-l-propanol

(ii) The salt which will not hydrolyse in aqueous solution is:
(1) Copper sulphate
(2) Sodium sulphate
(3) Potassium cyanide
(4) Sodium carbonate

(iii) Copper has the face centred cubic structure. The coordination number of each ion is:
(1) 4
(2) 12
(3) 14
(4) 8

(iv) For the reaction \(2 \mathrm{SO}_{2}+\mathrm{O}_{2} \rightleftharpoons 2 \mathrm{SO}_{3}\) the unit of equilibrium constant is:
(1) L mol-1
(2) J mol-1
(3) mol L-1
(4) [L mol-1]2

(v) The deficiency of vitamin D causes:
(1) Rickets
(2) Gout
(3) Scurvy
(4) Night blindness

(c) Answer the following questions: [5]
(i) Two metallic elements A and B have the following standard oxidation potentials:
A = 0.40 V, B = -0.80 V. What would you expect if element A was added to an aqueous salt solution of element B? Give a reason for your answer.
(ii) Two moles of NH3 are introduced into the one-litre flask in which it dissociates at high temperature as follows:
\(2 \mathrm{NH}_{3}(g) \rightleftharpoons \mathrm{N}_{2}(g)+3 \mathrm{H}_{2}(g)\)
Determine Kc if at equilibrium 1 mole of NH3 remains.
(iii) Give a balanced equation for the preparation of salicylaldehyde from phenol.
(iv) If the half-life period for a first-order reaction is 69.3 seconds, what is the value of its rate constant?
(v) Define cryoscopic constant.

(d) Match the following: [5]

(i) Colligative property (a) Polysaccharide
(ii) Nicol prism (b) Osmotic pressure
(iii) Activation energy (c) Aldol condensation
(iv) Starch (d) Polarimeter
(v) Acetaldehyde (e) Arrhenius equation

Answer:
(a) (i) anode, cathode
(ii) inactive, external
(iii) first, independent
(iv) potassium cyanide, benzoin
(v) decreased, common ion effect

(b) (i) (2) 2-butanol
(ii) (2) Sodium sulphate
(iii) (2) 12
(iv) (1) L mol-1
(v) (1) Rickets

(c) (i) Since the oxidation potential of A is higher than that of B, therefore, metal A will displace metal B from its salt solution.
ISC Chemistry Question Paper 2013 Solved for Class 12 image - 1
ISC Chemistry Question Paper 2013 Solved for Class 12 image - 2
(v) Cryoscopic constant: It is defined as the depression in freezing point of the solvent produced on dissolving one mole of a substance in 1000 g of it. It is also called molal depression constant.

(d) (i) Colligative property → (b) Osmotic pressure
(ii) Nicol prism → (d) Polarimeter
(iii) Activation energy → (e) Arrhenius equation
(iv) Starch → (a) Polysaccharide
(v) Acetaldehyde → (c) Aldol condensation

Part – II

Answer six questions choosing two from Section A, two from Section B and two from Section C.

Section – A
(Answer any two questions)

Question 2.
(a) (i) Ethylene glycol is used as an antifreeze agent. Calculate the amount of ethylene glycol to be added to 4 kg of water to prevent it from freezing at -6°C. (Kf for H2O = 1.85 K mole-1 kg) [3]
(ii) The freezing point of a solution containing 0.3 g of acetic acid in 30 g of benzene is lowered by 0.45 K. Calculate the Van’t Hoff factor.
(at. wt. of C = 12, H = 1, O = 16, Kf for benzene = 5.12 K kg mole-1) [2]
(b) Name the law or principle confirmed by the following observations:
(i) When water is added to 0.01 M aqueous solution of acetic acid the number of hydrogen ions increase. [2]
(ii) When 96500 coulombs of electricity is passed through acidulated water, 5.6 litres of oxygen at s.t.p. is liberated at the anode. [1]
(c) Arrange Ag, Cr and Hg metals in the increasing order of reducing power. Given: [1]
ISC Chemistry Question Paper 2013 Solved for Class 12 image - 25
(d) In a first-order reaction, 10% of the reactant is consumed in 25 minutes. Calculate: [2]
(i) The half-life of the reaction.
(ii) The time required for completing 17% of the reaction.
Answer:
ISC Chemistry Question Paper 2013 Solved for Class 12 image - 3
ISC Chemistry Question Paper 2013 Solved for Class 12 image - 4
ISC Chemistry Question Paper 2013 Solved for Class 12 image - 26

Question 3.
(a) Explain giving reasons why (Give equations in support of your answer):
(i) A solution of NH4Cl and NH4OH acts as a buffer. [2]
(ii) Cu is precipitated as CuS while Zn is not precipitated when H2S is passed through an acidic solution of Cu(NO3)2 and Zn(NO3)2 respectively. [2]
(b) (i) What is Schottky defect in a solid? [1]
(ii) A bcc element (atomic mass 65) has a cell edge of 420 pm. Calculate its density in g/cm3 [3]
(c) The rate of the reaction \(\mathrm{H}_{2}+\mathrm{I}_{2} \rightleftharpoons 2 \mathrm{HI}\) is given by:
Rate = 1.7 × 10-19 [H2] [I2] at 25°C. [2]
The rate of decomposition of gaseous HI to H2 and I2 is given by:
Rate = 2.4 × 10-21 [HI]2 at 25°C.
Calculate the equilibrium constant for the formation of HI from H2 and I2 at 25°C. [2]
Answer:
(a) (i) NH4Cl is a strong electrolyte hence, dissociates completely. NH4OH is a weak electrolyte and dissociates to a small extent. Its dissociation is further suppressed by common ion \(\mathrm{NH}_{4}^{+}\) provided by NH4Cl in the solution.
\(\begin{array}{c}{\mathrm{NH}_{4} \mathrm{Cl}_{(a q)} \rightleftharpoons \mathrm{NH}_{4(a q)}^{+}+\mathrm{Cl}_{\max }^{-}} \\ {\mathrm{NH}_{4} \mathrm{OH}_{(a q)} \Longrightarrow \mathrm{NH}_{4(a q)}^{+}+\mathrm{OH}_{(a q)}^{-}}\end{array}\)
This solution acts as a basic buffer and maintains its pH around 9.25. It resists the change in pH on the addition of a small amount of acid or alkali. This can be explained as below:
Upon adding a small amount of HCl to this solution, H+ ions of HCl gets neutralised by OH ions already present and more of NH4OH molecules get ionised to compensate for the loss of OH ions. Thus, pH practically remains unchanged.
Upon adding a small amount of NaOH to this solution, OH ions of NaOH combine with \(\mathrm{NH}_{4}^{+}\) ions already present to form weakly ionised NH4OH. Thus, pH of the solution remains practically unchanged.
(ii) This is because the Ksp of CuS is very low as compared to that of ZnS. Therefore, a low concentration of S2- ions can precipitate Cu2+ ions as CuS, whereas a high concentration of S2 ions is required to precipitate Zn2+ ions as ZnS.
Now, in the presence of HCl, dissociation of H2S is suppressed due to the common ion effect.
HCl → H+ + Cl (strong electrolyte)
H2S → 2H+ + S2- (weak electrolyte)
With decreased S2- ion concentration only Cu2+ is precipitated as CuS.

(b) (i) Schottky defect: It is a stoichiometric defect in which the equal number of cations and anions in the ionic crystal are missing from their lattice sites and electrical neutrality is maintained.
ISC Chemistry Question Paper 2013 Solved for Class 12 image - 5
ISC Chemistry Question Paper 2013 Solved for Class 12 image - 6

Question 4.
(a) (i) Give Lewis’definition of acids and bases. [1]
(ii) The solubility of Ag2CrO4 at 25°C is 8.0 × 10-5 moles litre-1. Calculate its solubility product. [1]
(b) (i) Define molar conductance of a solution. State its unit. How is it related to the specific conductance of a solution? [2]
(ii) Calculate the value of Ecell at 298 K for the following cell: [3]
ISC Chemistry Question Paper 2013 Solved for Class 12 image - 7
(c) (i) Calculate the degree of hydrolysis of 0.2 (M) sodium acetate solution. [1]
(Hydrolysis constant of sodium acetate = 5.6 × 10-10 and ionic product of H2O = 10-14 at 25°C)
(ii) Explain why high pressure is used in the manufacture of ammonia by Haber’s process. State the law or principle used. [2]
Answer:
(a) (i) A Lewis acid is defined as a substance (atom, ion or molecule) which is capable of accepting a pair of electrons.
Examples: Ag+, H+, AlCl3.
A Lewis base is defined as a substance (atom, ion or molecule) which is capable of donating a pair of electron.
Examples: NH3, Cl.
(ii) Let the solubility of Ag2CrO4 in s mol L-1
ISC Chemistry Question Paper 2013 Solved for Class 12 image - 8

(b) (i) Molar conductance: It is defined as the conductance of all ions produced from one mole of an electrolyte dissolved in a definite volume of the solution.
Its unit is ohm-1 cm2 mol-1. Molar conductance ∧m is related to specific conductance k by the following equation:
\(\wedge_{m}=\frac{k \times 1000}{\mathrm{M}}\)
where k is specific conductance and M is a molar concentration of the solution.
ISC Chemistry Question Paper 2013 Solved for Class 12 image - 9
ISC Chemistry Question Paper 2013 Solved for Class 12 image - 10
The forward reaction is accompanied by a decrease in the number of moles. On increasing pressure, then according to Le-Chatelier’s principle, the equilibrium will shift in a direction in which pressure decreases i.e., decrease in a number of moles takes place i.e., favour the formation of ammonia. Or in other words, an increase in pressure will favour the formation of ammonia. This is in accordance with Le-Chatelier’s principle, the law states ‘If a system at equilibrium is subjected to a change of concentration, temperature and pressure, then the equilibrium shifts itself in a direction that tends towards the effect of change imposed.’

Section – B
(Answer any two questions)

Question 5.
(a) Give the IUPAC names of the following coordination compounds: [2]
(i) K2[Zn(OH)4]
(ii) [CO(NH3)5 (CO3)] Cl.
(b) For the complex ion [Fe (CN)6]3- state: [1]
(i) The geometry of the ion.
(ii) The magnetic property of the ion.
(c) What type of structural isomers are [Co(NH3)5 Br] SO4 and [Co(NH3)5 SO4]Br? Give a chemical test to distinguish the isomers. [2]
Answer:
(a) (i) Potassium tetrahydroxozincate (II)
(ii) Pentaamminecarbonatocobalt (III) chloride

(b) (i) Octahedral geometry
(ii) Ion is paramagnetic.
(c) [Co(NH3)5 Br] SO4 and [Co(NH3)5SO4] Br are ionisation isomers.
The aqueous solution of [Co(NH3)5 Br] SO4 will give a white ppt. with BaCl2 solution insoluble in conc. HNO3, confirming the presence of free \(\mathrm{SO}_{4}^{2-}\) ion. An aqueous solution of [Co(NH3)5SO4] Br will not give this test.
ISC Chemistry Question Paper 2013 Solved for Class 12 image - 11

Question 6.
(a) For the molecule XeF2: [2]
(i) Draw the structure of the molecule indicating the lone pairs.
(ii) State the hybridisation of the central atom.
(iii) State the geometry of the molecule.
(b) Give balanced chemical equations for the following reactions: [2]
(i) Fluorine treated with dilute sodium hydroxide reactions.
(ii) Hydrogen sulphide treated with concentrated sulphuric acid.
(iii) Potassium iodide treated with acidified potassium permanganate solution.
Answer:
ISC Chemistry Question Paper 2013 Solved for Class 12 image - 12

Question 7.
(a) In the extraction of zinc from zinc blende: [2]
(i) Give an equation to show how zinc oxide is converted to zinc.
(ii) How is impure zinc finally electro-refined?
(b) Explain why:
(i) Transition elements form coloured compounds. [2]
(ii) Interhalogen compounds are more reactive than their constituent elements.
(iii) Cu+ is diamagnetic but Cu2+ is paramagnetic. (Z = 29)
Answer:
ISC Chemistry Question Paper 2013 Solved for Class 12 image - 13
(ii) Electro-refining of zinc:
The electrolyte used: ZnSO4 solution containing little H2SO4.
Anode: Block of impure zinc.
Cathode: Thin sheet of pure zinc.
On passing current zinc is obtained at the cathode.
ISC Chemistry Question Paper 2013 Solved for Class 12 image - 14

(b) (i) Transition metal ions generally possess one or more unpaired electrons. By absorbing a requisite amount of energy from visible light, the unpaired electrons present in the lower energy d-orbital get promoted to high energy d-orbitals. This is called a d-d transition. The energy involved in d-d transition is very small and such radiations are present within the visible region of light. Transition metal ions absorb such radiations from visible light and remaining wavelengths present in the visible region get transmitted and this imparts colour.
(ii) Interhalogen compounds are more reactive than their constituent elements because X-Y bonds present between two different halogens in them are weaker than X-X or Y-Y bonds.
(iii) Cu+ is diamagnetic because it has no impaired electrons in its core.
ISC Chemistry Question Paper 2013 Solved for Class 12 image - 16

Section – C
(Answer any two questions)

Question 8.
(a) How can the following conversions be brought about: [3]
(i) Nitrobenzene to benzene diazonium chloride.
(ii) Propanoic acid to ethylamine.
(iii) Benzoic acid to benzaldehyde. [2]
(b) Identify the compounds A, B, C, D, E and F: [3]
ISC Chemistry Question Paper 2013 Solved for Class 12 image - 17
Answer:
ISC Chemistry Question Paper 2013 Solved for Class 12 image - 18

Question 9.
(a) (i) Acetamide is heated with bromine and sodium hydroxide solution. [1]
(ii) Benzaldehyde is treated with 50% sodium hydroxide solution.
(b) Give one chemical test to distinguish between the following pairs of compound: [2]
(i) Acetone and phenol.
(ii) Formic acid and acetic acid.
(c) (i) Name the type of isomerism exhibited by the following pairs of compound: [2]
(1) (C2H5)2NH and CH3-NH-C3H7
(2) 1 – butanol and 2 methyl-1 -propanol.
(ii) Name the type of isomerism that the compound with molecular formula C3H6O2 exhibits. Represent the isomers.
Answer:
(a) (i) Methylamine is formed.
ISC Chemistry Question Paper 2013 Solved for Class 12 image - 19
This is Hoffmann’s bromamide reaction.
(ii) Sodium benzoate and benzyl alcohol are formed.
ISC Chemistry Question Paper 2013 Solved for Class 12 image - 20
This is Cannizzaro’s reaction.
(b) (i) Acetone, when heated with NaOH/I2, gives a yellow crystal of iodoform.
ISC Chemistry Question Paper 2013 Solved for Class 12 image - 21
Phenol does not give this test.
(ii) Formic acid gives a silver mirror with tollen’s reagent.
ISC Chemistry Question Paper 2013 Solved for Class 12 image - 22
Acetic acid does not give this test.

Question 10.
(a) Write balanced chemical equations for the following reactions: [4]
(i) Oxalic acid is treated with acidified potassium permanganate solution.
(ii) Benzoic acid is treated with a mixture of concentrated nitric acid and concentrated sulphuric acid.
(iii) Methyl magnesium iodide is treated with carbon dioxide and the product hydrolysed in acidic medium.
(iv) Ethylacetate is treated with ammonia.
(b) An organic compound [A] having molecular formula C2H7N on treatment with nitrous acid gives a compound [B] having molecular formula C2H6O. [B] on treatment with an organic compound [C] gives a carboxylic acid [D] and a sweet-smelling compound [E]. Oxidation of [B] with acidified potassium dichromate also gives [D]. [4]
(i) Identify [A], [B], [C], [D] and [E].
(ii) Write a balanced chemical equation of [D] with chlorine in the presence of red phosphorus and name the reaction.
(c) Acetamide is amphoteric in nature. Give two equations to support this statement. [2]
Answer:
ISC Chemistry Question Paper 2013 Solved for Class 12 image - 23
ISC Chemistry Question Paper 2013 Solved for Class 12 image - 24

ISC Class 12 Chemistry Previous Year Question Papers

ISC Sociology Question Paper 2015 Solved for Class 12

ISC Sociology Previous Year Question Paper 2015 Solved for Class 12

Maximum Marks: 70
Time allowed: Three hours

  • Candidates are allowed additional 15 minutes for only reading the paper. They must NOT start writing during this time.
  • Answer Question 1 from Part I and five questions from Part II,
  • The intended marks for questions or parts of questions are given in brackets [ ].

Part-I (20 Marks)
Answer all questions.

Question 1.
Answer briefly each of the following questions : [10 x 2]
(i) What is marriage by trial ?
(ii) What is gotra ?
(iii) What are the terms ‘amitate’, ‘avoidance’ and ‘couvade ’ known as ?
(iv) Explain the term ‘multiple possessory rights’.
(v) What is meant by sororate ?
(vi) How are capital goods and consumer goods different from each other ?
(vii) Mention two negative effects of mass media.
(viii) What is meant by shifting axe cultivation ?
(ix) Mention any two social reforms that led to the change in the status of women in Pre- British and British India.
(x) According to Frazer, what are the different types of magic ?
Answer:
(ii) Gotra refers to a line of descent usually traced through the male line amongst the Hindus. People who are believed to have descended from the same Rishi are considered to belong to the same gotra and are required to practice Gotra exogamy as a rule of marriage when it comes to the question of choosing spouses.

(iii) Amitate : When there is a special role for the father’s sister then it is known as amitate. Fathers’ sister is given more’respect than the mother.

Avoidance: It is a kinship term in which some kins are required to maintain distance from each other, with the view of prohibiting the creation of any form of incestuous relationship or creation of conflict due to clash of roles.
Eg: Brothers and sisters or parents-in-law and the daughter-in-law.

Couvade : It is a queer kinship found among the primitive tribes such as the Toda and the Khasi. Under this, the husband has to lead a life of an invalid along with his wife and has to take diets meant for the sick, when his wife is pregnant. He has to observe the same taboos as are observed by his wife.

(v) Sororate : Sororate is a form of marriage in which a man can marry the younger sister of his deceased wife. This type of marriage is mostly found to prevail amongst those tribes where there is an incidence of high bride price.

(vi) Two negative impacts of mass media are as follows :

  • Easy availability of non-traditional literature and graphic content. This in turn can harm the personality development process of the younger generation.
  • False news can often be created and circulated that can help to spread rumors and promote vandalism.

(viii) Shifting Axe Cultivation : Shifting Cultivation refers to the form of cultivation where a large area of land is cultivated for few years and then abandoned for some time until, the fertility of the land is restored naturally. This type of cultivation is practiced in tropical regions and consists of felling of trees on a hillside a little before the sowing season and setting them on fire.

Part-II (50 Marks)
Answer any five questions.

Question 2.
(a) Discuss Consanguine kinship and Affinal kinship. [5]
(b) Define degrees of kinship. Explain the three degrees of kinship giving one example of each. [5]
Answer:
(a) Consanguineous kinship : The kinship arising from birth or out of the blood relation ties is called consanguineous kinship. The relatives out of this kinship are known as consanguineous kin. Hence, the relationship between the parents and children and between the siblings is known as consanguineous kinship. Thus, son, daughter, father, mother, brother, sister, uncle, cousin are consanguineous kin. Besides the biological consideration, social recognition is also important, for instance, among the tribes of Melanesia, wife’s husband is the father of the child. Today perform the bow and arrow ceremony to give legitimacy to the child bom out of several brothers since the community practices levirate marriages.

At Tinial kinship : The kinship arising out of the bond of marriage through socially sanctioned and legally defined marital relationships, is called affinial kinship. When a person marries, his or her spouse’s relatives also get related to him or her. Hence, on getting married a woman becomes not only a wife but also a daughter-in-law, a sister-inlaw, aunt and so on (bhabhi, devrani, jethani, mami, etc). Similarly, the man on getting married becomes a husband, a son-in-law, a brother-in-law, uncle etc. (sala, nandoi, mausa, fufa, jija, sadhu, etc). .

(b) Dividing or classifying the relatives on the basis of nearness or distance of relationship with them is called degree of kinship. There are three degrees of kinship’s based on the relationship with them :

Primary kin : Those relatives with whom the relationship is very close, direct and near are called primary kin. They can be classified more minutely into primary consanguineous kin or primary affinal kin.
For example: father-son, husband-wife and brother-sister.

Secondary kin : They are primary kin of primary kin. In other words, they are related through primary kin. They are not our primary kin but are the primary kin of our primary kin, hence our secondary kin.
For example, father’s brother (chacha), sister’s husband (bahnoi) are secondary kin. The father is my primary kin and his brother is the primary kin of father. Therefore, father’s brother is my secondary kin, the primary kin of primary kin. Similarly, sister is my primary kin but her husband is my secondary kin.

Tertiary kin : They are the secondary kin of our primary kin or primary kin of our secondary kin. Thus, the wife of brother-in- law(sala) called sarhaj in Hindi is tertiary kin because brother-in- law is my secondary kin and his wife is the primary kin of brother-inlaw. Similarly the brother-in-law of my brother is my tertiary kin because the brother is my primary kin and his brother-in-law is the secondary kin of my brother.
ISC Sociology Question Paper 2015 Solved for Class 12 1
Question 3.
(a) Describe any five ways of acquiring mates in primitive societies [5]
(b) Examine polygamy as one of the forms of marriage. [5]
Answer:
(b) Polygamy is the practice of marriage to more than one person at the same time. In tribal India, polygamy is wide spread and differentiated into two forms :

Polygyny : is the marriage of one man to several women. If all the wives are reated to each other as sisters than it is termed as Sororal polygyny. In many societies amongst the tribals, having more than one wife was seen a symbol of power and prestige. Whereas amongst some, the barrenness of the first wife led to a second marriage of the husband. This is a more common form of polygamous marriage.

Polyandry : is the marriage of the woman to several men. Polygamy is found among Naga tribes, Gond and Baiga, the Toda, the Lushei clans. Polyandry is comparatively restricted in distribution. Polyandry can be termed as Fraternal or Adelphic polyandry if all the men in the marriage are related to each other as brothers and share a common wife. If the several husbands of the woman are not brothers of each other than it is known as non- fratemal polyandry. Polyandry was practiced amongst some tribes in order to prevent the creation of tiff amongst the several wives post marriage and also sometimes to prevent the sharing of property. Polyandry is a less common form of polygamous marriage.

Question 4.
(a) Functions of a family have undergone several changes in modern times. Discuss any five such changes. [5]
(b) Make a comparative study between religion and science. [5]
Answer:
(a) Following are the changes which have taken place in the functions of the family in the modem times :

The significant function of a family from a production and consumption unit has changed. It has mainly become a consumption unit which uses the goods and services.

The educational function of a family has been taken by the specialized agencies such as schools, colleges, etc.

The health related functions such as the care of aged and sick people were once done at home; even the delivery of the child was done at home. Now these functions have been taken over by the specialized agencies such as hospitals and old age homes.

The recreational function of a family through celebration of festivals and happy moments of life have also changed. The recreation has mainly become individualistic through movies, parks, etc.

The family once performed religious function of acquainting the child with the family customs and religious rituals through idol worship, prayers and sanction. Today, however, more specialized agencies are involved in religious functions such as temple, religious events, television, etc.

(b) Comparison between Religion and Science : Religion is incompatible with science and there is a continuous conflict between the two. While science is based on the evidence, religion is based on faith. According to science, all events occur due to natural causes while religion believes that behind every action there is some unseen supernatural or divine power. Science believes in what it sees and religion believes in what it believes. Science helps in solving the day to day problems by introducing new inventions.

It is important to note that the dogmatic religion has always opposed science and interfered with its development .Religion also opposes free enquiry. Science though deals with more realistic objects, but it has also not solved all the problems of life. The importance of religion cannot be denied, where science fails religion brings hope. Religion is important as long as there is suffering in life. It helps to counteract the stress and strains of modern life. There is reconcilable difference between science and religion.

Question 5.
(a) ‘Religion is a pacifying agent, but can also act as a divisive force Justify this statement with three functions and two dysfunctions of religion. [5]
(b) Define the word ‘Education’. State any five main functions of education. [5]
Answer:
(a) Religion is indeed a pacifying agent as it helps to resolve the inner conflicts and fills the life with hope. However, it also acts as a diversifying force.
Following are some of the functions of religion :

Religion explains all the human sufferings : Religion acts as a pacifying agent in times of sufferings as it soothes the emotions during the disappointments and helps in integration of the personality of a person. It sustains interest in life and helps in making it bearable.

It helps in enhancing self importance : It helps the person to believe in his capabilities and encourages him to play a significant role in the society.

It is a valuable source of social cohesion: Religion is the foundation upon which all the social values rest; which plays role in controlling individual’s actions. Hence, it helps in binding the society by giving common values.

Following are some of the dysfunctions of religion :
A Detrimental force : Religion has often proved to be a catastrophic force as many times wars are fought in the name of religion. Many times it stands in the way of national integration. For example, Mandir-Masjid issue in Ayodhya has brought a disastrous effect on the national integration.

Religion promotes fanaticism : People . following a particular religion often have a
fanatic approach to the religion to which they belong and it can often promote a sense of superiority whereby people then tend to consider religion of all others as inferior. This can create a sense of discontent often leading to outbreak of communal violence amongst the people.

(b) Education : Education is derived from the Latin word e due are which means to bring up. Thus, education can be defined as the process through which the person is brought up, in order to develop in him/her those habits and abilities which help him/her to face the future in a worthwhile manner.
Following are some of the functions of education:
To complete the process of socialization : It is through education that a person learns to differentiate between right and wrong and learns basic human values.

Transmission of cultural heritage : It is through education that culture is transmitted from one generation to another in the form of art, literature, text, religion and philosophy.

Reformation of attitudes : Education helps in reformation of already framed attitudes, so that the child can develop right attitudes and beliefs.

Occupational role : Education always has a utilitarian end and it enables the person to earn livelihood by doing a productive task.

To instill a sense of competition : The function of education is to inculcate the sense of competition, so that the student can work hard in order to perform well.

Question 6.
(a) Discuss the nature of primitive economies. [5]
(b) Define status. How has Low described the determinants of status ? [5]
Answer:
(a) Primitive economic systems have characteristics that make them structurally different from modem economic system :
They are :
Inadequate, ineffective and wasteful use of nature (in the absence of advanced techno¬logy), (For example shifting cultivation where agricultural surplus is also not created).

Money as a store of value and a medium of exchange is not widely used in primitive society. Therefore modern concepts such as banking and credit are absent. Relations in primitive society are based on barter.

The profit motive of economic dealings is absent in primitive economies. Mutual obli¬gation, sharing and solidarity are important.

The co-operative and collective efforts are important in these economies which are communal economies.

The rate of innovation is very low in these economies. Therefore there is stability and uniformity due to simplicity and uniformity of the techniques used.

The market as an institution is absent. Weekly markets or seasonal markets exist. Therefore competition and monopology are absent.

Economic activities in primitive societies are directed largely to the manufacture of consumption and not production of goods.

Question 7.
(a) Discuss the difference between primitive law and modern law. [5]
(b) Write a short note on the history of the Narmada Bachao Andolan. [5]

Question 8.
(a) Contact with wider society has led to tribal transformation. Analyze the given statement. [5]
(b) Describe the cultural classification of tribes. [5]
Answer:
(a) A ‘Contact with the wider society has led to tribal transformation’. This statement is true only to a little extent. The condition of the tribals inspite of modem means of education, science and technology, is same, pathetic and backward. Though a number of schemes and development programs are implemented for the welfare of the tribal, the results are still insignificant. On one hand, caste based inequity is still prevalent. On the other hand, tribal regions are simmering with discontent.

Technological developments instead of helping them to march towards development are making them more deprived and are ravaging them with increased severity. The situation in some of the tribal areas is especially disturbing such as the demand for separate Bodo land. Their virtually forced displacement for the sake of so called development projects is making the condition worst for them, making them restless and insecure.

Neither the reservation policy, nor the ban of untouchability has helped in betterment of their position and their situation is same over the years. They are still subject to exploitation and severe kind of discrimination. The reasons for their deprived condition are that mainly they are suspicious towards the development as they think it might lead to shattering of their identity and might snatch their linguistic unity and territorial uniqueness. Secondly, though several development programs are there for the welfare of the tribal, they are hardly aware about them due to their location in the remote areas, illiteracy, lack of means of communication and linguistic barrier. In fact the policies are used by the people of the creamy layer to fulfill their selfish motives. They misuse them by making fake identity claims through forged documents.

(b) Cultural classification of tribes in India : Tribals were classified on the basis of their cultural distance from rural and urban groups. This comparative approach is considered to be important in evolving a plan for rehabilitation because it focuses on those problems of tribal India, which are the outcome of haphazard contact with or isolation of tribes from the rural urban population.
Tribal Culture can be :
Assimilative
Adaptive : Adaptive further consist of:

  • Commensalic.
  • Symbiotic
  • Acculturative.

Commensalic : Those with common economic pursuits with their neighbors, and thus their growth is arrested.

Symbiotic : Their relationship with their neighbors is based on interdependence and acculturation is haphazard.

Acculturative : There is a one way flow of culture traits, where tribal’s are taking in culture traits from the surrounding rural and urban groups. This leads to a situation of culture crisis leading to sudden cultural changes.

According to Majumdar all three of the above are in a state of culture crisis. The main criticism to this theory lies in the fact that though this classification could help in the rehabilitation of tribals it does not include cultures that are culturally distant from rural and urban groups.

ISC Class 12 Sociology Previous Year Question Papers